Download as pdf or txt
Download as pdf or txt
You are on page 1of 156

Q1.

orthodeoxia is charecteristically seen in ;; 1 copd;; 2 chronic heart failure;; 3 pul embolism;; 4 hepato-pul syndrome ans=hepatopulmonary syn ref=harrison 17e chapter 295 The hepatopulmonary syndrome is characterized by platypnea and orthodeoxia, representing shortness of breath and oxygen desaturation that occur paradoxically upon assuming an upright position. Fall in arterial blood oxygen upon assuming the upright posture. Usually due to right-to-left cardiac or vascular shunting with a posturally induced fall in left sided pressure permitting a corresponding gradient across the shunt. An anatomical component in the form of an interatrial communication and a functional component that produces a deformity in the atrial septum and results in a redirection of shunt flow with the assumption of an upright posture. The former may be an atrial septal defect, a patent foramen ovale, or a fenestrated atrial septal aneurysm. The latter may be cardiac, such as pericardial effusion or constrictive pericarditis; pulmonary, such as emphysema, arteriovenous malformation, pneumonectomy, or amiodarone toxicity; abdominal, such as cirrhosis of the liver or ileus; or vascular, such as aortic aneurysm or elongation Q2. allosteric inhibition with atp affects;; 1 phosphofructokinase;; 2 phospho-eno-pyruvate;; 3 g-pd;; 4 pyruvate kinase A good example is the allosteric regulation of the glycolytic pathway. This catabolic pathway consumes glucose and produces ATP, NADH and pyruvate. A key step for the regulation of glycolysis is an early reaction in the pathway catalysed by phosphofructokinase-1 (PFK1). When ATP levels rise, ATP binds an allosteric site in PFK1 to decrease the rate of the enzyme reaction; glycolysis is inhibited and ATP production falls. This negative feedback control helps maintain a steady concentration of ATP in the cell. However, metabolic pathways are not just regulated through inhibition since enzyme activation is equally important. With respect to PFK1, fructose 2,6bisphosphate and ADP are examples of metabolites that are allosteric activators. Studies on ATP effects on the allosteric kinetics shown by pyruvate kinase from Phycomyces blakesleeanus NRRL 1555 (-) are reported. Phosphoenolpyruvate showed an allosteric ATPdependent substrate inhibition. The results supported the existence of spatially distinct catalytic binding sites and the inhibitory binding sites for phosphoenolpyruvate, and ATP showed opposite heterotropic effects with respect to these two types of binding site. With respect to Mg2+ ions, ATP caused a negative heterotropic effect. The global inhibitory effect of ATP was in agreement with the predictions postulated by the two-state concerted-symmetry model of Monod, Wyman and Changeux so Both PFK1 and phosphoenolpyruvate are affected by allosteric inhibition by ATP Pyruvate kinase is the third regulated enzyme of glycolysis. Like PFK, pyruvate kinase is regulated both by allosteric effectors and by covalent modification (phosphorylation). Pyruvate kinase is activated by F-1,6-BP in the liver, a second example of feedforward stimulation. ATP and alanine (a biosynthetic product of pyruvate) act as allosteric inhibitors of pyruvate kinase.

Phosphorylation of pyruvate kinase is regulated by blood glucose level, just like PFK. High glucagon (low blood sugar) causes phosphorylation, which in this case renders the enzyme inactive. Q3. WHAT IS THE MELANOCYTE -KERATINOCYTE RATIO;; 1 1-1;; 2 1-10;; 3 1-28;; 4 1-36 The melanocyte-keratinocyte ratio varies in different areas of the body and an average ratio of 1 : 36 answer is 1:36 Q4. NEOADJUVANT CHEMOTHERAPY IS USEFUL IN ALL OF THE FOLLOWING CANCERS, EXCEPT ;; 1 BREAST CARCINOMA;; 2 OSTEOSARCOMA;; 3 OVARIAN CARCINOMA;; 4 HEPATOCELLULAR CARCINOMA Q5. TARDIEUS SPOTS ARE SEEN IN DEATH DUE TO ;; 1 HAEMORRHAGE;; 2 VASOVAGAL SHOCK;; 3 BRAIN TUMOR;; 4 OBSTRUCTION OF RESPIRATORY PASSAGES ANS= OBSTRUCTION OF RESPIRATORY PASSAGES Tardieu's spots= spots of ecchymosis under the pleura after death by suffocation. Q6. luxury perfusion phenomena is seen in ;; 1 thrombolytic therapy;; 2 arf;; 3 encephalitis;; 4 renal artery stenosis ANS=ENCEPHALITIS( specifically arboviral encephalitis) The disease process in the CNS arises partly from direct neuronal infection and subsequent damage and partly from edema, inflammation, and other indirect effects. The usual pathologic picture is one of focal necrosis of neurons, inflammatory glial nodules, and perivascular lymphoid cuffing; the severity and distribution of these abnormalities vary with the infecting virus. Involved areas display the "LUXURY PERFUSION" phenomenon, with normal or increased total blood flow and low oxygen extraction. Q7. in which of the following sites is turbulent flow most likely to occur;; 1 ascending aorta;; 2 small arteries;; 3 capillaries;; 4 small veins;; ANS= ASCENDING AORTA Turbulent flow means that the blood flows crosswise in the vessel as well as along the vessel, usually forming whorls in the blood called eddy currents.

Reynolds number is the measure of the tendency for turbulence to occur( Re=(density* velocity*diameter)/viscosity Reynolds number for flow in the vascular system even normally rises to 200 to 400 in large arteries; as a result there is almost always some turbulence of flow at the branches of these vessels. In the proximal portions of the aorta and pulmonary artery, Reynolds number can rise to several thousand during the rapid phase of ejection by the ventricles; this causes considerable turbulence in the proximal aorta and pulmonary artery where many conditions are appropriate for turbulence: (1) high velocity of blood flow, (2) pulsatile nature of the flow, (3) sudden change in vessel diameter, and (4) large vessel diameter. However, in small vessels, Reynolds number is almost never high enough to cause turbulence. Q8. provenge is a vaccine treatment for which of the following cancers;; 1 hepatoma;; 2 multiple myeloma;; 3 prostate cancer;; 4 hodgkins lymphoma ans=prostate cancer In a widely anticipated announcement, the US Food and Drug Administration (FDA) has approved an immunotherapy, sipuleucel-T (Provenge, Dendreon), for the treatment of asymptomatic or minimally symptomatic metastatic, castration-resistant prostate cancer (mCRPC). Often referred to as a vaccine, sipuleucel-T is an autologous active cellular immunotherapy, meaning that it is made from the patient's own white blood cells and stimulates a patient's immune system to respond against the cancer. The treatment needs to be manufactured individually for each patient. "The availability of [sipuleucel-T] provides a new treatment option for men with advanced prostate cancer who currently have limited effective therapies available," said Karen Midthun, MD, acting director of the FDA's Center for Biologics Evaluation and Research, in a press statement. Sipuleucel-T becomes the first product approved by the FDA in a new therapeutic class known as active cellular immunotherapies, according to Dendreon press materials. To be approved by the FDA, investigators had to show that sipuleucel-T would extend survival in men with mCRPC. The latest data from the pivotal trial, known as IMPACT (Immunotherapy for Prostate Adenocarcinoma Treatment), were presented at the American Society of Clinical Oncology 2010 Genitourinary Cancers Symposium in March. Finally, FDA approved the vaccine on 29 April 2010, for use in the treatment of advanced prostate cancer. According to the FDA press release, the most common adverse reactions reported with the new product are chills, fatigue, fever, back pain, nausea, joint ache, and headache. The majority of adverse reactions were mild or moderate. Serious adverse reactions, which were reported in approximately one quarter of the patients receiving sipuleucel-T, included some acute infusion reactions and stroke. Cerebrovascular events, including hemorrhagic and ischemic strokes, were observed in 3.5% of patients in the sipuleucel-T group, compared with 2.6% of patients in the placebo control group Q9. zebra bodies are seen in ;; 1 alport syndrome;; 2 fabrys disease;; 3 renal amyloidosis;;

4 multiple myeloma; Zebra bodies: Under electron microscope, they are bound by a single membrane, concentric or lamellated with a periodicity of 5.6-5.8 nm. Zebra bodies are not unique to metachromatic leukodystrophy and can also be seen in Tay-Sachs disease, Farbrys disease, mucolipidoses and mucopolysaccharidoses. Q10. the abuse liability is greatest with ;; 1 caffeine;; 2 cocaine;; 3 benzodiazepines;; 4 marijuana;; Q11. transesophageal echocardiography combines ultrasonography with which other procedure;; 1 electrocardiogram;; 2 ercp;; 3 endoscopy;; 4 none of the above;; Q12. MIESCHERS GRANULOMA IS SEEN IN THE HISTOPATHOLOGY OF ;; 1 erythema nodosum;; 2 erythema induratum;; 3 nodular vasculitis;; 4 erythema marginatum Q13. work done in respiratory system can be calculated by ; 1 bohr equation;; 2 alveolar gas equation;; 3 single breath nitrogen technique;; 4 relaxation -pressure curve Q14. maximum number of days during which a pt can be kept for observation for diagnosis of mental illness is ;; 1 10;; 2 14;; 3 20 4 30;; Q15. ANOTHER WEIRD Q . WHOSE PERFECT ANSER IS NOT KNOWN TILL TODAY. WHICH IS NOT INVOLVED IN CARCINOGENESIS OF COLON;; 1 APC;; 2 K-RAS;; 3 BETA CATENIN;; 4 MISMATCH REPAIR

Back to top

Posted: Fri Jun 11, 2010 5:50 pm Post subject:

Q16. the first function to be lost during the onset of spinal anaesthesia is ;; 1touch;; 2temperature;; 3vibration;; 4autonomic activity;

Q17. the following are the branches of the internal iliac artewry , except ;; 1ovarian artery;; 2uterine artery;; 3sup vesical artery;; 4obturator artery;; Q18. dry submarine in custodial torture injuries refers to ;; 1falanga-beating of soles of feet;; 2tying of a plastic bag over the years;; 3immersing head in contaminated water;; 4suspending the accused by ankles; Q19. the following are radiological features of sigmoid vovulus, except ;; 1inverted U-shaped bowel loop;; 2liver overlap sign;; 3bird of prey sign;; 4cupola sign;

Guest

Back to top

Posted: Sat Jun 12, 2010 1:08 pm

Post subject:

The cupola sign (1) is seen at supine radiography as an arcuate lucency overlying the lower thoracic spine and projecting caudad heart . The superior border is well defined and the inferior margin is poorly delineated. The term cupola is used to indicate the inv cupshaped configuration of the lucency. X-RAY

A normal plain abdominal x-ray will demonstrate a huge air filled distended bowel like the shape of an inverted U, with the convex tube shape, ace of spades or Omega loop Sign. You can see an example down in the resource section. BARIUM ENEMA

the U facing the right upper abdominal quadrant. This shape has been described as the kidney bean shape, coffee bean shape, be

With a water soluble barium enema, the dilatation in the sigmoid colon can be demonstrated to be due to a twist, as it will show a of complete obstruction with some twisting in the so called bird beak or bird of prey sign. Colonoscopy could be done in rare cases, which would help to confirm diagnosis, as well as treating the obstruction the following are radiological features of sigmoid vovulus, except ;; 1 inverted U-shaped bowel loop;; 2 liver overlap sign;; 3 bird of prey sign;; 4 cupola sign;--answer

vinay.
Guest

Back to top
Posted: Mon Jun 14, 2010 1:13 pm Post subject:

Q20. radiologically -sub0acute haematogenous OM may resemble ;; 1 enchondroma;; 2 chondroma;; 3 ABC;; 4 OSTEOID OSTEOMA In 1982, Roberts et al modified and expanded Gledhill's classification to 6 forms based on morphology, location, and similarity to neoplasms, as follows Modified classification of subacute osteomyelitis... Type I is metaphyseal. Type Ia is a punched-out central metaphyseal lesion. Type Ib is an eccentric metaphyseal cortical erosion. Type II is diaphyseal. Type IIa is a localized cortical and periosteal reaction. Type IIb is a medullary abscess in the diaphysis without cortical destruction but with onionskin periosteal reaction. Type III is epiphyseal. Type IIIa is a primary epiphyseal osteomyelitis. Type IIIb is a lesion that crosses the epiphysis and involves both the epiphysis and the metaphysis. Type IV is a metaphyseal equivalent. Type IVa involves the vertebral body with an erosive or destructive process. Type IVb involves the flat bones of the pelvis. Type IVc involves the small bones, such as the tarsal bones.

* Type Ia lesions present as a punched-out radiolucency that is often suggestive of eosinophilic granuloma Type Ib lesions are similar to type Ia lesions but have a sclerotic margin and appear as a classic Brodie abscess. * Type II lesions erode the metaphyseal cortex and may appear similar to osteogenic sarcoma * Type III lesions are observed as a localized diaphyseal cortical and periosteal reaction simulating osteoid osteoma * Type IV diaphyseal lesions most often resemble Ewing sarcoma, with onionskin periosteal reaction * Type V lesions occur in the epiphysis and appear as a concentric radiolucency. * Type VI lesions involve the vertebral body with an erosive or destructive process. Answer is Osteoid osteoma Q21. blood flow through a vessel varies directly with ;; 1 resistance;; 2 viscosity;; 3 difference of pressure;; 4 length of vessel ;;

Mathematically, blood flow is described by Darcy's law (which can be viewed as the fluid equivalent of Ohm's law) and approxima

Hagen-Poiseuille equation The first equation below is Darcy's law, the second is the Hagen-Poiseuille equation: F =Difference of pressure/Resistance R =( vL/r^4)(8/pi) where: F = blood flow P = pressure R = resistance = fluid viscosity L = length of tube r = radius of tube

In the last equation it is important to note that resistance to flow changes dramatically with respect to the radius of the tube. This increase in radius of a vessel.

important in angioplasty, as it enables the increase of blood flow with balloon catheter to the deprived organ significantly with onl

Blood flow is inversely proportional to the Resistance and Length of the blood vessel and viscosity of the blood. It is directly propo to the Difference in the pressure. Q22. what percent of testicular volume is directly involved in producing sperms;; 1 20 percent'' 2 40 percent;; 3 50 percent;; 4 80 percent;;

In healthy European adult human males, average testicular volume is 18 cm per testis, with normal size ranging from 12 cm to

cm[5]. The average testicle size after puberty measures up to around 2 inches long, 0.8 inches in breadth, and 1.2 inches in hei 2 x 3 cm). Measurement in the living adult is done in two basic ways:

80 percent of testicle volume is from the sperm producing cells, and that they rely on quite high concs of testosterone to swell an fucntion Q23. which one of the following charecteristics is most similar in the systemic and pulmonary circulations;; 1 stroke work;; 2 preload;; 3 afterload;; 4 peak systolic pressure;; Q24. which muscle relaxant should not be used in myasthenia gravis;; 1 suxamethonium;; 2 pancuronium;; 3 atracurium;; 4 mivacurium;;

vinay.
Guest

Back to top
Posted: Tue Jun 15, 2010 1:25 pm Post subject:

Q24. suxamethonium is a depolarising muscle relaxant and it is contraindicated absolutely in conditions where there is a risk of lo hyperthermia; Glaucoma and ocular surgery; and genetic disorders of plasma psuedo cholinesterase.

In common with all neuromuscular blocking agents, atracurium paralyses the respiratory as well other skeletal muscles (e.g. mus administered only with adequate anaesthesia and only by an experienced anaesthetist familiar with its pharmacological properties The neuromuscular block of atracurium is increased during hypothermia and decreases when rewarming the patient.

Atracurium should be adminstered with care to patients with myasthenia gravis, other neuromuscular diseases, and severe electr Severe acidosis may result in a slight prolongation of action of atracurium.

As with other neuromuscular blocking agents, MIVACRON(Mivacurium) may have profound neuromuscular blocking effects in cach

in whom potentiation of neuromuscular block or difficulty with reversal may be anticipated, the initial dose should be decreased. A is recommended in such patients

Neuromuscular blocking agents may have a profound effect in patients with neuromuscular diseases (e.g., myasthenia gravis and

carcinomatosis), the use of a peripheral nerve stimulator and a dose of not more than 0.015 to 0.020 mg/kg MIVACRON is recom

In patients who are known to have myasthenia gravis or the myasthenic (Eaton-Lambert) syndrome, small doses of pancuronium value in monitoring the response to administration of muscle relaxants.

Answer is Suxamethonium as unlike the other options its actions are irreversible and CI in M.gravis while the other 3 can be used Q25. all of the following are associated with FITZ-CURTHS criteria, except ;; 1 bazexs syndrome;; 2 carcinoid syndrome;; 3 erythema gyratum repens;; 4 pityriasis rotunda Q26. IMP PRODUCTS OF GLYCINE ARE ALL, EXCEPT ;; 1 creatine;; 2 taurine;; 3 heme;; 4 purines;; creatine it is manufactured in the human body from L-arginine, glycine, and L-methionine

vinay.
Guest

Back to top
Posted: Tue Jun 15, 2010 1:25 pm Post subject:

Q27. chronic reflux nephropathy causes;; 1 MGN;; 2 FSGS;; 3 MPGN;; 4 MCD;;

FSGS can be classified as follows: * Primary (idiopathic) FSGS o FSGS with hyalinosis o Progression from minimal-change disease o Progression from immunoglobulin M (IgM) nephropathy o Progression from mesangial proliferative glomerulonephritis o Superimposed on other primary glomerulonephritis conditions (eg, membranous glomerulonephritis, immunoglobulin A [IgA] nephropathy) o Variants of primary FSGS + Collapsing form + Cellular variant (endocapillary and extracapillary hypercellularity) + FSGS with mesangial hypercellularity + FSGS with glomerular tip lesions * Secondary FSGS o Drugs + Intravenous heroin + Analgesics o Viruses + Hepatitis B + HIV + Parvovirus o Hemodynamic factors - With reduced renal mass + Solitary kidney + Renal allograft + Renal dysplasia + Renal agenesis + Oligomeganephronia + Segmental hypoplasia + VESICOURETERIC REFLUX o Hemodynamic causes - Without reduced renal mass + Massive obesity + Sickle cell nephropathy + Congenital cyanotic heart disease o Malignancies + Lymphomas + Other malignancies o Scarring - Postinflammatory in postinfectious glomerulonephritis o Miscellaneous + Hypertensive nephrosclerosis + Alport syndrome + Sarcoidosis + Radiation nephritis

Membranoproliferative GN is a form of glomerulonephritis caused by an abnormal immune response. Deposits of antibodies build

a part of the kidneys called the glomerular basement membrane. This membrane helps filter waste and extra fluid from the blood

The changes in this membrane lead disrupt the body's ability to filter urine. Protein and fluid leak out of the blood vessels into bod tissues, leading to swelling (edema). Nitrogen waste products may build up in the blood (azotemia) because of poor kidney functioning. MGN Primary/idiopathic

85% of MGN cases are classified as primary membranous glomerulonephritis -- that is to say, the cause of the disease is idiopath

(unknown). This can also be referred to as idiopathic membranous nephropathy. One study has identified antibodies to an M-type phospholipase A2 receptor in 70% (26 of 37) cases evaluated. Secondary The remainder is secondary due to: * autoimmune conditions (e.g., systemic lupus erythematosus) * infections (e.g., syphilis, malaria, hepatitis B) * drugs (e.g., captopril, NSAIDs, gold, mercury, penicillamine, probenecid). * inorganic salts common. Pathogenesis

* tumors, frequently solid tumors of the lung and colon; hematological malignancies such as chronic lymphocytic leukemia are les

MGN is caused by circulating immune complex. Current research indicates that the majority of the immune complexes are formed binding of antibodies to antigens in situ to the glomerular basement membrane. The said antigens may be endogenous to (from) basement membrane, or "planted" from systemic circulation.

The immune complex serves as an activator that triggers a response from the C5b - C9 complements, which form a membrane at complex (MAC) on the glomerular epithelial cells. This, in turn, stimulates release of proteases and oxidants by the mesangial and an unknown mediator that reduces nephrin synthesis and distribution. MCD 90% of children 10 and under with the Nephrotic Syndrome end up having MCD when biopsied (as opposed to 20% of adults).

epithelial cells, damaging the capillary walls and causing them to become "leaky". In addition, the epithelial cells also seem to sec

Minimal Change Disease is one of the most common causes of the Nephrotic Syndrome however, especially in children. In fact, al

Doctors tend to put MCD in two categories, primary and secondary. Primary means that the disease occurs on its own, for no obv another medical condition. MCD has been associated with all of the following, usually in adultsDrugs NSAIDs, lithium, some antibiotics, bisphophonates Malignancy Leukemia, Lymphoma Infection Syphillis, HIV, Hepatitis Allergy Associated with multiple environmental allergies secondary MCD is very uncommon. Answer is FSGS

reasons. This is by far the most common type. Secondary means that we think the MCD was caused by, or is at least associated w

vinay.
Guest

Back to top

Posted: Wed Jun 16, 2010 5:05 pm

Post subject:

Q28. the modern radiation detector used in ct is ;; 1 sodium iodide;; 2 xenon;; 3 ceramic based gadolinium compound;; 4 ZNCDS;; Q29. the following remains unchanged in pregnancy;; 1 factor 2;; 2 platelet count;; 3 factor 11;; 4 clotting time;;

sujit.
Guest

Back to top
Posted: Fri Jun 18, 2010 1:43 pm Post subject:

Q30. WHICH OF THE FOLLOWING IS NORMAL CAST IN URINE;; 1 granular;; 2 waxy;; 3 epithelial;; 4 hyaline; Epithelial cell casts

This cast is formed by inclusion or adhesion of desquamated epithelial cells of the tubule lining. Cells can adhere in random order

sheets and are distinguished by large, round nuclei and a lower amount of cytoplasm. These can be seen in acute tubular necrosis toxic ingestion, such as from mercury, diethylene glycol, or salicylate. In each case, clumps or sheets of cells may slough off death as well. Hyaline casts

simultaneously, depending of the focality of injury. Cytomegalovirus and viral hepatitis are organisms that can cause epithelial ce

The most common type of cast, hyaline casts are solidified Tamm-Horsfall mucoprotein secreted from the tubular epithelial cells o

individual nephrons. Low urine flow, concentrated urine, or an acidic environment can contribute to the formation of hyaline casts and, as such, they may be seen in normal individuals in dehydration or vigorous exercise. Hyaline casts are cylindrical and clear,

a low refractive index, so that they can easily be missed on cursory review under brightfield microscopy, or in an aged sample wh Tamm-Horsfall protein, other cast types are formed via the inclusion or adhesion of other elements to the hyaline base. Granular casts The second-most common type of cast, granular casts can result either from the breakdown of cellular casts or the inclusion of aggregates of plasma proteins (e.g., albumin) or immunoglobulin light chains. Depending on the size of inclusions, they can be

dissolution has occurred. On the other hand, phase contrast microscopy leads to easier identification. Given the ubiquitous presen

classified as fine or coarse, though the distinction has no diagnostic significance. Their appearance is generally more cigar-shaped can also be seen for a short time following strenuous exercise. Waxy casts

of a higher refractive index than hyaline casts. While most often indicative of chronic renal disease, these casts, as with hyaline ca

Thought to represent the end product of cast evolution, waxy casts suggest the very low urine flow associated with severe,

longstanding kidney disease such as renal failure. Additionally, due to urine stasis and their formation in diseased, dilated ducts, t

casts are significantly larger than hyaline casts. While cylindrical, they also possess a higher refractive index and are more rigid, demonstrating sharp edges, fractures, and broken-off ends. Waxy casts also fall under the umbrella of broad casts, a more gen term to describe the wider cast product of a dilated duct. Answer is Hyaline casts. Q31. doppler technique is based on ;; 1 VELOCITY SHIFT;; 2 AMPLITUDE SHIFT;; 3 FREQUENCY SHIFT;; 4 WAVELENTHSHIFT;;

The Doppler effect (or Doppler shift), named after Austrian physicist Christian Doppler who proposed it in 1842, is the change in frequency of a wave for an observer moving relative to the source of the wave. It is commonly heard when a vehicle sounding a s during the approach, it is identical at the instant of passing by, and it is lower during the recession. Answer is Frequency shift.

or horn approaches, passes, and recedes from an observer. The received frequency is higher (compared to the emitted frequency

vinay.
Guest

Back to top
Posted: Sat Jun 19, 2010 9:54 am Post subject:

Q32. ALL OF THE FOLLOWING are assciated with low complement levels , except ;; 1. lupus nephritis;; 2 mesangiocapillary;; 3 diarrhoea associated with hemolytic uraemic syndrome;; 4 post - infectiouous glomerulonephritis

Deficiency of early components of classical pathway (C1, C4, C2): Autoimmune disease, especially systemic lupus erythematosus (SLE), is the most common presentation in patients with early component deficiency. Complement Factor H deficiency is also associated with atypical (diarrhea-negative) hemolytic-uremic syndrome (HUS) and glomerulonephritis. C3 deficiency is associated with mesangiocapillary glomerulonephritis.

Nephritis with low complement 1. Postinfectious proliferative glomerulonephritis 1. Poststreptococcal Glomerulonephritis (classic) 2. Subacute Bacterial Endocarditis 3. Osteomyelitis 2. Membranoproliferative glomerulonephritis 3. Systemic Lupus Erythematosus 4. Cryoglobulinemia 5. Diabetes Mellitus 6. Hepatitis C Virus

* Factor H deficiency H deficiency. o Factor H deficiency also occurs with membranoproliferative glomerulonephritis. o Parents have one half the normal levels of factor H. o HUS associated with factor H deficiency is characterized by verotoxin-negative (diarrhea-negative) HUS. Atypical and recurrent HUS is also seen in CD46 deficiency. C3 nephritic factor (C3NeF) and mesangiocapillary glomerulonephritis (MCGN)

o Factor H deficiency is associated with hemolytic-uremic syndrome (HUS), especially in familial cases that involve homozygous fa

* C3Nef is an autoantibody that binds to and stabilizes the C3 convertase (C3bBb). * The association of C3NeF and MCGN, especially MCGN type II, has been well defined. Different isotypes of C3NeF are recognize

main one being an IgG autoantibody against factor H. Factor H is an important regulator of the C3 conversion step in the alternat Continuous C3 activation in vivo results in the well-known consequences of very low serum levels of C3 in MCGN. * C3NeF may act directly within glomeruli to cause local complement activation and ensuing renal damage. * C3Nef is also associated with partial lipodystrophy. Answer is None Q33. drug of choice to treat -theophylline toxicity;; 1 diazepam;; 2 neostigmine;; 3 propranlol;; 4 atropine Benzodiazepines and other sedative agents These agents are used to terminate seizures and for seizure prophylaxis in high-risk patients. They help to alleviate nausea and vomiting and decrease tremors and anxiety induced by theophylline. Diazepam (Valium) Depresses all levels of CNS (eg, limbic and reticular formation), possibly by increasing activity of GABA. Answer is Diazepam

pathway. C3NeF inhibits functions of factor H, which leads to overwhelming complement activation at the stage of C3 conversion.

vinay.
Guest

Back to top
Posted: Sun Jun 20, 2010 2:49 pm Post subject:

Treatment Prehospital Care * Establish airway, breathing, and circulation (ABCs). * Intravenous benzodiazepines may abort seizures. Emergency Department Care Evaluate ABCs and, if indicated, perform endotracheal intubation.

* Vascular access for hemoperfusion may be required. * Endotracheal intubation may be needed in patients who require high-dose benzodiazepines or barbiturates to control seizures.

* Consider gastric lavage (unless contraindicated) if the patient has recently (<1 h) ingested a significant amount or a sustained-

release preparation of theophylline or if theophylline bezoar formation is suspected. Gastric lavage should be considered in intuba patients. Endoscopic bezoar fragmentation and retrieval may be utilized if lavage is not efficacious. * Administer activated charcoal. o Multidose activated charcoal (MDAC) enhances elimination of theophylline. + It is a very effective method of elimination, and it is considered the mainstay treatment of theophylline toxicity. + It is important to aggressively control nausea and vomiting in order to perform MDAC treatment. be detrimental. o Administer the cathartic, sorbitol, with the activated charcoal one time. * Perform whole-bowel irrigation (WBI) in patients with exposure to sustained-release theophylline preparations. o Administer polyethylene glycol electrolyte solution. + Adults: 2 liters per hour until clear rectal effluent

+ It is also important that the patient is able to protect his or her airway in order to prevent aspiration of activated charcoal, whic

+ Children: 500 mL/h until clear rectal effluent * Theophylline-induced seizures tend to be resistant to treatment. Benzodiazepines (eg, lorazepam) are considered the first line o o Acute overdose with theophylline levels higher than 80 mcg/mL o Chronic toxicity with levels higher than 40 mcg/mL o Patients older than 60 years or younger than 3 years * Benzodiazepines (IV) and phenobarbital may be used to treat seizures. o CAVEAT: Barbiturates can precipitate hypotension.

treatment. Historically, phenobarbital prophylaxis was used in patients at high risk for seizures. High-risk cases include the follow

o Phenobarbital has the added advantage of enhancing the hepatic metabolism of theophylline. * Hypotension resistant to isotonic fluids (10-20 mL/kg) may require vasopressors with predominantly alpha-agonistic activity (eg phenylephrine, norepinephrine). o In patients with theophylline toxicity, beta-blockade with propranolol has been shown to successfully reverse peripheral beta receptor-mediated hypotension without apparent effect on concomitant tachycardia. o However, always consider the risk of beta-adrenergic blockade to patients with preexistent bronchospastic disease. * Esmolol, a short-acting beta-blocker, has been used successfully for unstable SVT and related hypotension in theophylline overdose.2 o Exercise caution with beta-blocking agents because of their negative inotropic effects. o Esmolol is a relatively selective beta1-receptor antagonist; thus, it may not have as much effect on beta2-mediated hypotensio less-selective agents (eg, propranolol), although it is less likely to induce bronchospasm than other beta-blockers. * Consider hemoperfusion with the following: o Symptomatic patients with levels exceeding 90 mcg/mL in acute ingestions o Theophylline concentrations exceeding 40 mcg/mL (chronic ingestion) o Presence of life-threatening toxicity + Persistent seizures + Hypotension that is not responding to IV fluids + Ventricular dysrhythmias o Hemodialysis is an alternative method of elimination enhancement but is considerably less effective than hemoperfusion. * Correct electrolyte abnormalities in patients with ECG changes (eg, QTc prolongation) and/or ventricular dysrhythmias. o Hypocalcemia o Hypophosphatemia o Hypokalemia include the following: o Fluid bolus with isotonic fluid (20 mL/kg) o Metoclopramide or ondansetron to help control vomiting o Propranolol to increase blood pressure - Reportedly propranolol treatment can correct hypokalemia.3 o Benzodiazepine to decrease anxiety, decrease risk of seizures, and help control vomiting o Phenylephrine or norepinephrine to further increase blood pressure o Charcoal hemoperfusion guided by response to treatment, underlying medical problems, and theophylline level + Because charcoal hemoperfusion is a somewhat complicated process that is not routinely used lately, most of the centers will perform routine hemodialysis.

* Current recommendations for treating patients with tachycardia, hypotension, anxiety, and vomiting from theophylline overdose

+ Hemodialysis in combination with MDAC will most of the time be sufficient for the treatment of severe theophylline toxicity.

seema.
Guest

Back to top
Posted: Sun Jun 20, 2010 2:49 pm Post subject:

Factor H is a member of the regulators of complement activation family and is a complement control protein. It is a large (155

kilodaltons), soluble glycoprotein that circulates in human plasma (at a concentration of 500800 micrograms per milliliter). Its m job is to regulate the Alternative Pathway of the complement system, ensuring that the complement system is directed towards pathogens and does not damage host tissue

seema.
Guest

Back to top
Posted: Sun Jun 20, 2010 2:51 pm Post subject:

Q34. mixing waves of stomach ;; 1 originate in body of stomach;; 2 originate in fundus of stomach;; 3 originate at incisura angularis;; 4 originates in any part of stomach;; Answer is Fundus. The food mass enters the stomach, and several minutes later mixing waves pass over the stomach. These are gentle, rippling

peristaltic movements which pass over the stomach every 15 to 25 seconds when there is food in the stomach. The movement of waves mixes the food with the secretions of the gastric glands, softening the food mass, and reducing it to a thin liquid called chy stored is the fundus for an hour or more without becoming mixed with gastric juice. During this storage time, salivary digestion

(pronounced kim). The fundus portion of the stomach is mainly a storage area, and few mixing waves take place there. Foods ma

continues. The food progresses through the stomach from the fundus to the body where the mixing waves become stronger and e stronger as the food reaches the pylorus. At the pylorus, each mixing wave forces a small amount of the stomach contents into th duodenum. Q35. all of the following are condensations of the deep cervical fasca of the neck, except ;; 1 the pretracheal layer;; 2 the prevertebral layer;; 3 the spheno mandibular ligaments ;; 4 the stylomandibular ligament;; The Pretracheal Fascia (p. 823) * This is a thin layer of deep cervical fascia that is anterior to the trachea and limited to the anterior aspect of the neck. * It extends inferiorly from the thyroid cartilage and the arch of the cricoid cartilage into the thorax. * The pretracheal fascia lies deep to the infrahyoid muscles.

* It splits to enclose the thyroid gland, trachea, and oesophagus and blends laterally with the carotid sheath. Thus, it encloses the

viscera of the neck. The Prevertebral Fascia (p. 825)

* This layer of deep cervical fascia forms part of a tubular sheath for the prevertebral muscles that surrounds the vertebral colum * It is also continuous with the deep fascia covering the muscular floor of the posterior triangle of the neck.

* The prevertebral fascia extends from the base of the skull to the third thoracic vertebra, where it fuses with the anterior longitu ligament.

* The prevertebral fascia extends inferiorly and laterally as the auxiliary sheath, which surrounds the axillary vessels and brachial plexus. The superficial (investing) layer of deep cervical fascia is attached posteriorly to the external occipital protuberance, the nuchal

ligament and the spine of the seventh cervical vertebra. It forms an investment for the trapezius muscle, and from the anterior b

of this muscle it continues forwards, covering the posterior triangle of the neck (it forms part of the roof of this triangle), to the posterior border of the sternocleidomastoid muscle. Here, the layer divides to enclose the muscle, and at the anterior margin aga

forms a single lamina which covers the anterior triangle of the neck and reaches forwards to the median plane where it is continu

with the corresponding layer from the opposite side. In the median plane, it is adherent to the symphysis menti, the body of the h

bone and to the anterior and posterior surfaces of the manubrium sterni. Above, the fascial layer is attached to the superior nuchal line, the mastoid process and the whole of the base of the mandible. Be

the angle of the mandible and the mastoid process, the layer divides to ensheathe the parotid gland. The superficial layer, the pa

fascia, covers the gland and extends upwards to become fixed to the zygomatic arch. From here, it continues upwards as the tem

fascia to become attached to the superior temporal line. This layer covers the temporalis muscle which also originates in part from fascia. The deeper layer of the superficial layer, known as the stylomandibular ligament, passes deep to the parotid gland and asc to become attached to the styloid process and the tympanic bone.

The sphenomandibular ligament (internal lateral ligament) is a flat, thin band which is attached above to the spina angularis of th

sphenoid bone, and, becoming broader as it descends, is fixed to the lingula of the mandibular foramen. The ligament is derived f Meckel's cartilage.

vinay.
Guest

Back to top
Posted: Sun Jun 20, 2010 2:54 pm Post subject:

Q36. RATCHET PRINCIPLE is applicable in relation to ;; 1 performance apprisal;; 2 system analysis;; 3 network analysis;; 4 none of the above Q37. most common adverse effect on eye in using oral contraceptive is ;; 1 color blindness;; 2 ring scotoma;; 3 optic neuritis;; 4 nystagmus Q38. THE BENDERS AND THE THE STAGERS ARE SEEN IN ;; 1 sickle cell disease;; 2 caissons disease;; 3 radiation necrosis;;

4 osteoarthritis Q39. IN GLYCOLYSIS, which of the ions is most imp ;; 1 ca;; 2 mg;; 3 cu;; 4 zn;; Q40. the most charecteristic finding in lichen planus is ;; 1 civatte bodies;; 2 basal cell degeneration;; 3 thinning f nail plate;; 4 violaceous lesions;;

vinay.
Guest

Back to top
Posted: Mon Jun 21, 2010 1:19 pm Post subject:

Q41. NANO SCALE CANTILEVER ARRAY ARE BEING USED FOR ;; 1 TREATMENT of retinal detachment ;; 2 to detect biomarker of cancer cell;; 3 to detect amyloidosis in the early stage ;; 4 to detect early plaques of atherosclerosis

Nanotechnology extends the limits of molecular diagnostics to the nanoscale. Nanotechnology-on-a-chip is one more dimension o microfluidic/lab-on-a-chip technology. Biological tests measuring the presence or activity of selected substances become quicker,

sensitive and more flexible when certain nanoscale particles are put to work as tags or labels. Magnetic nanoparticles, bound to a

suitable antibody, are used to label specific molecules, structures or microorganisms. Magnetic immunoassay techniques have bee developed in which the magnetic field generated by the magnetically labeled targets is detected directly with a sensitive magneto coding for biological assays has been achieved by embedding different-sized quantum dots into polymeric microbeads. Nanopore

Gold nanoparticles tagged with short segments of DNA can be used for detection of genetic sequence in a sample. Multicolor optic

technology for analysis of nucleic acids converts strings of nucleotides directly into electronic signatures. DNA nanomachines can function as biomolecular detectors for homogeneous assays. Nanobarcodes, submicrometer metallic barcodes with striping patter

prepared by sequential electrochemical deposition of metal, show differential reflectivity of adjacent stripes enabling identification devices. Q42. MEDICATIONS KNOWN TO ADVERSELY AFFECT GLYCAEMIC CONTROL IS ;; 1. anticonvulsants;; 2 antihistamines;; 3 antidepressants;; 4 calcium channel blockers;;

striping patterns by conventional light microscopy. All this has applications in population diagnostics and in point-of-care handhe

vinay.
Guest

Back to top
Posted: Tue Jun 22, 2010 1:00 pm Post subject:

Q43. SONTAGS METHOD S A METHOD TO STUDY;; 1 bone age;; 2 sexual development;; 3 hair growth;; 4 stage of adolescence;; serial evaluation of children on growth hormone and other therapy

Q44. which one of the following protein segment is charecterised by its ability to bind to - phosphotyrosine -containing peptides;; 1 a helx-loop helix domain;; 2 a leucine zipper;; 3 an src homology 2 domain;; 4 a pleckstin motiff;; Q45. antoni a and antoni b patterns are seen in ;; 1 schwannoma;; 2 neurofibroma;; 3 meningioma;; 4 teratoma;;

vinay.
Guest

Back to top
Posted: Wed Jun 23, 2010 1:31 pm Post subject:

OEDIPIAL COMPLEX IS PART OF WHICH STAGE OF FREUDS THEORY OF DEVELOPMENT;; 1ORAL STAGE;;--answer 2ANAL STAGE;; 3LATENCY STAGE;; 4 NONE OF THE ABOVE Freud's Theory of Development the Oedipus Complex

Oedipus complex is one of the stages in Sigmund Freud's theory of sexual development. Freud's theory actually describes four sta development: oral, anal, phallic, and the Oedipus complex. All of these stages are necessary for proper development of the child. Oedipus complex is one of the most interesting though because of its description of the family structure.

The first three stages of sexual development make up the foundation for the last phase, the Oedipus complex. The child is born w what is called polymorphous perversity, where every event that it experiences is sexual. This polymorphous perversity begins wit

first stage: oral. During this phase the child senses a sexual experience while feeding from the mother's breast. At this point the c develops an object-cathexis which is later intensified during the Oedipus complex stage. Then as the child gets a little older it ent

the next stage, which is anal development. The child learns to control their bowels and therefore reaches a certain level of independence. After this comes the phallic stage where the child gets a bit more older and realizes the difference between the sex

simply put he figures out that his mother the doesn't have a penis. After all of these phases have been passed the child begins th and most important stage called the Oedipus complex. This stage is so crucial that Freud credits any future disorder to a malfunct

the transitional phase between the Oedipus complex stage and the earlier stages of sexual development. In the Oedipus complex phase the male child "develops an object cathexis for his mother, which originally related to the mother breast ..." (26). His mother becomes the love-object meanwhile "the boy deals with his father by identifying himself with him. Fo time these two relationships proceed side by side until the boy's sexual wishes in regard to his mother become more

vinay.
Guest

Back to top
Posted: Wed Jun 23, 2010 1:34 pm Post subject:

Q46. actin is a microfilament that is involved wiyj all of the following activities, except ;; 1 endocytosis;; 2 exocytosis;; 3 cell locomotion;; 4 mitotic spindle formation Q48. OEDIPIAL COMPLEX IS PART OF WHICH STAGE OF FREUDS THEORY OF DEVELOPMENT;; 1 ORAL STAGE;; 2 ANAL STAGE;; 3 LATENCY STAGE;; 4 NONE OF THE ABOVE Q49. the ligament of landsmeer is found in ;; 1 phalange;; 2 hip; 3 wrist;; 4 elbow

ANSWER PHALANGE The capsule, extensor tendon, and skin are very thin and lax dorsally, allowing for both phalanx bones to flex more than 100 unt base of the middle phalanx makes contact with the condylar notch of the proximal phalanx.

At the level of the PIP joint the extensor mechanism splits into three bands. The central slip attaches to the dorsal tubercle of the

middle phalanx near the PIP joint. The pair of lateral bands, to which contribute the extensor tendons, continue past the PIP joint dorsally to the joint axis. These three bands are united by a transverse retinacular ligament, which runs from the palmar border o

lateral band to the flexor sheath at the level of the joint and which prevents dorsal displacement of that lateral band. On the palm proximal phalanx to the terminal extensor tendon. In extension, the oblique ligament prevents passive DIP flexion and PIP hyperextension as it tightens and pulls the terminal extensor tendon proximally Q50. culture medium used for chancroid is all, except ;; 1 blood agar;; 2 enriched blood agar;; 3 chocolate agar;; 4 mc coy cell line

side of the joint axis of motion, lies the oblique retinacular ligament [of Landsmeer] which stretches from the flexor sheath over t

Significant progress in the microbiologie diagnosis of chancroid occurred when Hammond and coworkers11 (1978) described a me and vancomycin hydrochloride (2.04 |xmojVL=^ mg/L). Using this selective solid medium, they successfully recovered H ducreyi

consisting of gonococcal agar base, 1% bovine hemoglobin, 1% Iso- VitaleX enrichment (BBL Microbiology Systems, Cockeysville,

eight of 16 genital ulcrations. Sottnek et al12 demonstrated good growth (90% + sensitivity) on a medium containing heart infu agar (BBL), 10% fetal bovine serum, and vancomycin hydrochloride (2.04 p-mol/L)- This group believed that the sensitivity of be superior to trypticase soy agar for isolation of H ducreyi. Muelle''' Hinton solid medium was also more sensitive than either

Hammond's medium was increased by the addition of 10% fetal bovine serum-/ Oberhofer and Back13 found Miieller-Hinton agar

heatinactivated or noninactivated clotted human blood in their study; only 12 of 15 specimens positive on Meller-Hintot' agar we positive on clot media. Sng and coworkers14 isolated 13 strains ofH ducreyi from genital ulcers using a specify agar (Bacto Protea No. 3 agar, Difco Laboratories) with added soluble 0.1% starch, 1% IsoVitaleX (BBL), 15% human blood, and vancomycin in the

concentration as previously reported.

In Kenya in 1983, Nsanze et al" studied 201 patients with genital ulcrations. They recovered Hducreyi from 143 specimens (71%

gonococcal base agar with 2% bovine hemogl0' bin and 5% fetal alfserum (GC-HgS) with vancomycin hydro- chloride (2.04 xmo

and 122 specimens (61%) on Mellei-- Hinton agar with both 5% chocolatized horse blood (MH-HB' and vancomycin added. When used together, the recover)' rate was 81%; consequently, these investigators recommended the routine use of both media,

incorporated into a singly biplate to acilitate their use." These results were supported by another Kenyan study reported by Dylew

and coworkers" who found an isolation rate of 61% on GC-HgS, 48% of MH-HB, and 68% using both media in a study of 111 men

wit'1 clinical chancroid. Both media contained vancomycin hydro- chloride (2.04 |i.mol/L). A repeated culture at 48 hours yielded additional 8% positive cultures. A review of culture-negative patients revealed 11 with primary syphilis, thus resulting in an overa impractical in many venereal disease clinics where follow-up tends to be poor.

recovery rate of 75% using one culture on both media, 82% if a second culture was obtained." A second culture, however, may p

McCoy cells are applied for culture of various microorganisms, which are dependent in their development on the eukaryotic host c During their interaction with pathogens the cells are subjected to various changes leading often to lethality of infected cells.As rec reports show the cells are actually important for culturing viruses, chlamydia, vaccine studies, development of models for C. trachomatis or cytotoxic activity. A new direction of McCoy application is the creation of McCoy- Plovdiv serum-free cell line. Answer is MC Coy cell line.

vinay.
Guest

Back to top
Posted: Thu Jun 24, 2010 1:18 pm Post subject:

vinay.
Guest

Q51. THE cerebellum of a human brain reaches its adult size at the age of '' 1 6 months;; 2 12 months;; 3 15 months;; 4 24 months Q52. FAIRBANKS CHANGES IN THE KNEE JOINT SIGNIFIES ;; 1 patellofemoral pathology;; 2 chronic mcl injury;; 3 early deg joint disease;; 4 discoid meniscus; Q53. mitral valve calcification is best visualised with --projecton of chest radiograph;; 1 posteroanterior;; 2 anteroposterior;; 3 lateral;; 4 lordotic;;

Back to top

Posted: Thu Jun 24, 2010 1:20 pm

Post subject:

Q1 ALCOHOLIC PARANOIA..WHICH OF THESE IS SEEN?? 1. IMPULSIVE BEHAVIOUR 2. HALLUCINATION 3. FIXED DELUSION 4. AGITATION alcoholic paranoia A type of alcohol-induced psychotic disorder in which delusions of a self-referential or persecutory nature are prominent. Alcoholic jealousy is sometimes included as a form of alcoholic paranoia.

alcoholic jealousy A type of chronic, alcohol-induced psychotic disorder, characterised by delusions that the marital or sexual part unfaithful. The delusion is typically accompanied by intense searching for evidence of infidelity and direct accusations that may le

violent quarrels. It was formerly regarded as a distinct diagnostic entity, but this status is now controversial. Synonyms: amorous paranoia; conjugal paranoia. Answer is fixed delusion.

Q2 .WHICH OF FOLLOWING IS NOT PROVEN PART OF COGNITIVE BEHAV THERAPY?? 1. PRE CONTEMPLATION 2. CONTEMPLATION 3. ACTION 4. CONSOLIDATION

Change implies phenomena occurring over time. However, this aspect was largely ignored by alternative theories of change. Beha

change was often construed as an event, such as quitting smoking, drinking, or over-eating. The Transtheoretical Model construe change as a process involving progress through a series of five stages. Precontemplation is the stage in which people are not intending to take action in the foreseeable future, usually measured as the

six months. People may be in this stage because they are uninformed or under-informed about the consequences of their behavio they may have tried to change a number of times and become demoralized about their ability to change. Both groups tend to avo

reading, talking or thinking about their high risk behaviors. They are often characterized in other theories as resistant or unmotiv or as not ready for health promotion programs. The fact is traditional health promotion programs are often not designed for such individuals and are not matched to their needs. Contemplation is the stage in which people are intending to change in the next six months. They are more aware of the pros of changing but are also acutely aware of the cons. This balance between the costs and benefits of changing can produce profound ambivalence that can keep people stuck in this stage for long periods of time. We often characterize this phenomenon as chronic contemplation or behavioral procrastination. These people are also not ready for traditional action oriented programs.

Preparation is the stage in which people are intending to take action in the immediate future, usually measured as the next month

They have typically taken some significant action in the past year. These individuals have a plan of action, such as joining a healt education class, consulting a counselor, talking to their physician, buying a self-help book or relying on a self-change approach. T are the people that should be recruited for action- oriented smoking cessation, weight loss, or exercise programs.

Action is the stage in which people have made specific overt modifications in their life-styles within the past six months. Since act observable, behavior change often has been equated with action. But in the Transtheoretical Model, Action is only one of five stag

Not all modifications of behavior count as action in this model. People must attain a criterion that scientists and professionals agre sufficient to reduce risks for disease. In smoking, for example, the field used to count reduction in the number of cigarettes as ac

or switching to low tar and nicotine cigarettes. Now the consensus is clear--only total abstinence counts. In the diet area, there is consensus that less than 30% of calories should be consumed from fat. The Action stage is also the stage where vigilance against relapse is critical.

Maintenance is the stage in which people are working to prevent relapse but they do not apply change processes as frequently as people in action. They are less tempted to relapse and increasingly more confident that they can continue their change. Answer is consolidation.

Q3 WHICH ONE IS NOT A COGNITIVE DYSFUNCTION 1. THOUGHT BLOCK 2. CATASTROPHIZATIION 3. SELECTIVE ABSRACTION 4. OVERGENERALISATION

Cognitive distortions can cloud perceptions, judgments and assumptions of a depressed individual. They include distorted ideas an false assumptions, which need to be checked in order to restore mental health. Overgeneralization is one of the common cognitive distortions that a person can experience during anxiety or depression. Over-generalization: You take a negative event as a pattern of your life. We naturally relate new experiences to our old ones. So commonly generalize based on our past experience. Over-generalizations also make up many of our stereotypes of other people. like taking something that happened before and thinking that it will always happen again i.e. I always screw up. A thought block is a disorder of thinking whereby the clients capacity to maintain a train of thought is constantly interrupted. selective abstraction is a type of cognitive distortion in which focus on one aspect of an event negates all other Cognitive distortions

These are maladaptative thinking patterns that distort reality in a negative way, and make us perceive the world as being more h cognitive distortions. Arbitrary inference refers to the drawing of an unjustified conclusion. For example, a businessman never takes his wife on any of other possible explanations.

than it actually is. Arbitrary inference, selective abstraction, over-generalisation, magnification, and minimisation are examples of

official trips. His wife is upset. She concludes that he is concealing something from her, perhaps an extramarital affair. She neglec

Selective abstraction is the focussing of attention on one detail without regard to the rest of the picture. For example, a young ma consider that he has several assets such as intelligence and a pleasing personality. To him, these are of little importance. Over-generalisation is the drawing of a general conclusion based upon a limited event. For example, a father discovers that his adolescent son has been smoking. He is distraught. He concludes that the boy has picked up this habit from bad company. He

depressed because he does not have a motorcycle. He feels that no girl will take him seriously unless he has a "bike". He does no

concludes that the boy is probably taking drugs as well. He concludes that his son is untrustworthy, and requires close supervisio

neglects to take into account the possibility that his son, like many other youngsters his age, has probably merely been experime with the experience.

Magnification is making mountains of molehills. Failing in an important examination is an unhappy event, but it is not the end of t world. A sensible student would grieve briefly, then pick up the pieces of his life and begin studying again.

Minimisation is an undervaluation of positive attributes. A woman may have low self-esteem because she is not well-off. She negl

the respect that she commands for being an efficient employee, a good mother, a caring wife, a cheerful neighbour, and a loyal fr Repeated intrusive (automatic) thoughts

When we are unhappy, our sadness is often sustained by repeated, intrusive thoughts. These push themselves into consciousness preoccupy or even dominate the mind, leaving little opportunity for the experience of happier thoughts Answer is catastrophization.

suman.
Guest

Back to top
Posted: Fri Jun 25, 2010 1:14 pm Post subject:

Q54. MOST COMMON MANIFESTATION OF LYME CARDITIS;; 1 left ventricular dysfunction;; 2 concomitant myopercarditis;; 3 conduction abnormalities;; 4 rt ventricular dysfunction

Cardiovascular manifestations of Lyme disease often occur within 21 days of exposure and include fluctuating degrees of atrioven (AV) block, acute myopericarditis or mild left ventricular dysfunction and rarely cardiomegaly or fatal pericarditis. AV block can va first-, second-, third-degree heart block, to junctional rhythm and asystolic pauses.

About 10 percent of patients with Lyme disease develop evidence of transient cardiac involvement, the most common manifestati being variable degrees of atrioventricular block at the level of the atrioventricular node. Syncope due to complete heart block is fr

with cardiac involvement because often there is an associated depression of ventricular escape rhythms. Ventricular tachycardia o

uncommonly. Diffuse ST segment and T wave abnormalities and transient, usually asymptomic, left ventricular dysfunction may b

in some patients, although cardiomegaly or symptoms of congestive heart failure are rare. A positive gallium or indium antimyosi antibody scan may point to suspected cardiac involvement in this disease. The demonstration of spirochetes in myocardial biopsie that immune-mediated mechanisms may be involved as well. Answer is conduction abnormalities. Q55. which vaccine loses potency , when frozen;; 1 BCG;; 2 OPV;; 3 MMR;; 4 DPT;;

some patients with Lyme carditis suggests that the cardiac manifestations are due to a direct toxic effect, although there is specu

Diphtheria, tetanus and/or acellular pertussis- containing vaccines Includes DTPa, DTPa- hepB, DTPa-Hib, DTPa-IPV, DTPa- hepBDTPa-IPV- Hib, DTPa-IPV/Hib, DTPa-hepB-IPV-Hib, dTpa, DT (CDT), dT(ADT). DO NOT FREEZE.As Vaccines loses significant potency when stored at 5C to 10C. Safe to store at 2C to 8C for 24 months in spite of continuous slow decrease in potency of the pertussis component. BCG (freeze-dried or lyophilised vaccine) Can be stored at up to 20C. Do not expose to light (ultraviolet and/or fluorscent). Safe storage for 12 months at 2 to 8 degrees. Do not expose to light (ultraviolet and/or fluorescent). Diluent do not freeze (5) Store between 2C and 8C.

Measles-mumps-rubella (MMR) (freeze-dried or lyophilised vaccine) May be stored in freezer at 0C or below. Protect from light, which may inactivate virus. Safe storage for 2 years at 2C to 8C. Diluent do not freeze. Store between 2C & 8C. Oral poliomyelitis vaccine (OPV) opened vials May be stored for up to 2 years at around 20C.

The freeze-thaw-refreeze cycle can occur until the vial is empty. Answer is DPT. Q56. psychoanalysis as a form of psychotherapy includes all of the following except ;; 1 free association ;; 2 resistance;; 3 countertransference;; 4 meditation Q57. the retinal pigment epithelium is charecterised by which of the following ;; 1 the presence of the photoreceptor perikarya;; 2 phagocytosis of worn0out components of photoreceptor cells;; 3 origin from the inner layer of the optic cup during embryonic develpment;; 4 presence of amacrine cells;;
vinay.
Guest

Back to top vinay.


Guest Posted: Mon Jun 28, 2010 11:46 am Post subject:

Q64. teratogenic potential is a side effect of ;; 1 stavudine;; 2 zidovudine;; 3 efavirinez;; 4 lamivudine Q65. which one of the following is not part of processing of mrna transcripts in eukaryotic cells;; 1 splicing;; 2 methylation;; 3 addition of a 5u cap;; 4 addition of a polyadenylate tail to the 3 u end Q66. ABDUCTOR LURCH is a common finding after , which of the following osteotomies;; 1 salters;; 2 chiaris;; 3 triple innominate;; 4 periacetabular osteotomy Q67. MESNA IS USED AS ANTIDOTE FOR ALL, EXCEPT ;; 1 ifosphamide;; 2 fludarabine;; 3 busulphan;; 4 acrolein Q68. BOWL OF SPAGHETTI -phenomenon i s seen in ;; 1 adeno virus;; 2 guinea-worm disease;; 3 toxocara;; 4 ebola virus

Back to top Guest


Posted: Mon Jun 28, 2010 11:47 am Post subject:

Adverse effects-Efavirenz

* Psychiatric symptoms, including insomnia, confusion, memory loss, and depression, are common, and more serious symptoms such as psychosis may occur in patients with compromised liver or kidney function. * Rash, nausea, dizziness and headache may occur * Efavirenz can cause birth defects and should not be used in women who might become pregnant * Safety in children has not been established * Use of efavirenz can produce a false positive result in some urine tests for marijuana

Back to top Guest


Posted: Mon Jun 28, 2010 11:49 am Post subject:

Polyadenylation

Polyadenylation is the addition of a poly(A) tail to an RNA molecule. The poly(A) tail consists of multiple adenosine monophosphates; in other words, it is a stretch of RNA which only has Adenine bases. In eukaryotes, polyadenylation

of the process that produces mature messenger RNA (mRNA) for translation. It therefore forms part of the larger pro gene expression.

The process of polyadenylation begins as the transcription of a gene finishes. The 3'-most segment of the newly-mad

first cleaved off by a set of proteins; these proteins then synthesise the poly(A) tail at the RNA's 3' end. In some gen proteins may add a poly(A) tail at any one of several possible sites, polyadenylation can therefore produce more than transcript from a single gene, similar to alternative splicing.

The poly(A) tail is important for the nuclear export, translation and stability of mRNA. The tail is shortened over time when it is short enough, the mRNA is enzymatically degraded. However, in a few cell types, mRNAs with short poly(A

are stored for later activation by re-polyadenylation in the cytosol. In contrast, when polyadenylation occurs in bacter promotes RNA degradation. This is also sometimes the case for eukaryotic non-coding RNAs. The wide distribution of polyadenylation among living organisms indicates that this process evolved early in the history of life on Earth.

Polyadenylation Primer on RNA

RNAs are a type of large biological molecules, whose individual building blocks are called nucleotides. The name poly(

(for polyadenylic acid tail)[6] reflects the way RNA nucleotides are abbreviated, with a letter for the base the nucleoti contains (A for adenine, C for cytosine, G for guanine and U for uracil). RNAs are produced (transcribed) from a DNA

template. By convention, RNA sequences are written in a 5' to 3' direction. The 5' end is the part of the RNA molecule transcribed first, and the 3' end is transcribed last. The 3' end is also where the poly(A) tail is found on polyadenylate

Messenger RNA (mRNA) is RNA that has a coding region that acts as a template for protein synthesis (translation). Th

the mRNA, the untranslated regions, tune how active the mRNA is. There are also many RNAs that are not translated non-coding RNAs. Like the untranslated regions, many of these non-coding RNAs have regulatory roles

In prokaryotes and organelles In many bacteria, both mRNAs and non-coding RNAs can be polyadenylated. This poly(A) tail promotes degradation b

degradosome, which contains two RNA-degrading enzymes: polynucleotide phosphorylase and RNase E. Polynucleotid phosphorylase binds to the 3' end of RNAs and the 3' extension provided by the poly(A) tail allows it to bind to the RN

whose secondary structure would otherwise block the 3' end. Successive rounds of polyadenylation and degradation o

end by polynucleotide phosphorylase allows the degradosome to overcome these secondary structures. The poly(A) t also recruit RNases that cut the RNA in two.[63] These bacterial poly(A) tails are about 30 nucleotides long.

In as different groups as animals and trypanosomes, the mitochondria contain both stabilising and destabilising poly( Destabilising polyadenylation targets both mRNA and noncoding RNAs. The poly(A) tails are 43 nucleotides long on av

The stabilising ones start at the stop codon, and without them the stop codon (UAA) is not complete as the genome o polyadenylation at all.

encodes the U or UA part. Plant mitochondria only have destabilising polyadenylation, and yeast mitochondria have n

While many bacteria and mitochondria have polyadenylate polymerases, they also have another type of polyadenylat performed by polynucleotide phosphorylase itself. This enzyme is found in bacteria, mitochondria, plastids and as a

constituent of the archeal exosome (in those archaea that have an exosome). It can synthesise a 3' extension where majority of the bases are adenines. Like in bacteria, polyadenylation by polynucleotide phosphorylase promotes degra of the RNA in plastids and likely also archaea

Deadenylation

In eukaryotic somatic cells, the poly(A) tail of most mRNAs in the cytoplasm gradually get shorter, and mRNAs with s

poly(A) tail are translated less and degraded sooner. However, it can take many hours before an mRNA is degraded. deadenylation and degradation process can be accelerated by microRNAs complementary to the 3' untranslated regio translated. They are then activated by cytoplasmic polyadenylation after fertilisation, during egg activation.

mRNA. In immature egg cells, mRNAs with shortened poly(A) tails are not degraded, but are instead stored without b

In animals, poly(A) ribonuclease (PARN) can bind to the 5' cap and remove nucleotides from the poly(A) tail. The leve

access to the 5' cap and poly(A) tail is important in controlling how soon the mRNA is degraded. PARN deadenylates l

RNA is bound by the initiation factors 4E (at the 5' cap) and 4G (at the poly(A) tail), so this is why translation reduce

deadenylation. The rate of deadenylation may also be regulated by RNA-binding proteins. Once the poly(A) tail is rem the decapping complex removes the 5' cap, leading to a degradation of the RNA. Several other enzymes that seem to involved in deadenylation have been identified in yeast.

Back to top Guest


Posted: Mon Jun 28, 2010 11:50 am Post subject:

Q64. Efavirenz is an orally active, nonnucleoside reverse transcriptase inhibitor (nnRTI) that is specific for human immunod

virus, type 1 (HIV-1). It is indicated, in combination with other antiretroviral agents, for the treatment of HIV-1 infec

Other drugs in this class are delavirdine and nevirapine. Efavirenz has a terminal half-life of 5276 hours after a single

and 4055 hours after multiple dosing. The shorter elimination time after chronic dosing is a result of P450 enzyme ind that induces its own metabolism.

n reproduction studies, cynomolgus monkeys were administered oral efavirenz (60 mg/kg/day) throughout pregnanc

coital days 20150). This dose produced plasma drug concentrations similar to those achieved in humans with 600 mg Three of 20 exposed newborns had major congenital malformations, compared with 0 of 20 in nonexposed control mo

The defects observed were 1 case each of anencephaly and unilateral anophthalmia, microphthalmia, and cleft palate

pregnant rats, doses producing plasma concentrations similar to those in humans resulted in an increase in fetal reso

Neither mating nor fertility was impaired in rats at these doses. No teratogenic or toxic effects were observed in rabb doses producing plasma concentrations similar to those in humans

Back to top vinay.


Guest Posted: Mon Jun 28, 2010 11:51 am Post subject:

drug which is not antioxidant?? allopurinol trimetazidine amifostine chloroquine ans given amifostine... Amifostine is a pharmacological antioxidant used as a cytoprotectant in cancer chemotherapy and radiotherapy. It is

to protect normal tissues relative to tumor tissue against oxidative damage inflicted by cancer therapies by becoming

concentrated at higher levels in normal tissues. The degree to which amifostine nevertheless accumulates in tumors a

protects them against cancer therapies has been debated. Guidelines have been published that direct its use in chem and radiation, taking into consideration the concerns of tumor protection.

Back to top vinay.


Guest Posted: Tue Jun 29, 2010 12:46 pm Post subject:

Q68. curve of carus is the nomenclature given to ;; 1 normal pelvic axis;; 2 abnormal pelvic axis;; 3 foetal axis;; 4 position of coccyx Q69. TRANSCELLULAR BIOSYNTHESIS IS REPRESENTED BY ;; 1 prostaglandins;; 2 thromoxanes;; 3 leukotrienes;; 4 lipoxins;; Q70. NHALATIONAL AGENT THAT CAN BE USED IN SHOCK PATIENTS IS ;; 1 ISOFLURANE;; 2 CYCLOPROPANE;; 3 HALOTHANE;; 4 DESFLURANE Q71. WHICH of the following statement is true about ocd;; 1 thoughts are perceived as excessive and absurd;; 2 thoughts are inserted by some ext agency;; 3 obsessions are always accompanied by compulsions;; 4 socio-occupational functionning remains intact;;

Back to top vinay.


Guest Posted: Wed Jun 30, 2010 12:57 pm Post subject:

Q72. star gazer fetus , term is related to ;; 1 cephalic presentation;; 2 breech;; 3 transverse lie;; 4 face; ans- breech

star gazing position is Hyperextension of the fetal head in breech position. This can be evaluated with ultrasound. the head resting against the back of the neck. Caesarean delivery is absolutely necessary, because vaginal birth with the baby's head in this position confers a high spinal cord trauma and death Q73. which one of the following is attached to the lingula of the mandible;; 1 stylomandibular ligament;; 2 sphenomandibular ligament;; 3 pterygomandibular typhe;; 4 middle constrictor of pharynx Q74. best source of complement is ;; 1 horse;; 2 guinea pig;; 3 man;; 4 dog;

Less than 5% of breech babies have their heads in the "star gazing" position, face looking straight upwards and the b

Back to top vinay.


Guest Posted: Thu Jul 01, 2010 1:01 pm Post subject:

Q76. which of the following antiepileptic drugs has theshortest half-life;; 1 ethosuximide;; 2 phenobarbitol;; 3 phenytoin;; 4 valproate;; ans: 4 valproate ( ref: KDT 6th edition ) drug --half life 1.ethosuximide--48hours(adults), 32 hrs (children) 2.phenobarbitone---80 to 120 hours 3.phenytoin --12 to 24 hours 4.valproate --10 to 15 hours Q77. children having minimal change disease have selective proteinuria because of ;; 1 reduction of negative charge on GBM;; 2 EXTENSIVE DAMAGE TO THE gbm;; 3 DEPOSITS OF IGG AND C3 ON GBM;; 4 INCRESED MESANGIAL MATRIX IN THE GLOMERULI;; ans= reduction of negative charge on GBM REF- robbins PBD 8e chapter 20 extract

The current leading hypothesis is that minimal-change disease involves some immune dysfunction, eventually resultin elaboration of a cytokine that damages visceral epithelial cells and causes proteinuria. The ultrastructural changes po

primary visceral epithelial cell injury, and studies in animal models suggest the loss of glomerular polyanions. Thus, d

the charge barrier may contribute to the proteinuria. The actual route by which protein traverses the epithelial cell po

the capillary wall remains an enigma. Possibilities include transcellular passage through the epithelial cells, passage t residual spaces between remaining but damaged foot processes or through abnormal spaces developing underneath t have become detached from the basement membrane

portion of the foot process that directly abuts the basement membrane, or leakage through foci in which the epithelia

Back to top divakar .


Guest Posted: Fri Jul 02, 2010 1:39 pm Post subject:

Q78. CHROMOSOME WALKING IS A TECHNIQUE USED TO DESCRIBE;; 1 more chromosome around the nucleus;; 2 more pigment of chromosomal dna from one area of chrosome to another;; 3 a method used to locate a gene using a set of clones from a dna library;; 4 recombination between chromosomal dna of two different species;; answer 3

Chromosome walking is a technique to clone a gene (e.g., a disease gene) from its known closest markers. The close

marker (e.g., EST or a known gene) to the gene is used to probe a genomic library. A restriction fragment isolated fro

end of the positive clones is used to reprobe the genomic library for overlapping clones. This process is repeated seve to walk across the chromosome and reach the gene of interest. Q79. the adverse event that limits the use of flutamide as a primary treatment of bph is ;; 1 breast tenderness;; 2 diarrhoea;; 3 erectile dysfunction;; 4 loss of libido;; imp points from goodman n gillman ANDROGEN RECEPTOR ANTAGONISTS

Flutamide, Bicalutamide, and Nilutamide These relatively potent AR antagonists have limited efficacy when used alon because the increased LH secretion elevates serum testosterone concentrations. They primarily are used in conjunctio

GnRH analog in the treatment of metastatic prostate cancer, where they block the action of adrenal androgens that a

inhibited by GnRH analogs. AR antagonists used in this fashion include flutamide (EULEXIN), bicalutamide (CASODEX

nilutamide (NILANDRON). Bicalutamide is replacing flutamide for this purpose because it has less hepatotoxicity and once daily. Nilutamide apparently has worse side effects than flutamide and bicalutamide. Flutamide also has been us effectively to treat hirsutism in women, but the association with hepatotoxicity argues against its routine use for this purpose.

Monotherapy : In clinical studies, the most frequently reported adverse reactions to DROGENIL Tablets are gynaecom treatment or reduction in dosage. Q80. the commonest paritcles used in teletherapy are ;; 1 photons;; 2 neutrons;; 3 electrons;; 4 gamma radiation;;

and/or breast tenderness, sometimes accompanied by galactorrhoea. These reactions disappear upon discontinuation

X-rays consist of electromagnetic radiation of wavelengths shorter than those of ultraviolet rays and longer than thos gamma rays. X-ray wavelengths fall roughly in the range from 0.0110 nm (nanometers, or billionths of a meter). X-rays are produced artificially when high-speed electrons collide with a tungsten or other such heavy metal target. Q81. induction agent of choice for hyperthyroid patient ;; 1 propofol;; 2 thiopentone;; 3 ketamine;;

4 etomidate;; Therapeutic radiation is delivered in three ways: (1) teletherapy, with beams of radiation generated at a distance and aimed at the tumor within the patient; (2) brachytherapy, with encapsulated sources of radiation implanted directly into or adjacent to tumor tissues; and (3) systemic therapy, with radionuclides targeted in some fashion to a site of tumor. Teletherapy is the most commonly used form of radiation therapy.

Radiation from any source decreases in intensity as a function of the square of the distance from the source (inverse law). Thus, if the radiation source is 5 cm above the skin surface and the tumor is 5 cm below the skin surface, the in of radiation in the tumor will be 52/102, or 25% of the intensity at the skin . By contrast, if the radiation source is moved to 100 cm from the patient, the intensity of radiation in the tumor will 1002/1052[these 2 r squares], or 91% of the intensity at the skin. Teletherapy maintains intensity over a larger volume of target tissue by increasing the source-to-surface distance. In brachytherapy, the source-to-surface distance is small; thus, the effective treatment volume is small. X-rays and gamma rays are the forms of radiation most commonly used to treat cancer.

They are both electromagnetic, nonparticulate waves that cause the ejection of an orbital electron when absorbed. Th electron ejection is called ionization. X-rays are generated by linear accelerators; gamma rays are generated from decay of atomic nuclei in radioisotopes such as cobalt and radium These waves behave biologically as packets of energy, called photons

Particulate forms of radiation are also used in certain circumstances. Electron beams have a very low tissue penetran are used to treat skin conditions such as mycosis fungoides Neutron beams may be somewhat more effective than x-rays in treating salivary gland tumors.

However, aside from these specialized uses, particulate forms of radiation such as neutrons, protons, and negative m

which should do more tissue damage because of their higher linear energy transfer and be less dependent on oxygen not yet found wide applicability to cancer treatment.

Back to top Vinay.


Guest Posted: Sat Jul 03, 2010 12:22 pm Post subject:

Q82. most common site of atopic dermatitis in an infant is ;; 1 scalp;; 2 elbow;; 3 trunk;; 4 ante cubital fossa The distribution and skin reaction pattern varies according to the patient's age and disease activity

During infancy, the AD is generally more acute and primarily involves the face, scalp, and the extensor surfaces of th

extremities ( Fig. 122-7). The diaper area is usually spared.

In older children, and in those who have long-standing skin disease, the patient develops the chronic form of AD with lichenification and localization of the rash to the flexural folds of the extremities

AD often subsides as the patient grows older, leaving an adult with skin that is prone to itching and inflammation whe exposed to exogenous irritants. Chronic hand eczema may be the primary manifestation of many adults with AD Q83. FLUOPHER IS SEEN IN ;; 1 FLUROSIS;; 2 OSTEOPETROSIS;; 3 SCURVY;; 4 CARBON TETRAFLOURIDE POISONING; Q84. PAUCI-immune glomerulonephritis isseen in ;; 1 RPGN;; 2 IGA NEPHROPATHY;; 3 MICROSCOPIC POLYANGITIS;; 4 FSGS;; answer 3 microscopic polyangitis

group of patients with small-vessel vasculitis (arterioles, capillaries, and venules; rarely small arteries) and glomerulo

have serum ANCA; the antibodies are of two types, anti-proteinase 3 (PR3) or anti-myeloperoxidase (MPO) (Chap. 31 ANCA are produced with the help of T cells and activate leukocytes and monocytes, which together damage the walls vessels. Endothelial injury also attracts more leukocytes and extends the inflammation. Wegener's granulomatosis,

microscopic polyangiitis, and Churg-Strauss syndrome belong to this group because they are ANCA-positive and have

immune glomerulonephritis with few immune complexes in small vessels and glomerular capillaries. Patients with any three diseases can have any combination of the above serum antibodies, but anti-PR3 antibodies are more common i

Wegener's and anti-MPO antibodies are more common in microscopic polyangiitis or Churg-Strauss. While each of the

diseases have some unique clinical features, most features do not predict relapse or progression, and as a group they

generally treated in the same way. Only the presence of upper-airway involvement, persistent pulmonary injury, and antibodies suggests that the course of disease will be more difficult. Induction therapy usually includes some combina steroids are tapered soon after acute inflammation subsides, and patients are maintained on cyclophosphamide or azathioprine for up to a year to minimize the risk of relapse. Microscopic Polyangiitis

plasmapheresis, methylprednisolone, and cyclophosphamide. The benefit of plasmapheresis in this setting is uncertai

Clinically, these patients look somewhat similar to those with Wegener's granulomatosis, except they rarely have sign lung disease or destructive sinusitis. The distinction is made on biopsy where the vasculitis in microscopic polyangiitis without granulomas. Some patients will also have injury limited to the capillaries and venules. Q85. WHICH STAIN IS NOT USED FOR THE DIAGNOSIS OF ;;pneumocystis carinii infection;; 1 toluidine blue;; 2 giemsa;; 3 acridine orange;; 4 wright stain;;

Back to top vinay.


Guest Posted: Sun Jul 04, 2010 5:22 pm Post subject:

Q86. snowstorm knee is pathognomic of ;; 1synovial chondromatosis;;

2osteochondroitis dissecans;; 3fat embolism;; 4chondromalacia paptellae The aetiology of multiple loose bodies. Snow storm knee PR Kay, AJ Freemont, and DR Davies Park Hospital, Manchester, England. We report four patients who showed hundreds of brilliant white loose bodies at arthroscopy of the knee after a short

pain and crepitus. Histological, historical and clinical evidence is presented which indicates that the aetiology of this c

is the culture of chondrocytes in synovial fluid. It is suggested that reversal of the usually accepted order of events in

osteochondromatosis could provide a better and unified explanation for both that condition and multiple loose bodies. term 'snow storm knee' is proposed to describe the dramatic picture seen at arthroscopy.

Numerous white rice-grain size free bodies, presenting the so-called "snow storm appearance" were observed inside t

articular cavity of the left knee. Proliferation of the synovium on the ACL was identified and cartilaginous tissue was in Although there were numerous free bodies, no proliferation was found in the synovium except around the ACL, and n

damage was found in the meniscus and cartilage. The synovium that proliferated around the ACL was removed as mu

possible using a shaver and forceps, and free bodies that were assumed to be cartilaginous segments were washed o Q87. skier thumb is due to ;; 1injury to ulnar collateral ligament;; 2fracture of base of 1st metacarpal;; 3fracture of trapezoid;; 4fracture of scaphoid tubercle;; answer 1 The skier's thumb injury was described as an acute injury to the ulnar collateral ligament. When a skier falls with his

hand caught in a ski pole, the thumb can be pulled away from the hand. Because of the shape of the ski pole, the thu

tends to get caught and significant stresses are placed on the ulnar collateral ligament. If the ulnar collateral ligamen

pulled far enough, it will tear. While there are many ways to injure the ulnar collateral ligament, a skier's thumb is th eponym for an acute injury to the ligament.

The other injury is called a gamekeeper's thumb; this refers to a more chronic pattern of injury that leads to loosenin

ulnar ligament over time. The name comes from the European gamekeepers who would kill their game by grasping th of the animal between their thumb and index finger to break its neck. Over time, the ulnar collateral ligament is stret collateral ligament. Q88. the following are correct regarding omphalocele, except ;; 1is usually covered by a translucent membrane;; 2is frequently associated with other congenital malformations;; 3is lateral to the umbilical stump;; 4is within the umbilical ring answer 3 it is gastrochisis which is lateral to umblical stump not omphalocele Q89. which of the following is pathognomonic feature of t an abscess on ct ;; 1hypodense centre;; 2presence of air;; 3thick enhancing wall;; 4adjacent fat stranding;

would eventually cause problems. Again, gamekeeper's thumb is the proper eponym to describe chronic injuries to th

Q90. removal of proximal segments of the small intestine results in a decrease in ;; 1maximal acid output;; 2gastric emptying of liquids;; 3 gastric emptying of solids;; 4pancreatic enzyme secretion;;

with the removal of small intestine receptors which give information regarding composition of chyme to brain are rem and reflex secretion of pancreatic and liver enzyme is affected

Inflammation or removal of the upper small intestine leads to a decrease in pancreatic and hepatobiliary function. Th proximal small intestine contains a number of "receptors" that monitor the physical (volume) and chemical (pH, fat caloric density, osmolality) composition of the chyme emptied from the stomach. Stimulation of these receptors releases hormones and activates neural reflexes that initiate pancreatic enzyme and

bicarbonate secretion, stimulate gallbladder emptying, and provide feedback for inhibitory regulation of gastric functi (enterogastrone, enterogastric reflex). Removal of these reflexes decreases pancreatic secretion and gallbladder emptying and increases gastric emptying output. Q91. which one of the following is a cofactor and not a coenzyme;; 1biotin;; 2tetrahydrofolic acid;; 3copper;; 4 methycobalamine;;

A cofactor is a non-protein chemical compound that is bound to a protein and is required for the protein's biological a loosely-bound cofactors termed coenzymes and tightly-bound cofactors termed prosthetic groups. An inactive enzyme, without the cofactor is called an apoenzyme, while the complete enzyme with cofactor is the holoenzyme. cofactors: such as the metal ions Mg2+, Cu+, Mn2+ or iron-sulfur clusters. Q92. DRUG CAUSING OLIGOSPERMIA;; 1LEFLUONAMIDE;; 2D-PENICILLAMINE;; 3METHOTREXATE;; 4CORTICOSTEROIDS;;

Back to top vinay.


Guest Posted: Mon Jul 05, 2010 1:49 pm Post subject:

Q93. RUBOXISTAURIN IS A NEW DRUG BEING USED FOR , WHICH OF THE FOLLOWING CONDITIONS;; 1prevention of diabetic retinopathy;; 2prevention of hypertensive retinpathy;; 3anti-asthmatic drug;; 4lipid lowering agent

The most common sites of mandibular fracture are at the body (21-40%), condyles (15-20%), and angle (20-31%). addition, 10-15% affect the parasymphysis; 3-9%, the ramus; 3-5%, the alveolar ridge; and 1-2%, the coronoid pro

Again, these numbers vary depending on the characteristics of a given community. Automobile accidents are associat

higher percentages of condylar fractures, whereas motorcycle accidents are frequently associated with symphyseal an

parasymphyseal fractures. When the patient's history is of assault, body and angle fractures are more common.

watz most cmnly fractured site of mandible.............?? above frm emedicine... below wikipediaLocation * Condyle 30% * Angle 25% * Body 25% * Symphesis 15% * Ramus 3% * Coronoid process 2% The mandible may be dislocated anteriorly (to the front) and inferiorly (downwards) but ve

Back to top vinay.


Guest Posted: Mon Jul 05, 2010 1:51 pm Post subject:

Q94. THE CALCIUM CHANNEL blocker , which can reverse the resistance of cancer cells to chemotherapeutic drugs is 1verapamil;; 2diltiazem;; 3bepiridil;; 4nicardipine;;

a review of the pharmacological background and clinical activity of different Ca2+ channel blockers concerning revers

anticancer drug resistance in tumors suggests that verapamil and trifluoperazine may be the most immediate candida Reversal of resistance to anthracyclines and vinca alkaloids originally observed in mouse experimental leukemia cell l now been extended to other animal cell lines and human tumors as well----------Q95. oil- drop phenomenon is seen in ;; 1clubbing ;; 2psoriasis of nails;; 3lichen planus of nails;; 4tinea umguum;

A yellow oil droplet lesion in the nail is typical for psoriasis. Dermatophyte infection and lymphedema may also caus chest infections or lymphedema. Yellow nails

yellowing of the lateral nail border. A yellow nail syndrome, with all nails yellow without cuticles, can be seen in chr

Q96. JORDANS SIGN IS SEEN IN ;; 1.congenital dislocation of patella;; 2nail patella syndrome;; 3JRA;; 4PIGMENTED VILLONODULAR SYNOVITIS;; Q97. which of the following indices is not an index of measurement of obesity;; 1body mass index;; 2ponderal index;; 3broca index;; 4sullivans index; Q98 . tunica reaction is seen in ;; 1h.infuenja;;

2plague;; 3r.mooseri;; 4b;;anthrax

Endemic typhus there is swelling of scrotum and inflammation of tunica vaginalis in male guinea pig (Tunica reacti Rickettsia typhi, causes Endemic typhus

Back to top vinay.


Guest Posted: Mon Jul 05, 2010 1:54 pm Post subject:

Q99. GHENT CRITERIA IS USED IN DIAGNOSIS OF WHICH CONDITION ;; 1SCLERODERMA;; 2SLE;; 3MARFANS SYNDROME;; 4VASCULITIS;;

Marfan syndrome is currently diagnosed using criteria based on an evaluation of the family history, molecular data, a

organ systems. The diagnosis cannot be based on molecular analysis alone because molecular diagnosis is not genera available, mutation detection is imperfect, and not all FBN1 mutations are associated with Marfan syndrome. With the

previous Berlin criteria, Marfan syndrome was diagnosed on the basis of involvement of the skeletal system and 2 oth ectasia).

systems, with the requirement of at least one major manifestation (ie, ectopia lentis, aortic dilatation or dissection, o

Ghent criteria, they identify major and minor diagnostic findings, which are largely based on clinical observation of va

organ systems and on the family history. A major criterion is defined as one that carries high diagnostic precision bec

relatively infrequent in other conditions and in the general population. The Ghent criteria were intended to serve as a international standard for clinical and molecular studies and for investigations of genetic heterogeneity and genotypephenotype correlations. The clinical diagnosis in adults should be made using the Ghent criteria, which are unreliable children. The major criteria include the following: A first-degree relative (parent, child, or sibling) who independently meets the diagnostic criteria

Presence of an FBN1 mutation known to cause Marfan syndrome Inheritance of an FBN1 haplotype known to be associated with unequivocally diagnosed Marfan syndrome in the fami In family members, major involvement in one organ system and involvement in a second organ system Q100. endosperules may be observed in the tissues or fluids of patients infected with;; 1blastomyces dermatitidis;; 2coccidioides inmitis;; 3cryptococcus neoformans;; 4sporothrix schenckii;; Q101. THE CENTRE OF GRAVITY FOR THE HUMAN BODY IS ;; 1just ant to hip joint;; 2through the symphisis pubis;; 3just ant to s2;; 4lust posterior to l5;; Q102. which of the following is an activator of LCAT;; 1APOB100;; 2APOB48;; 3APOE;;

4APOA1; answer apo a1 Lysolecithin (lysophosphatidylcholine) may be formed by an alternative route that involves lecithin: cholesterol

acyltransferase (LCAT). This enzyme found in plasma, catalyzes the transfer of a fatty acid residue from the 2 positio

lecithin to cholesterol to form cholesteryl ester and lysolecithin and is considered to be responsible for much of the ch

ester in plasma lipoproteins. Long-chain saturated fatty acids are found predominantly in the 1 position of phospholip whereas the polyunsaturated acids (eg, the precursors of prostaglandins) are incorporated more into the 2 position. T incorporation of fatty acids into lecithin occurs by complete synthesis of the phospholipid, by transacylation between cholesteryl ester and lysolecithin, and by direct acylation of lysolecithin by acyl-CoA. Thus, a continuous exchange of acids is possible, particularly with regard to introducing essential fatty acids into phospholipid molecules. LCATand the LCAT activator apo A-I (Chapter 24). The nonpolar cholesteryl esters move into the hydrophobic interior of the bilayer, whereas lysolecithin transferred to plasma albumin. Thus, a nonpolar core is generated, forming a spherical, pseudomicellar HDL covered by a surface film of polar lipids and tissues.

bind to the disk, and the surface phospholipid and free cholesterol are converted into cholesteryl esters and lysolecith

apolipoproteins. In this way, the LCAT system is involved in the removal of excess unesterified cholesterol from lipop

Back to top vinay.


Guest Posted: Wed Jul 07, 2010 12:24 pm Post subject:

Q103. which tendon is most commonly involved in impingement syndrome ;; 1.bicipital tendon;; 2subscapularis;; 3supraspinatus;; 4infraspinatus supraspinatus tendon... impingment syndrome = supraspinatus tendonitis+ sub acromial bursitis Q104. complications of bronchial artery embolisation include all, except ;; 1cerebrovascular accident;; 2paraplegia;; 3renal failure;; 4pul monary infarcton Q105. hoffman -rigler sign is used for the assessment of ;; 1left ventricular enlargement;; 2right ventricular enlargement;; 3right atrial enlargement;; 4left atrial enlargement;; Hoffmann's sign A tingling sign is a sensation triggered by a mechanical stimulus in the distal part of an injured nerve. This sensation

peripherally, from the point where it is triggered to the cutaneous distribution of the nerve. The tingling response can unpleasant sensation is not a severe pain and does not persist. Q106. which of the following alcoholic drink has the most purine content;; 1.whisky;;

compared with that produced by a weak electric current, as in transcutaneous electrical nerve stimulation (TENS). Th

2vodka;; 3beer;; 4 rum;; Q107. PSEUDOHYPERKALAEMIA , CAUSES INCLUDE ALL, EXCEPT ;; 1prolonged tourniquette application ;; 2leukocytosis;; 3thrombocytosis;; 4hyporeninaemia Q108. which of the following statement is not true about chronic papilloedema;; 1collection of extracellular fluid;; 2stasis of axoplasmic transport;; 3disruption of neurofilament;; 4swelling of axon Q109. limits of confidence of a hypothesis is determined by ;; 1power factor;; 2level of significance;; 3 1-power factor;; 4 1 -level of significance

Back to top jain.


Guest Posted: Thu Jul 08, 2010 12:44 pm Post subject:

107. PSEUDOHYPERKALAEMIA, CAUSES INCLUDE ALL, EXCEPT ;; 1prolonged tourniquette application ;; 2leukocytosis;; 3thrombocytosis;; 4hyporeninaemia--causes hyperlalaemia

Pseudohyperkalemia is a rise in the amount of potassium that occurs due to excessive leakage of potassium from cell

or after blood is drawn. It is a laboratory artifact rather than a biological abnormality and can be misleading to caregi

Pseudohyperkalemia is typically caused by hemolysis during venipuncture (by either excessive vacuum of the blood d

by a collection needle that is of too fine a gauge); excessive tourniquet time or fist clenching during phlebotomy (whi presumably leads to efflux of potassium from the muscle cells into the bloodstream); or by a delay in the processing

blood specimen. It can also occur in specimens from patients with abnormally high numbers of platelets (>500,000/m whose blood must be separated immediately to avoid pseudohyperkalemia. 108. which of the following statement is not true about chronic papilloedema;; 1collection of extracellular fluid;; 2stasis of axoplasmic transport;; 3disruption of neurofilament;; 4swelling of axon collection of extracellular fluid "Electron microscopic studies indicate that most of the increase in tissue elevation that occurs in papilledema results intra-axonal swelling and not from extracellular edema. The intra axonal swelling is accompanied by an increase in mitochondria, disorganization of neurofilaments, and the accumulation of dense intracellular membrane-enclosed bodies."Walsh and Hoyt's Clinical Neuro-Ophthalmology, 2005 Answer is a, according to me

leukocytes (> 70 000/mm), or erythrocytes (hematocrit > 55%). People with "leakier" cell membranes have been fo

Back to top vinay.


Guest Posted: Thu Jul 08, 2010 12:49 pm Post subject:

Q110. which of the following charecteristics of an axon is most dependant on its diameter;; 1the magnitude of its resting potential;; 2the duration of its refractory period;; 3the conduction velocity of its action potential;; 4the overshoot of its action potential

Answer 3 Comparison of the conduction velocities of myelinated and unmyelinated axons in the cat. The conduction velocity v o myelinated neurons varies roughly linearly with axon diameter d (that is, v d), whereas the speed of unmyelinated varies roughly as the square root (v d). The red and blue curves are fits of experimental data, whereas the dotted are their theoretical extrapolations. Q111. which of the following antihypertensive drug shows therapeutic window phenomenon'' 1sodium nitroprusside;; 2clonidine;; 3methyldopa;; 4reserpine Answer 2

Therapeutic window phenomenon This is an unusual feature seen with certain drugs Optimal therapeutic effect is exe

over a narrow range of plasma drug concentrations or drug doses Both below & above this range, the beneficial effec

suboptimal. i.e; the effect declines if the dose is increased beyond a certain level E.g. tricyclics ( Imipramine), Clonid Glipizide Q112. which of the following sugars is absorbed from the small intestine by facilitated diffusion;; 1glucose;; 2galactose;; 3fructose;; 4sucrose Answer 3

Facilitated diffusion is a movement of nutrients against a concentration gradient and usually does not require energy, intestines.

require a carrier. Fructose absorption uses a carrier without expending energy pulling water with it upon entering the

Passive diffusion is a movement of compounds across the cell membrane so as to equalize the concentration of the su

on both sides of the membrane. This process applies only to such small molecules as electrolytes, water, and small su

does not apply to such large molecules as starch or large proteins. Almost all the water-soluble nutrients are absorbe manner or by osmosis (passive absorption).

Active transport is a process requiring energy (ATP) and a nutrient carrier to move an essential nutrient against a gra

Most carriers are nutrient-specific, and almost all required nutrients are actively transported. Minerals are the excepti they require both passive diffusion and carrier-mediated transport. Amino acids are actively transported, but also req carrier.

Pinocytosis or phagocytosis happens when the absorptive cell engulfs the material. This process is used for fat absorp

Back to top Guest


Posted: Fri Jul 09, 2010 12:54 pm Post subject:

Luxury perfusion phenomenon in acute herpes simplex virus encephalitis. Tanaka M, Uesugi M, Igeta Y, Kondo S, Sun X, Hirai S. Department of Neurology, Gunma University School of Medicine, Japan. Abstract

In a patient with acute herpes simplex virus (HSV) encephalitis, positron emission tomography (PET) demonstrated in

cerebral blood flow in the affected temporal lobe accompanied by reduction in the cerebral oxygen extraction fraction

cerebral metabolic rate of oxygen, i.e., luxury perfusion. Follow-up PET studies showed reduction in cerebral perfusio was more closely coupled with oxygen metabolism after the resolution of the acute inflammation. These findings supp

previous single photon emission computed tomographic data and provide a pathophysiological background for the occ disease other than cerebral ischemia.

of hyperperfusion in HSV encephalitis. This is an interesting example of the luxury perfusion phenomenon occurring in

Back to top vinay.


Guest Posted: Fri Jul 09, 2010 12:56 pm Post subject:

Q113. ALL OF THE FOLLOWING are features of the foetal hydantoin syndrome, except ;; 1dysmorphic facies;; 2hypoplasia of distal phal;anges and nails;; 3iugr;; 4renal dysplasia

About one third of children whose mothers are taking this drug during pregnancy typically have children who have int growth restriction with microcephaly and develop minor dysmorphic craniofacial features and limb defects including delay, or mental retardation. Rare side effects include methemoglobinemia. Fetal hydantoin syndrome was first described in 1973 by Loughnan et al. It consists of an array of anomalies, including craniofacial anomalies, distal digital hypoplasia, epicanthal folds, hypertelorism, low-set ears, and developmental delay.

hypoplastic nails and distal phalanges (birth defects). A smaller population will have growth problems and developme

The early descriptions of this syndrome were noted in 12 infants, 11 of whom had been exposed to other AEDs, notab barbiturates. Of infants born to women who used phenytoin during pregnancy, 10-30% are reported to exhibit some

syndromic findings, most commonly distal digital hypoplasia. However, few infants exposed to monotherapy have the constellation of findings. Of note, in a long-term neurodevelopmental study, 16 patients whose mothers received phe took carbamazepine and with normal controls. more on phenytoin

monotherapy during pregnancy demonstrated slightly delayed locomotor development compared with children whose

Varied Hand and foot defects include fingerlike thumbs, aplasia or hypoplasia of the distal phalanges, supernumerary

phalangeal epiphyses, and clubfoot.Dermatoglyphic abnormalities consist of abnormal palmar creases and nail hypop aplasia.[b] [b]General defects include growth retardation, atypical facial appearance, hirsutism, and low hairlines.Facial problem

microcephaly, brachycephaly, midfacial hypoplasia, wide fontanels, metopic ridging, mild micrognathia, low-set defor

ears, blepharoptosis, mild hypertelorism, strabismus, short nose with a broad depressed bridge and epicanthal folds, bow of the upper lip, and occasionally cleft lip and palate.

Reported torso abnormalities include short neck with mild webbing, widely spaced pages , umbilical or inguinal hernia anomalies.Internal abnormalities include variable coarctation of the aorta, endocardial cushion defect, double-outlet r ventricle, ventricular septal defect, atrial septal defect, bicuspid pulmonic valve, and intestinal malrotation. Ambiguous genitalia are rarely associated with this syndrome

.A patient with the dysmorphic characteristics of fetal hydantoin syndrome presented with unusual hyperpigmentation several fingernails.

Another neonate presented with gum hypertrophy, digitalization of the thumbs, hypoplasia of the distal phalanges an mild form of this syndrome. This syndrome may be associated with neonatal acne.

epicanthal folds, pseudohypertelorism, epidermoid cyst, and geographic tongue.Onychopathy can be a monosymptom

Studies: Phenytoin affects folate and vitamin K metabolism, which may increase fetal bleeding. Some investigators ha suggested that vitamin K be administered to mothers taking phenytoin in the third trimester to prevent hemorrhage.

However, the available evidence does not justify this policy of giving vitamin K throughout the last third of pregnancy

women being treated with anticonvulsants. When used in pregnancy, phenytoin can cause a syndrome of birth defect referred to with various names, such as Dilantin congenital defects, fetal hydantoin syndrome (FHS), Meadow syndro syndrome. Various malformations have been reported to occur because of phenytoin intake during pregnancy. Q114. TURP SHOULD COMMENCE WITH ;; 1incision of the bladder neck;; 2resectionm of the middle lobe;; 3resection of tissue at 12 oclock;; 4resection of tissue at 3 to 9 oclock;; 1.Approximately 25% of all candidates for TURP present with urinary retention and require preoperative catheter dra

congenital hydantoin syndrome, Dilantin syndrome, fetal Dilantin syndrome, fetal phenytoin syndrome, and hydantoin

Some of these men may develop postobstructive diuresis and other electrolyte disturbances, which require appropria management. An attempt should be made to lower elevated BUN and creatinine levels in patients who are thought to anticipated surgery.

azotemic due to urinary obstruction. For this, use continuous Foley catheter drainage for at least 2 weeks prior to any

2.Prolonged catheter drainage before a TURP may also be helpful in patients with decompensated hypotonic bladders respond well to medical management with alpha-blockade, 5-alpha reductase inhibitors, or both. 3.Preoperative urodynamic studies are only indicated in patients considered to have underlying neurologic disease or

them regain a more normal bladder capacity and muscle tone. Despite the severity of the obstruction, many of these

who may have detrusor decompensation from chronic or recurrent urinary retention. Urodynamic testing results can h

predict whether the chronically distended bladder has the capability to regain adequate muscle tone and capacity. On

technique is to leave an indwelling catheter for a period of time (usually 2-4 wk) and then remeasure the bladder cap voluntary voiding pressure. A reduction in bladder capacity or an increase in voluntary voiding pressure suggests the for reasonably normal bladder function eventually

4.The use of preoperative finasteride (Proscar) and dutasteride (Avodart) has been demonstrated to help reduce blee during and after TURP surgery, although the optimal timing is unclear. A recent prospective study found that 2-4 mon pretreatment with finasteride significantly increased immediate postoperative hemoglobin levels, but only in patients

resected prostatic weight was more than 30 grams, suggesting this may be most helpful in patients with larger prosta Finasteride (Proscar) and dutasteride (Avodart) also reduce recurrent episodes of hematuria secondary to BPH. They help control excessive or prolonged hematuria after prostatic needle biopsy. These 5-alpha-reducatase inhibitor medications decrease suburethral prostatic microvessel density, inhibit prostate

angiogenesis, and lower prostatic vascular endothelial growth factor, which are the likely mechanisms by which they prostatic bleeding.

5.Surgical alternatives to TURP are designed to decrease blood loss, inpatient hospitalization, and fluid absorption, wh removing or destroying the obstructing prostatic tissue. These include the vaporization TURP (VaporTrode), bipolar TURP, photoselective vaporization of the prostate (PVP), and holmium laser enucleation. PVP....

KTP laser energy at 532 nm is highly absorbed by oxyhemoglobin and only penetrates 1-2 mm deep into the prostati making it theoretically superior to other types of prostatic laser vaporization procedures. Modern KTP lasers produce penetration, compared to Nd:YAG lasers, minimizes the adverse effects associated with other types of prostate laser vaporization.

or more of average power and 240 watts of peak power, which vaporizes the prostatic tissue fairly rapidly. The limite

The PVP procedure has excellent hemostasis because the blood vessels are rapidly sealed by the KTP laser energy. No

is available for analysis, but the procedure can be performed relatively quickly. It can safely be performed even in pa who cannnot be taken off their anticoagulant medications, which is a major advantage over TURP. In the hands of sk operators, typical operating times are less than 1 hour for glands up to 100 grams.

General principles of transurethral prostate resection

Use of a plastic barrier sheath (eg, Lingeman sheath, O'Connor sheath) helps maintain sterility and protects the oper field, while allowing digital manipulation of the prostate through the rectum.

Make sure the patient is positioned with the buttocks flush with the end of the cystoscopy table. Resection of the ante

prostate opposite the bladder neck can be impaired when deflection of the resectoscope is restricted by the edge of th

cystoscopy table. Varying patient height to a comfortable level and using the Trendelenburg position appropriately sh make the resection easier and more comfortable and can facilitate visualization.

####procedure always begins with a careful cystoscopic inspection of the anterior urethra, external urinary sphincte

verumontanum, prostatic urethra, bladder neck, prostatic median lobe, trigone, ureteral orifices, and the rest of the b using a small-caliber cystoscope (see image below). This inspection is important not only to verify the absence of ass mental image of the patient's specific anatomical features and relationships.

pathologies (eg, bladder tumors, urethral strictures, vesical stones), but also to help the surgeon obtain a clear 3-dim

always know exactly where the verumontanum is located and do not resect it. Without this anatomical landmark, one

easily lose orientation and risk damaging the external sphincter muscle, causing the patient to be permanently incont

during the case you are not absolutely certain of your exact location, orientation, or position relative to the verumont stop resecting immediately and reorient by finding a stable landmark such as the bladder neck or verumontanum.

Irrigating fluid should be kept at the lowest height (pressure) level possible to maintain an adequate flow. Raising the

irrigating fluid level from 60 cm to 70 cm height has been shown to dramatically affect fluid absorption. A starting flu of 60 cm is suggested and is usually sufficient. PERFORMING RESECTION

Resect tissue only when pulling or withdrawing the cutting loop toward the resectoscope, never when pushing it forwa and extravasation.

cleanly separates the resected tissue from the rest of the prostate gland and prevents tunneling, perforation, bladder

If a very large and obstructing median lobe is present, resect this first regardless of the method chosen for the rest o transurethral resection

When resecting at the bladder neck, make sure enough fluid is present in the bladder to keep the posterior bladder w

from the surgical area. Approximately 100 mL prevents accidental posterior bladder wall injury and perforation. Befor orifices to prevent their inadvertent resection

resecting at the 4 to 5-o'clock and 7 to 8-o'clock positions at the bladder neck, visually check the position of the urete

When cutting the anterior tissue at the 12-o'clock position, making relatively shallow straight cuts is better, which lea

surface fairly flat, rather than trying to make it concave or curved. The prostate is thinnest here; the external sphinct

its most proximal position and the risk of perforation is relatively high, especially if this area is resected early in the c Try to resect all the prostatic tissue possible without perforating the capsule or unduly extending the operating time. much is taken out that causes the postoperative problems, it is how much is left in." Cautery involving the verumontanum is discouraged because it can result in painful ejaculation.

tissue can regrow, foster infection, increase fluid absorption, and tends to bleed. An old urology axion states "It is no

Do not attempt a TURP on a prostate that is too large for the procedure to be reasonably completed in 90 minutes of

time. Either do the case via an open technique, send the patient to a colleague with more experience in TURP surgery

perform some alternative procedure. If in the middle of the case and unable to complete the entire resection in 90 m

least finish one lateral lobe, the median lobe (if enlarged), and the bladder neck. This often results in very good clinic outcomes and offers essentially the same symptom relief as the completed TURP.

Leaving a small amount of nonobstructing prostate tissue near the verumontanum is much better than risking perma incontinence through an injury to the external sphincter muscle.

The most common area of injury to the external sphincter is at the 12-o'clock position, where direct visualization of t the 12-o'clock position opposite the verumontanum, to avoid inadvertent damage to the external sphincter. VARIOUS TECHNIQUES

verumontanum is not possible. Therefore, be extremely careful at the distal surgical margins, particularly when resec

The Nesbit technique is probably the best-known and most commonly performed TURP method. It was first described M. Nesbit (see image below) of Michigan in his landmark 1943 book on transurethral prostatectomy and is currently stages: (1) intravesical or proximal, (2) extravesical, and (3) apical.

considered the standard approach to TURP surgery.15 As originally described by Nesbit, the procedure is divided into

intravesical portion, the resectoscope is positioned with the tip between the bladder neck and the midpoint of the pro begins by removing the intravesical portion of the prostate and bladder neck tissue. This is removed, along with the immediately adjacent prostatic adenoma, starting at the 12-o'clock position and working clockwise

urethra proximal to the verumontanum. This point is determined by the relative size of the intravesical prostate. Rese

extravesical phase of the procedure, the resectoscope is repositioned just in front of the verumontanum, and the rese

continued from the previous distal resected margin to just proximal to the verumontanum, starting again at the 12-o

position. This channel is continued from the 12-o'clock to the 4-o'clock position on the left side and to the 8-o'clock p on the right side. The intent is to create a channel between the surgical capsule and the bulk lateral lobe tissue (see s

image below). This encircling maneuver, combined with the intravesical and bladder neck resection performed earlier

in the bulk of the 2 lateral lobes falling onto the floor of the prostatic fossa (see third image below). In addition, they

essentially devascularized and can be resected easily with minimal bleeding. The bulk of the prostate tissue is resecte this part of the procedure.

the final stage of the procedure is the apical stage, in which the remaining apical tissue around the verumontanum is

removed (see image below). Again, Nesbit starts the apical portion of the procedure with an anterior resection (12-o' position), using the verumontanum as the main landmark. MILNER TECHNIQUE

initial resection groove is made at the 9-o'clock position and carried down to capsular fibers. This groove extends from

bladder neck to a point parallel to the verumontanum at the level of the ejaculatory ducts. Coagulation is not used, e

major bleeding sites when the resection has reached the surgical capsule. The groove is first extended upwards towa 11-o'clock position and then downward toward the 7-o'clock position. No attempt is made to encircle the bulk tissue.

the intent is to resect the lateral lobe tissue from the inside out quickly and to reach the surgical capsule expeditiousl entire lobe is finished. The process is repeated on the opposite side final area of resection is the anterior tissue between the 11-o'clock and the 1-o'clock positions. The anterior resected

which point the perforating and bleeding vessels can be cauterized if necessary. The next section in line is resected u

is left relatively straight and flat to avoid perforation in this area. The risk of perforation when resecting the anterior t

relatively high because this is where the prostate is thinnest and no distal landmark is visible on the anterior surface, risks possible inadvertent injury to the external sphincter. Also, the anterior tissue is rarely obstructing; therefore, th resected last in case the procedure needs to be terminated early for any reason. CONCLUSION if prostate is large more than 100mg.. first to b dissected median lobe

n accord to nesbit tech which is stil best n most common tech resction shud commence at 4 TO 8 ie intravesical>>>> extravesical>>> apical so answer here after seein so many books n disc i m in favour of choice 4..IE LATERAL LOBE REMOVED FIRST nesbit is more common procedure n for choice 4 is accord to milner tech

Milner personally instructed this author in his method, which differs slightly from the Nesbit technique described earli the modern era, and it is offered here as a tribute to this highly skilled surgeon and teacher.

the author's knowledge, this is the first time his particular technique for transurethral prostatectomy has been publish

Q115. 3 MILLION PLAN WAS GIVEN BY WHICH COMMITTEE;; 1bhore committee;; 2kartar singh committee;; 3mudaliar committee;; 4chadah committee;; ANSWER bhore committe

his committee, known as the Health Survey & Development Committee, was appointed in 1943 with Sir Joseph Bhore Chairman. It laid emphasis on integration of curative and preventive medicine at all levels. It made comprehensive recommendations for remodeling of health services in India. The report, submitted in 1946, had some important recommendations like :1.Integration of preventive and curative services of all administrative levels. 2. Development of Primary Health Centres in 2 stages : 2 doctors, one nurse, four public health nurses, four midwives, four trained dais, two sanitary inspectors, two health

a. Short-term measure one primary health centre as suggested for a population of 40,000. Each PHC was to be ma

assistants, one pharmacist and fifteen other class IV employees. Secondary health centre was also envisaged to prov support to PHC, and to coordinate and supervise their functioning.

b. A long-term programme (also called the 3 million plan) of setting up primary health units with 75 bedded hospita each 10,000 to 20,000 population and secondary units with 650 bedded hospital, again regionalised around district with 2500 beds.

3. Major changes in medical education which includes 3 - month training in preventive and social medicine to prepare physicians. ___________________________________________________________________________________________ MUDALIAR COMMITTEE. 1962

This committee known as the Health Survey and Planning Committee, headed by Dr. A.L. Mudaliar, was appointed t the performance in health sector since the submission of Bhore Committee report. This committee found the conditio opened.

PHCs to be unsatisfactory and suggested that the PHC, already established should be strengthened before new ones a

Strengthening of sub divisional and district hospitals was also advised. It was emphasised that a PHC should not be m

cater to more than 40,000 population and that the curative, preventive and promotive services should be all provided Indian Medical service. ____________________________________________________________________________________________ CHADHA COMMITTEE, 1963.

PHC. The Mudaliar Committee also recommended that an All India Health service should be created to replace the ers

This committee was appointed under chairmanship of Dr. M.S. Chadha, the then Director General of Health Services, about the necessary arrangements for the maintenance phase of National Malaria Eradication Programme. The comm

suggested that the vigilance activity in the NMEP should be carried out by basic health workers (one per 10,000 popu vital statistics data collection under supervision of family planning health assistants. _____________________________________________________________________________________ MUKHERJEE COMMITTEE. 1965. The recommendations of the Chadha Committee, when implemented, were found to be impracticable because the ba workers, with their multiple functions could do justice neither to malaria work nor to family planning work

who would function as multipurpose workers and would perform, in addition to malaria work, the duties of family plan

. The Mukherjee committee headed by the then Secretary of Health Shri Mukherjee, was appointed to review the perf

in the area of family planning. The committee recommended separate staff for the family planning programme. The f

planning assistants were to undertake family planning duties only. The basic health workers were to be utilised for pu other than family planning.

The committee also recommended to delink the malaria activities from family planning so that the latter would receiv undivided attention of its staff. ___________________________________________________________________________________________ MUKHERJEE COMMITTEE. 1966.

Multiple activities of the mass programmes like family planning, small pox, leprosy, trachoma, NMEP (maintenance ph etc. were making it difficult for the states to undertake these effectively because of shortage of funds. A committee o health secretaries, headed by the Union Health Secretary, Shri Mukherjee, was set up to look into this problem. The committee worked out the details of the Basic Health Service which should be provided at the Block level, and some consequential strengthening required at higher levels of administration ___________________________________________________________________________________________ JUNGALWALLA COMMITTEE, 1967

This committee, known as the Committee on Integration of Health Services was set up in 1964 under the chairman

Dr. N Jungalwalla, the then Director of National Institute of Health Administration and Education (currently NIHFW). I asked to look into various problems related to integration of health services, abolition of private practice by doctors in

government services, and the service conditions of Doctors. The committee defined integrated health services as :-

a. A service with a unified approach for all problems instead of a segmented approach for different problems. b. Medical care and public health programmes should be put under charge of a single administrator at all levels of hiera __________________________________________________________________________________ KARTAR SINGH COMMITTEE. 1973.

his committee, headed by the Additional Secretary of Health and titled the "Committee on multipurpose workers unde and Family Planning" was constituted to form a framework for integration of health and medical services at periphera supervisory levels. Its main recommendations were :-

a. Various categories of peripheral workers should be amalgamated into a single cadre of multipurpose workers (male female). The erstwhile auxiliary nurse midwives were to be converted into MPW(F) and the basic health workers, mal

surveillance workers etc. were to be converted to MPW(M). The work of 3-4 male and female MPWs was to be superv supervisor.

one health supervisor (male or female respectively). The existing lady health visitors were to be converted into femal

b One Primary Health Centre should cover a population of 50,000. It should be divided into 16 subcentres (one for 30 3500 population) each to be staffed by a male and a female health worker.

_____________________________________________________________________________________________ SHRIVASTAV COMMITTEE. 1975. This committee was set up in 1974 as "Group on Medical Education and Support Manpower" to determine steps need

reorient medical education in accordance with national needs & priorities and (ii) develop a curriculum for health assis who were to function as a link between medical officers and MPWs. It recommended immediate action for : 1. Creation of bonds of paraprofessional and semiprofessional health workers from within the community itself. 2. Establishment of 3 cadres of health workers namely multipurpose health workers and health assistants between community level workers and doctors at PHC. 3. Development of a Refferal Services Complex

4. Establishment of a Medical and Health Education Commission for planning and implementing the reforms needed in and medical education on the lines of University Grants Commission.

Acceptance of the recommendations of the Shrivastava Committee in 1977 led to the launching of the Rural Health S ___________________________________________________________________________________ BAJAJ COMMITTEE, 1986.

"Expert Committee for Health Manpower Planning, Production and Management" was constituted in 1985 under Dr. J. the then professor at AIIMS. Major recommendations are :1. Formulation of National Medical & Health Education Policy. 2. Formulation of National Health Manpower Policy. 3. Establishment of an Educational Commission for Health Sciences (ECHS) on the lines of UGC. 4. Establishment of Health Science Universities in various states and union territories. 5. Establishment of health manpower cells at centre and in the states.

6. Vocationalisation of education at 10+2 levels as regards health related fields with appropriate incentives, so that g quality paramedical personnel may be available in adequate numbers.

7. Carrying out a realistic health manpower survey.


Posted: Sat Jun 26, 2010 1:12 pm Post subject:

Q58. The most recently advocated thery of PEM pathogenesis is ;; 1 theory of adaptation;; 2 theory of free radicals;; 3 classical thery of pem;; 4 gopalans thery Q59. TRACHYONYCHIA IS SEEN IN ALL, EXCEPT ;; 1. LICHEN PLANUS;; 2 PSORIASIS;; 3 ALOPECIA AREATA;; 4 PARONYCHIA Twenty-nail dystrophy is also known as trachyonychia. It could be said that twenty-nail dystrophy is widespread trachyonychia involving all 20 nails. The condition is characterised by longitudinal ridging (alternating elevation and depression), pitting, loss of lustre, and roughening (similar to sandpaper) of the nail surface. Twenty-nail dystrophy most commonly occurs in childhood. In one study the condition was most evident in young males in the 10-20 year old age group (52%). Slight nail abnormalities may be evident at birth with the condition slowly progressing over the years to cause changes in the texture of fingernails and toenails. Typical signs and symptoms include: * Nails become grubby, rough and brittle * Some nails may become distorted * Nail examination shows longitudinal ridging, pitting, roughening and splitting * Nails lose their lustre and may change to a muddy, greyish-white colour (sand-blasted appearance) The cause of twenty-nail dystrophy is unknown but in some cases it appears to be associated with other skin conditions such as lichen planus, eczema, psoriasis and alopecia areata. In some cases an autosomal dominant pattern of inheritance has been found; in one report twenty-nail dystrophy occurred in 4 males in 3 successive generations. Other cases of twenty-nail dystrophy are of unknown origin and begin gradually in early childhood. These cases tend to be self-limiting and may resolve slowly with age. Q60. Biliary leak after liver transplantation is associated with, which of the following vascular complications;; 1 portal vein thrombosis;; 2 hepatic vein thrombosis;; 3 hepatic artery thrombosis;; 4 mesenteric ischemia;; Q61. 24 hrs refrigeration test is done to assess;; 1 hyperuricaemia;; 2 hyperlipoprotaenaemia;; 3 hypercalcaemia;; 4 determining etiology of jaundice lipemia-refrigeration test refrigeration of a lipemic blood sample may distinguish between triglyceride-rich lipoproteins, which persist in the turbid sample, and chylomicra, which rise to form a flocculent top layer while the sample clears

Q62. PAS STAINS ALL THE FOLLOWING, EXCEPT ;; 1 GLYCOGEN;; 2 LIPIDS;; 3 FUNGAL CELL WALL;; 4 BASEMENT MEMBRANE OF BACTERIA answer 2 Periodic Acid-Schiff (PAS) This histology stain is particularly useful for staining glycogen and other carbohydrates, but is useful for many things. It is often used to show glomeruli, basement membranes, and glycogen in the liver. PAS stains glycogen, mucin, mucoprotein, and glycoproteins magenta. The nuclei will stain blue. Collagen will stain pink. Q63. Which of the following drug binds to bcl-2 messenger rna causing fragmentation of the protein message ;; 1 oblimersen sodium;; 2 bevacizumab;; 3 endostatin;; 4 angiostatin Genasense (oblimersen sodium) Injection inhibits the production of a protein known as Bcl-2. By reducing production of Bcl-2 in cancer cells, Genasense seeks to restore the biological process whereby cancer cells are killed by standard anticancer treatments a process known as apoptosis. Genasense is an investigational anticancer agent undergoing testing to evaluate its safety and efficacy in various forms of cancer when used in conjunction with standard therapies. Studies are currently underway to examine the potential role of Genasense in a variety of clinical indications.

Posted: Sat Jul 10, 2010 12:38 pm

Post subject:

Q116. tendency of colonic carcinoma to metastasize is best assessed by a.size of tr b.cea levels c.depth off penetration of bowel wall d.prop of bowel circumfrence involv Q117. hair perforation test is done for ;; 1microsporum canis;; 2trichophyton mentagrophyte;; 3epidermophyton;; 4trichophyton rubrum B. To distinguish between isolates of dermatophytes, particularly Trichophyton mentagrophytes and its variants. Ingredients: Autoclaved blonde pre-pubital hair cut into short pieces (1cm) Sterile distilled water 5 ml in a suitable vial. Method: 1. Place hair in water in vial. 2. Inoculate with small fragments of the test fungus.

3. Incubate at room temperature. 4. Individual hairs are removed at intervals up to 4 weeks and examined microscopically in lactophenol cotton blue. Isolates of T. mentagrophytes produce marked localised areas of pitting and marked erosion whereas those of T. rubrum do not. Q118. septated uteri are most commonly surgically repaired using ;; 1straussman uteroplasty;; 2hysteroscopic resection ;; 3laser laparscopy;; 4tompkins metroplasty Q119. which of the following primary tumors is most likely to metastasise to the ovary ;; 1breast cancer;; 2thyroid cancer;; 3astrocytoma;; 4osteosarcoma;; answer breast Metastatic tumours, accounting for about 5% of ovarian malignancies, and usually arise from breast, colon, endometrium, stomach and cervical cancers Q120. in mseleni joint disease -the joint commnly affected is ;; 1knee joint;; 2hip joint;; 3ankle joint;; 4wrist joint Mseleni joint disease (MJD) is an unusual form of progressive and widespread degenerative osteoarthropathy that has been identified in several hundred people in the remote Mseleni region of northern Zululand. Affected individuals experience articular discomfort in childhood and may be seriously handicapped as adults, often requiring prosthetic hip joint replacement Q121. allopurinal is used in the treatment of ;; 1trypanosomiasis;; 2leishmaniasis;; 3amoebiasis;; 4trichomoniasis; Q122. which is not a feature of CARNEY COMPLEX;; 1MYXOMA;; 2PIGMENTATION;; 3SCWANNOMA;; 4THYROID ADENOMA ANSWER SCHWANNOMA Carney complex (also known as "LAMB syndrome," and "NAME syndrome") is an autosomal dominant condition comprising myxomas of the heart and skin, hyperpigmentation of the skin (lentiginosis), and endocrine overactivity DIFFERENTIATED FROM carney triad... Carney triad, as originally described in 1977, is the association of 3 tumors: gastric gastrointestinal stromal tumor (gastric GIST) (ex-gastric epithelioid leiomyosarcoma), extra-

adrenal paraganglioma, and pulmonary chondroma. gastric stromal tumor in Carney triad showed distinctive features: female predilection, young patient age, epithelioid cell predominance, multifocality, frequent lymph node metastasis, serial tumor occurrence, and unpredictable behavior. A123.

Back to top

vinay.
Guest

Posted: Sun Jul 11, 2010 5:19 pm

Post subject:

Q123. which of the following is a retinoid selective receptor;; 1tretinoin;; 2alitreinoin;; 3isotretinoin;; 4bexaratone;; Bexarotene is a retinoid specifically selective for retinoid X receptors, as opposed to the retinoic acid receptors. Retinoids have been effective in vitro against many types of cancer, including that of the breast . However, many breast cancers become resistant to the chemotherapeutic effects of these drugs . Recently, RXR-selective ligands were discovered that inhibited proliferation of all-trans RA resistant breast cancer cells in vitro and caused regression of the disease in animal models. These RXR-selective ligands did not produce the side effects that are normally associated with retinoid chemotherapy . However, a phase II trial using one of these ligands (bexarotene) showed limited response in patients with metastatic breast cancer. Interestingly, a high-risk lesion for invasive breast cancer, ductal carcinoma in situ, has been shown to overexpress RXR- A retinoid X receptor (RXR)-selective retinoid reveals that RXRalpha is potentially a therapeutic target in breast cancer cell lines, and that it potentiates antiproliferative and apoptotic responses to peroxisome proliferator-activated receptor ligands. Q124. systemic disorder associated with hyperprolactinaemia , are all , except ;; 1CRF;; 2CIRRHOSIS;; 3EPILEPSY;; 4CHF;; Q125. WHICH OF THE FOLLOWING SITES CONTAINS -striated muscle that is not under voluntary control;; 1bladder;; 2esophagus;; 3gallbladder;;

4 stomach; Esophageal skeletal muscle is under control of motor neurons originating in the nucleus ambiguus in the brain stem, The cervical esophagus is composed of striated muscle that is directly innervated by lower motor neurons Q126. all of the following are true about -FAP. EXCEPT ;; 1fundic polyps are not premalignant;; 2the gene associates is a tumor suppressor gene;; 3if left untreayed colonic cancer occurs in virtually 100 percent affected;; 4is the most common hereditary colorectal cancer Q127. MOST SENSITIVE METHOD FOR DIAGNOSING CHRONIC PANCREATITIS;; 1CECT ABDOMEN;; 2MRI;; 3ERCP;; 4EUS;; Q128. STEINSTRASSE OCCURRENCE CAN BE COUNTERED BY ;; 1doing a ureterostomy;; 2placing a ureteric stent;; 3doing multiple repeated basketting procedures;; 4none of the above;; Percutaneous nephrostomy tube placement to the patient under local anesthesia will provide symptomatic relief. With proximal diversion, the steinstrasse often passes spontaneously.

Back to top dishan.


Guest Posted: Mon Jul 12, 2010 12:58 pm Post subject:

Q126. Explanation

Gastric polyps are usually an incidental finding on endoscopy, detected in 2% to 3% of gastroscopic evaluations. Fundi sessile lesions in the body and fundus, most commonly in healthy gastric mucosa.

polyps constitute 47% of all gastric polyps and have no malignant potential. Typically, they present as multiple 2- to 3-

Most cases are sporadic, but gastric polyps can occur in 53% of patients with familial adenomatous polyposis or Gardne to 60% of patients with gastric fundic gland polyps

syndrome. Although the polyps themselves are non-neoplastic, retrospective studies have reported colorectal neoplasm

Tumor suppressor genes produce proteins that inhibit tumor formation by regulating mitotic activity and providing inhib

cycle control. Tumor formation occurs when these inhibitory controls are deregulated by mutation. Point mutations, los

heterozygosity (LOH), frame-shift mutations, and promoter hypermethylation are all types of genetic changes that can

failure of a tumor suppressor gene. These genes are often referred to as gatekeeper genes because they provide cell cy inhibition and regulatory control at specific checkpoints in cell division. The failure of regulation of normal cellular funct to initiate tumor formation.

tumor suppressor genes is appropriately described by the term loss of function. Both alleles of the gene must be nonfu

The adenomatous polyposis coli (APC) gene is a tumor suppressor gene located on chromosome 5q21.

Its product is 2843 amino acids in length and forms a cytoplasmic complex with GSK-3 (a serine-threonine kinase),

and axin. -Catenin, a multifunctional protein, is a structural component of the epithelial cell adherens junctions and th

cytoskeleton; it also binds in the cytoplasm to Tcf/LEF and is then transported into the nucleus, where it activates trans of genes like c-myc and others that regulate cellular growth and proliferation. APC therefore participates in cell cycle control by regulating the intracytoplasmic pool of -catenin.

Wnt signaling proteins are closely associated with the APC--catenin pathway. APC also influences cell cycle proliferatio regulating Wnt expression. Wnt gene products are extracellular signaling molecules that help regulate tissue developm conditions, reduced intracytoplasmic -catenin levels inhibit Wnt expression.

throughout the organism. The Wnt signaling proteins are closely associated with the APC--catenin pathway. Under no

When APC is mutated however, -catenin levels rise, and Wnt is activated. Overexpression of Wnt leads to activation o target genes such as cyclin D1 and Myc, which drive cell proliferation and tumor formation earliest mutations in the adenoma-carcinoma sequence occur in the APC gene. ###NEW MCQ###

The earliest phenotypic change present is known as aberrant crypt formation, and the most consistent genetic aberrati within these cells are abnormally short proteins known as APC truncations. Most clinically relevant derangements in AP truncation mutations created by inappropriate transcription of premature termination codons.

A germline APC truncation mutation is responsible for the autosomal dominantinherited disease, FAP.

Thirty percent of cases of FAP are de novo germline mutations, thus presenting without a family history of the disease.

rare, with an estimated incidence of 1 in 8000 of the U.S. population, occurring without gender predilection . It is classically characterized by greater than 100 adenomatous polyps present in the colon and rectum. These polyps number in the thousands and are almost always manifest by the late second or early third decade of life

AIIMS MAY 2010

The variability of the FAP phenotype is also demonstrated by the presence or absence of extraintestinal manifestations

disease. In the past, the term Gardener's syndrome was used to describe the coexpression of profuse colonic adenoma polyps along with osteomas of the mandible and skull, desmoid tumors of the mesentery, and periampullary neoplasms Many other associated disorders have been subsequently described, including thyroid papillary tumors medulloblastomas, hypertrophic gastric fundic polyps, and congenital hypertrophy of the retinal pigmented epithelium of the iris (CHRPE) when the truncation occurs in a very small area of the mutational cluster region

. The expression of extraintestinal manifestations of FAP is dependent on mutation location, with most of these signs se

IMP MCQ POINTS

Larger adenomas are found to harbor cancers more often than smaller ones, and the larger the polyp, the higher the ri cancer.

Although the cellular characteristics of the polyp are important, with villous adenomas carrying a higher risk than tubul

adenomas, the size of either polyp is also important. The risk for cancer in a tubular adenoma smaller than 1 cm in dia less than 5%, whereas the risk for cancer in a tubular adenoma larger than 2 cm is 35%. A villous adenoma larger tha size carries a 50% chance of containing a cancer

Removal of polyps decreases the incidence of cancer. Patients with small adenomas have a 2.3 times increased risk for undergo polypectomy

after the polyp is removed, compared with an 8-fold increased incidence of colorectal cancer in patients with polyps wh

Patients with FAP will develop colorectal cancer virtually 100% of the time in the absence of surgical intervention. ##ch this mcq## The adenomas that characterize this syndrome are histologically the same as sporadic adenomas.

The peak incidence for the discovery of benign colorectal polyps is 50 years of age. The peak incidence for the develop colorectal cancer is 60 years of age. This suggests a 10-year time span for the progression of an adenomatous polyp to cancer. It has been estimated that a polyp larger than 1 cm has a cancer risk of 2.5% in 5 years, 8% in 10 years, and 20 years.

Rare extraintestinal malignancies in FAP patients include cancers of the extrahepatic bile ducts, gallbladder, pancreas, thyroid, and liver. An interesting marker for FAP is CHRPE, which can be detected by indirect ophthalmoscopy in about 75% of affected individuals.

The gene is expressed in 100% of patients with the mutation. Autosomal dominance results in expression in 50% of of There is a negative family history in 10% to 20% of affected individuals, who apparently acquire the syndrome as the r a spontaneous mutation. All patients with the defective gene will develop cancer of the colon if left untreated.

The average age of discovery of a new patient with FAP is 29 years. The average age of a patient who is newly discove have colorectal cancer related to FAP is 39 years. Eponymous polyposis syndromes now recognized to belong to the general disorder of FAP include Gardner's syndrome polyps, epidermal inclusion cysts, osteomas) and Turcot's syndrome (colonic polyps and brain tumors). Osteomas usually present as visible and palpable prominences in the skull, mandible, and tibia of individuals with FAP. or congenitally absent teeth.

virtually always benign. Radiographs of the maxilla and mandible may reveal bone cysts, supernumerary and impacted

Desmoid tumors can present in the retroperitoneum and abdominal wall of affected patients, usually after surgery. The small intestine can result in death. SURGERY

tumors seldom metastasize but are often locally invasive, and direct invasion of the mesenteric vessels, ureters, or wal

Surgical treatment of patients with FAP is directed at removal of all affected colonic and rectal mucosa. Restorative proctocolectomy with IPAA has become the most commonly recommended operation. The procedure is usually accomp

a distal rectal mucosectomy to ensure that all premalignant colonic mucosa is removed, and the IPAA is fashioned betw

ileal pouch and the dentate line of the anal canal. Patients who undergo this procedure for FAP have a better functional

than patients similarly treated for ulcerative colitis, in that the incidence of inflammation in the ileal pouch (pouchitis) i lower in patients with FAP than in patients with ulcerative colitis.

less spreaded disease ..... abdominal colectomy and ileorectal anastomosis is the observation that sulindac and celecox been observed to cause the regression of adenomatous polyps in some patients with FAP. The disadvantages are that t

rectum remains at high risk for the formation of new precancerous polyps, a proctoscopic examination is required ever

months to detect and destroy any new polyps, and there is a definite increased risk for cancer arising in the rectum wit

passage of time There have been sporadic reports of regression of desmoid tumors after treatment with sulindac, tamoxifen, low-dose

methotrexate, radiation, and various types of chemotherapy. The initial treatment is usually with sulindac or tamoxifen #########HNPCC is the most frequently occurring hereditary colorectal cancer syndrome in the United States and history of colorectal cancer###########

Europe. It accounts for about 3% of all cases of colorectal cancer and for about 15% of such cancers in patients with a

ANSWER IS CHOICE D

Back to top dishan.


Guest Posted: Mon Jul 12, 2010 12:58 pm Post subject:

Q127. Explanation

Radiographs or CT scans showing pancreatic calcifications are diagnostic of chronic pancreatitis . Those calcifications re deposition of calcium carbonate in the intraductal protein plugs that frequently, but not invariably, occur in chronic pan Thus, the absence of pancreatic calcifications does not rule out a diagnosis of chronic pancreatitis.

Perhaps the most sensitive methods for diagnosing chronic pancreatitis are those that provide images of the pancreatic system. ERCP, CT cholangiopancreatography, or magnetic resonance cholangiopancreatography may be particularly va the diagnosis of chronic pancreatitis

chronic pancreatitis can be shown to have major ducts that have the appearance of a chain of lakes or a string of pe is the result of segments of dilated duct separated by areas of ductal stricture. Transcutaneous and endoscopic ultrasou Ultrasound examination is more operator dependent and perhaps less sensitive than either CT or MRI

also be used to diagnose chronic pancreatitis if duct dilation, calcifications, pseudocysts, or parenchymal fibrosis are se

Back to top vinay.


Guest Posted: Mon Jul 12, 2010 1:04 pm Post subject:

Q129. which of the following artery shows maximum frequency of arteriographic abnormalities in takayasu arteritis;; 1subclavian artery;; 2aortic arch;; 3common carotid artery;; 4 coronary artery; answer def subclavian artery

Takayasu's arteritis is an inflammatory and stenotic disease of medium- and large-sized arteries characterized by a str predilection for the aortic arch and its branches. For this reason, it is often referred to as the aortic arch syndrome. most prevalent in adolescent girls and young women. Although it is more common in Asia, it is neither racially nor geographically restricted. The disease involves medium- and large-sized arteries, with a strong predilection for the aortic arch and its branches;

pulmonary artery may also be involved. The most commonly affected arteries seen by angiography are listed in Table 3 The involvement of the major branches of the aorta is much more marked at their origin than distally

The disease is a panarteritis with inflammatory mononuclear cell infiltrates and occasionally giant cells. There are mark

intimal proliferation and fibrosis, scarring and vascularization of the media, and disruption and degeneration of the elas changes in various organs reflect the compromise of blood flow through the involved vessels.

lamina. Narrowing of the lumen occurs with or without thrombosis. The vasa vasorum are frequently involved. Patholog

Disease-related mortality most often occurs from congestive heart failure, cerebrovascular events, myocardial infarctio

aneurysm rupture, or renal failure. Even in the absence of life-threatening disease Takayasu's arteritis can be associate

significant morbidity. The course of the disease is variable, and although spontaneous remissions may occur, Takayasu arteritis is most often chronic and relapsing. Although glucocorticoid therapy in doses of 4060 mg prednisone per day

alleviates symptoms, there are no convincing studies that indicate that they increase survival. The combination of gluc

therapy for acute signs and symptoms and an aggressive surgical and/or angioplastic approach to stenosed vessels has

markedly improved outcome and decreased morbidity by lessening the risk of stroke, correcting hypertension due to re artery stenosis, and improving blood flow to ischemic viscera and limbs. Unless it is urgently required, surgical correcti

stenosed arteries should be undertaken only when the vascular inflammatory process is well controlled with medical th encouraging results

individuals who are refractory to or unable to taper glucocorticoids, methotrexate in doses up to 25 mg per week has y

Takayasu's arteritis (also known as "Aortic arch syndrome," and "Pulseless disease":841) is an inflammatory disease w

unknown cause. It affects the aorta, the main blood vessel from the heart, as well as the blood vessels that attach to it

Although it has been reported worldwide, it is more common in young Asian women. Females are about 8-9 times mor get it than males. People usually get the disease between 15 and 30 years of age. In the Western world, atherosclerosi more frequent cause of obstruction of the aortic arch vessels than is Takayasu's arteritis Q130. the nucleus involved in papez circuit is ;; 1pulvinar;; 2intralaminar;; 3vpl nucleus;; 4ant nucleus;; Q131. structure that cross the midline is ;; 1arteirathyroidea ima;; 2median sacral artery;; 3zenkers diverticulum;; 4left gonadal vein;;

median sacral artery (or middle sacral artery) is a small vessel, which arises from the back of the aorta, a little above i bifurcation. It descends in the middle line in front of the fourth and fifth lumbar vertebr, the sacrum and coccyx, and ends in the coccygeum (coccygeal gland). From it, minute branches are said to pass to the posterior surface of the rectum. On the last lumbar vertebra it anastomoses with the lumbar branch of the iliolumbar artery; in front of the sacrum it anastomoses with the lateral sacral arteries, and sends offsets into the anterior sacral foramina. Q132. DALBAVANCIN - IS MAINLY BEING USED FOR WHICH INFECTION;; 1PSEUDOMONAS;; 2MRSA;; 3KLEBSIELLA;; 4ANAEROBE INF

Dalbavancin is a novel second-generation lipoglycopeptide antibiotic. It belongs to the same class as vancomycin, the m (MRSA).

widely-used and one of the few treatments available to patients infected with methicillin-resistant Staphylococcus aure

It possesses in vitro activity against a variety of Gram-positive pathogens including MRSA and MRSE It is a once-weekl dose antibiotic .

Q133. COMPARED TO the other antidepressant drugs , mirtazapine has distinct ability to act as an antagonist of ;; 1beta receptors;; 2d2 receptors;; 3alpha 2 receptors;; 45-ht receptors;; answer 3 mitrazapine tetracyclic antidepressant (TeCA) used primarily in the treatment of depression. It is also sometimes used

anxiolytic, hypnotic, antiemetic, appetite stimulant, and antihistamine, among other indications. Along with its close an

mianserin and setiptiline, mirtazapine is one of the few noradrenergic and specific serotonergic antidepressants (NaSSA no significant act on dopamine receptor Q134. TERMINAL PHALANX RESORPTION IS NOT SEEN IN ;; 1HYPERPARATHYROIDISM;; 2REITERS SYNDROME;; 3SCLERODERMA;; 4PSORIASIS;; Q135. ALL OF THE FOLLOWING CAN BE DONE FOR FEMORAL HERNIA REPAIR, EXCEPT ;; 1lockwood repair;; 2mc evedy repair;; 3lothessions repair;; 4beclard repair; Lockwoods infra-inguinal approach Lotheissens trans-inguinal approach McEvedys high approach answer is obviously beclard repair.. beclard is hernia thru saphenous opening

Lockwood is remembered for his surgical work with femoral and inguinal hernias. He developed an infra-inguinal appro femoral hernia operations that is known today as the low approach or Lockwood's operation Classically three approaches are described to open femoral hernia repair: Lockwood's infra-inguinal, Lotheissen's trans-inguinal and McEvedy's high approach. Each approach describes a separate skin incision and dissection to access the femoral sac. lockwood repair...

The infra-inguinal approach is the preferred method for elective repair, approaching the femoral canal from below throu oblique incision 1 cm below and parallel to the inguinal ligament. This approach however offers little scope for resecting compromised bowel.

lotheissen ... The trans-inguinal approach involves a skin incision 2 cm above the inguinal ligament, dissecting through the inguinal c may form later which would be difficult to repair. In addition, if necrotic bowel is encountered the risk of infection may the use of synthetic mesh to repair the inguinal canal and predispose to inguinal hernia occurrence. mc evedy....

thus weakening this important structure. The danger with this, particularly in the presence of wound infection, is that a

high approach involves an oblique skin incision 3 cm above the pubic tubercle running laterally to cross the lateral bord

rectus muscle, that is divided allowing preperitoneal dissection of the sac. This approach is preferred in the emergency when strangulation is suspected allowing better access to and visualisation of bowel for possible resection

Q136. A 40 yr old female p3+0 is observed to have CIN grade 3 on colposcopic biopsy .which is best management? (AI 09/AIIMS 10) a)cryosurgery b)conization c)LEEP d)Hysterectomy

Back to top vinay.


Guest Posted: Tue Jul 13, 2010 1:07 pm Post subject:

Q136. glycoproteins are degraded by ;; 1ubiquitins;; 2cathepsins;; 3lysosomes;; 4peroxisomes;; Q137. master slave system is related to ;; 1SILS;; 2NOTES;; 3HALS;; 4ROBOTS;; Q138. which of the following 5ht 3 receptor blockers has highest receptor affinity;; 1ondansetron;; 2granisetron;; 3dolaserton;; 4palonosetron;; Q139. which is not correct with ref to histopathology of psoriasis;; 1hyperkeratosis;; 2hypergranulosis;; 3parakeratosis;; 4microabscess;;

Psoriasis is a chronic relapsing disease of the skin characterized by variable clinical features. The cutaneous lesions are

so distinct that a clinical diagnosis is easy to make. The lesions are classified as erythrosquamous, which indicates that vasculature (erythema) and the epidermis (increased scale formation) are involved. Inverse psoriasis . The skin in the intertriginous areas is highly erythematous, and typical scaling is lacking.

A total of 2147 patients suffering from various types of psoriasis were examined and the age of onset was recorded. Mo

patients of both sexes had an early onset [age 16 (F) and 22 (M)], while nearly one-quarter had a late onset (age 56).

Psoriasis also may present in a pustular form. There is a generalized form, usually referred to as pustular psoriasis (von Zumbusch), and a localized variant, confined to the palms and soles, known as pustulosis palmaris et plantaris (PPP; se with pustules) ###new mcq###

70). In rare instances in psoriasis of the plaque type or guttate psoriasis, pustules may develop after acute relapses (p

First signs appear in males at a mean age of 29 and in females at age 27. A study of the onset of psoriasis in 2400 pat

showed a peak incidence at 22.5 years of age; a second peak of onset around age 55 was found in 11.8 percent of the

4 (see Fig. 42-5A).

In a study of psoriasis in 245 children, the mean age of onset was 8.1 years, 5 and in a census study from the Faeroe I the mean age of onset was 12.5 years.

An early onset (before age 15) predicts more severe disease relative to the percentage of body surface involved with p

and the response to therapy. Also, the earlier the onset, the greater is the probability of a positive family history of pso

The HLA types most frequently reported to be associated with psoriasis are HLA-B13, HLA-Bw57, HLA-Cw6, and HLA-D hla cw6 newly discoverd and mc in india ref iams CLINICAL POINTS four prominent features: (1) they are sharply demarcated with clear-cut borders, (2) the surface consists of noncoherent silvery scales, (3) under the scale the skin has a glossy, homogeneous erythema, (4) there is an Auspitz sign.

The size of a single lesion varies from a pinpoint to plaques that cover large areas of the body. The clinical presentation psoriasis is better understood when it is realized that disease activity can range from a chronic stationary phase, to a r

process, to flares of disease that may be associated with numerous sterile pustules. The Auspitz sign is a specific featu

erythrosquamous lesion of psoriasis. It is noted when the hyperkeratotic scales are mechanically removed from a psori

plaque by scraping. Within a few seconds after mechanical removal of the scale, small blood droplets appear on the shi erythematous surface ). The Auspitz sign has diagnostic value; it is not present in inverse or pustular psoriasis and ma differentiate psoriasis from other skin conditions with a similar morphology. The Auspitz phenomenon in its three phases: . Native psoriatic lesion. 1 Scratching generates silvery-opaque scale. 2. Further scratching leads to removal of the scale, and a glossy area is visible. 3 Further scratching produces blood droplets

1. the proximal nail fold 2. caused by psoriatic processes located in the nail bed 3. Severe onychodystrophy results in yellowish keratinous material. This morphologic pattern is believed to be secondary psoriasis involving the nail matrix Common Laboratory Abnormalities in Psoriasis

Pits are evident within the nail plate. This morphologic pattern apparently is due to defective keratinization of the dorsa

Yellowish macules beneath the nail plate often extend distally toward the hyponychium. This morphologic pattern appe

Serum uric acid is elevated in up to 50 percent of patients and is mainly correlated with the extent of lesions and the a disease. There is an increased risk of developing gouty arthritis. Elevated serum uric acid levels usually cease after the

In severe psoriasis vulgaris, generalized pustular psoriasis, and erythroderma, a negative nitrogen balance can be dete mainly consisting in a decrease of serum albumin.

C-reactive protein, a 2-macroglobulin, as well as sedimentation rate, can be increased in psoriasis related to disease ac and body involvement

. Increased serum IgA levels and IgA immune complexes have been observed in psoriasis, the role of this phenomenon being unclear mild anemia Histopathology of psoriasis.

A. The earliest changes in psoriasis consist of papillary edema with a marked infiltrate of mononuclear cells, which tend invade the spongiotic epidermis. Later, neutrophils follow. B. Pustular psoriasis with large spongiform abscesses in the subcorneal zone without marked epidermal hyperplasia. C. Neutrophils migrating into the stratum corneum form so-called Munro's microabscesses.

D. In plaque-type psoriasis, large numbers of CD3+ T cells are present and tend to migrate into the affected epidermal portions characteristic feature of involved skin of psoriatic subjects is hyperproliferation, first defined by Van Scott and Ekel.

Currently, there is evidence of more than an eightfold shortening of the epidermal cell cycle (36 h versus 311 h for nor involved skin of patients with psoriasis. Further, there is a twofold increase in the proliferative cell population, and 100 subjects. These alterations result in a hyperplastic epidermis generating 35,000 cells/mm per day from a proliferative

of the germinative cells of the epidermis appear to enter the growth fraction compared with 60 to 70 percent for norma

compartment containing approximately 52,000 cells/mm of skin surface. Normal skin produces only 1218 cells/mm per from a proliferative compartment of 26,000 cells/mm ****** Clinical Classification of Psoriasis******

PSORIASIS VULGARIS, CHRONIC STATIONARY PSORIASIS, PLAQUE-TYPE PSORIASIS This clinical pattern is the most f little alteration in shape or distribution of individual plaques. Areas of predilection are the elbows , the knees, the scalp and, in particular, the retroauricular region, the lumbar area umbilicus.

Red scaly lesions, as described earlier, persist for months to years. There is constant production of large amounts of sc

Single small lesions may become confluent, forming plaques in which the borders resemble a land map (psoriasis geog gyrata). Occasionally there is partial central clearing, resulting in ringlike lesions (annular psoriasis) Eruptive (guttate) psoriasis

(see Fig. 42-3B). Lesions may extend laterally and become circinate because of the confluence of several plaques (psor

Typically, this pattern presents as small (0.51.5 cm in diameter) lesions over the upper trunk and proximal extremitie form is characteristic of psoriasis of an early age of onset and as such is found frequently in young adults. As noted bel

streptococcal throat infection frequently precedes the onset or flare of guttate psoriasis. Occasionally, a disseminated m

drug eruption may precede this pattern of psoriasis. Very active lesions of psoriasis of many types can have pustules th

to 2 mm in diameter and are surrounded by an intensive wall of erythema. This process usually signals an acute exace of disease. Predisposing factors for such an event are bacterial infection, aggressive local therapy, or withdrawal of systemic glucocorticoids

Psoriatic erythroderma Psoriatic erythroderma

represents the generalized form of the disease that affects all body sites, including the face, hands, feet, nails, trunk, a

extremities. [/b ]Although all the symptoms of psoriasis are present, erythema is the most prominent feature, and scal

usually is less severe compared with chronic stationary psoriasis. Psoriatic erythroderma may have different degrees of

activity, presenting suddenly as a generalized erythema or evolving gradually from chronic plaque psoriasis into a gene exfoliative phase. In the latter phase there are usually some areas of uninvolved skin. Annular pustular psoriasis

A rare variant of pustular psoriasis is an annular, or circinate, form of the disease occurring during episodes of pustular may develop during the course of generalized pustular psoriasis. The main features are pustules on a ringlike erythema that sometimes resembles erythema annulare centrifugum. Histologically, there is mild acanthosis and a neutrophil accumulation with formation of microabscesses.

eruptions. Lesions may appear at the onset of pustular psoriasis, with a [b]tendency to spread and form enlarged rings

Identical lesions are found in patients with impetigo herpetiformis, a pustular form of psoriasis associated with pregnan Generalized pustular psoriasis (von Zumbusch)

Pustular psoriasis of the von Zumbusch type ( Fig. 42-11) appears as a distinctive acute variant of psoriasis. It is unusu

other forms of psoriasis on the skin at the same time. Attacks of pustular psoriasis are characterized by fever that lasts

days. A sudden generalized eruption of sterile pustules 2 to 3 mm in diameter parallels the onset of fever. The pustules disseminated over the trunk and extremities, including the nail beds, palms, and soles. The pustules usually arise on h erythematous skin, first as patches and then becoming confluent as the disease becomes more severe.

In addition to the pustule formation of the nail matrix and loss of the entire nail, the fingertips may become atrophic in

with prolonged disease. As with other forms of psoriasis, the face usually remains free of lesions. The erythema that su the pustules often spreads and becomes confluent, leading to erythroderma Topical Treatment

ANTHRALIN Anthralin (1,8-dihydroxyanthrone; cignolin, dithranol) was introduced by Galewsky and Unna in 1916. This compound is still a widely used remedy for psoriasis in different vehicles and application modes. A major advantage of is the lack of any long-term side effects,

Mode of action Anthralin possesses antiproliferative activity on human keratinocytes. In recent years it has become cle

this compound also exerts strong anti-inflammatory effects mainly on cells of the inflammatory infiltrate. Inhibition of n

and monocyte functions and production and ?-oxidation of leukotriene B 4 from neutrophils have been observed. It wa

recently that anthralin induces nuclear transcription factor NF-? B in murine keratinocytes. Since NF-?B is involved in th transcription of proinflammatory cytokines such as IL-6, IL-8, and TNF-a, these findings may be helpful in explaining th properties of anthralin.

VITAMIN D 3 AND ANALOGUES Since the first report about the beneficial effects of vitamin D 3 in psoriasis by Morimot new analogues have been developed to decrease hormonal effects on calcium/phosphate homeostasis and to maintain

on keratinocyte proliferation and differentiation. Soon after the introduction of the first vitamin D 3 analogue, calcipotri

called calcipotriene in some countries), to topical therapy, it became a widely used remedy for plaque-type psoriasis. A vitamin D 3 analogue, tacalcitol, is available in some countries. Recently, active hormone 1,25-dihydroxyvitamin D 3 (c was registered for topical psoriasis treatment in a number of countries.

Mode of action Vitamin D 3 and its analogues inhibit keratinocyte proliferation and induce terminal differentiation. Antiinflammatory properties of these compounds include inhibition of nuclear factor NF-?B protein in lymphocytes, leading

reduced transcription of IL-2. Calcitriol and calcipotriol can inhibit production of IL-6 from cytokine-stimulated human d dendritic cell differentiation

microvascular endothelial cells and reduce the antigen-presenting function of Langerhans cells. Calcitriol is a potent inh

TAZAROTENE Tazarotene is a retinoid for topical use that reduces mainly scaling and plaque thickness, with limited

effectiveness on erythema. In comparison with other topical drugs for psoriasis, tazarotene has a lower efficacy. Howev efficacy can be enhanced by combination with UVB therapy.

TAR The use of tar either as coal tar or wood tar (birch, pine, beech) has a long history in antipsoriatic therapy. These contain a great variety of compounds, most of which are not well defined. Very little is known about how tars act in ski These preparations are nonirritant, and serious side effects are not seen even after long periods of treatment.

Preparations of 2% to 5% tar in various bases have been shown to be preferentially effective in chronic plaque-type ps

BALNEOPHOTOTHERAPY Empirically, it has been known that the combination of salt-water bathing and sunlight exposu effective treatment for psoriasis

SELECTIVE UVB THERAPY Treatment with UVB without UVA, also known as selective UVB phototherapy (SUP), can be performed as monotherapy or preferably in combination with topical treatments such as glucocorticoids, vitamin D 3 an

analogues, tazarotene, or anthralin. SUP is very effective in guttate psoriasis and also improves lesions of the plaque ty psoriasis. systemic therapy mtx,cyclosporine.glucocorticoids, retinoids lik.. Etretinate and acitretin are clinically effective in pustular forms of psoriasis FUMARIC ACID ESTERS A mixture of fumaric acid monoethyl and dimethyl esters is approved in Germany for systemic treatment of severe psoriasis. Clinical experience exists since 1959 when fuamrates were introduced.

Today, narrow-band UVB treatment (311 nm, Philips TL01 bulbs) has become a standard therapy for plaque-type and g

Mode of action Inhibition of TNF-ainduced keratinocyte ICAM-1 expression by dimethylfumarate has been demonstrat

Monomethylfumarate, the main metabolite of dimethylfumarate, was shown to stimulate release of T H2 cytokines IL-4 5 from human peripheral blood T cells without changing production of T H1 cytokines IL-2 and interferon-?.

Dimethylfumarate potently inhibits dendritic cell differentiation. Recently, dimethylfumarate was shown to induce apop number of cells, including dendritic cells. Furthermore, this compound inhibits cytokine production by blocking NF?B sig BATH PUVA

Another way to deliver the photosensitizer (8-MOP or 5-MOP) to the skin is by addition of these compounds to bath wa described by Fischer and Alsins in 1976. Major advantages of bath PUVA are the lack of systemic effects, such as

gastrointestinal complaints (nausea is present in about 13 percent of patients taking 8-MOP orally), and the overall red

UV dose down to one-quarter of that required to obtain therapeutic results similar to those of conventional PUVA, thus the risk of nonmelanoma skin cancer. Furthermore, erythema is less frequent in bath PUVA patients, and eye protectio sunglasses is not required. Q140. WHICH OF THE FOLLOWING cytokine downregulates the immunological response;; 1il-1;; 2il-2;; 3il-6;; 4 il-10

Interleukin-10 (IL-10 or IL10), also known as human cytokine synthesis inhibitory factor (CSIF), is an anti-inflammator cytokine. In humans IL-10 is encoded by the IL10 gene.

This cytokine is produced primarily by monocytes and to a lesser extent by lymphocytes. This cytokine has pleiotropic e immunoregulation and inflammation. It down-regulates the expression of Th1 cytokines, MHC class II antigens, and costimulatory molecules on macrophages. Q141. ORNITHINE CITRULLINE TRANSPORTER DEFECT CAUSES ALL, EXCEPT ;; 1HYPERAMMONAEMIA;; 2HYPERORNITHINEMIA;;

3HYPERHOMOCITRULLINEMIA;; 4HYPERCITRULLINAEMIA;; Q142. THE WORLD HEALTH DAY THEME FOR THE YEAR 2010 IS ;; 1HEALTH FACILITIES IN EMERGENCIES;; 2PROTECT HEALTH FROM CLIMATE CHANGES;; 3URBANISATION AND HEALTH;; 4 NONE OF THE ABOVE;; Q143. THE POSTEROMEDIAL corner of the knee joint is very vital to its rotatory stability ;;it includes all, except ;; 1post oblique ligament;; 2multiple insertions of semimembranosus;; 3oblique popliteal ligament;; 4the arcuate ligament;; Q144. the first response of osmoreceptors to hyperosmolality is 1renin secretion;; 2 thirst;; 3vasopressin secretion;; 4 none;;

Back to top vinay.


Guest Posted: Wed Jul 14, 2010 12:59 pm Post subject:

Q145. treatment for diabetic retinopathy is ;; 1primordial prevention;; 2primary prevention;; 3sec prevention;; 4tertiary prevention Q146. carisoprodol activation forms -which metabolite ;; 1amphetamine;; 2meprobomate;; 3doxylamine;; 4dimethadione;; meprobomate

Carisoprodol is a centrally-acting skeletal muscle relaxant. It is a colorless, crystalline powder, having a mild characteri and a bitter taste. Carisoprodol is slightly soluble in water and freely soluble in alcohol, chloroform and acetone. The dr solubility is practically independent of pH.

carisoprodol has abuse potential as a product of meprobamate and/or potentiator of hydrocodone, dihydrocodeine, cod similar drugs, it continues to be prescribed in North America, alongside orphenadrine and cyclobenzaprine. In Europe, d used in Canada.

favor cyclobenzaprine. In the United Kingdom, benzodiazepines are preferred instead. All of the above plus chlorzoxazo

As of November 2007, carisoprodol (Somadril, Somadril comp.) has been taken off the market in Sweden due to proble dependence and side effects. The agency overseeing pharmaceuticals has considered other drugs used with the same market in Norway as well Q147. finger in glove appearance on cect chest is found in ;; 1sarcoidosis;; 2emphysema;;

indications as carisoprodol to have the same or better effects without the risks of the drug. In May 2008 it was taken o

3allergic bronchopulmonary aspergillosis;; 4interstitial lung diseases;;

The finger in glove sign seen on CXR and CT chest and refers to the characteristic sign of a bronchocoele, as seen in al bronchopulmonary aspergillosis (ABPA). Rarely a similar appearance can occur with bronchial atresia. The same appearance has also been referred to as: rabbit ear appearance Mickey Mouse appearance toothpaste shaped opacities Y-shaped opacities V-shaped opacities Q148. maddery score is used as prognostic indicator in ;; 1chronic hepatitis b;; 2chronic hepatitis c;; 3alcoholic hepatitis;; 4primary biliary cirrhosis;; Maddreys Discriminant Function for Alcoholic Hepatitis Pt's Prothrombin Time (PT) sec Lab Control Prothrombin Time (Pt) sec Pt's Serum Total Bilirubin Discriminant Function = 4.6 * (Pt's PT - Control PT) + TBili >32 points indicates poor prognosis and pt may benefit from glucocorticoid therapy. the modified Maddrey's discriminant function) was originally described by Maddrey and Boitnott to predict prognosis in hepatitis. It is calculated by a simple formula: Prospective studies have shown that, it is useful in predicting short term prognosis especially mortality within 30 days. more than 32 implies poor outcome with one month mortality ranging between 35% to 45%.

Back to top vinay.


Guest Posted: Thu Jul 15, 2010 12:48 pm Post subject:

Q149. which one of the GAGS plays an imp role in the develpment of atherosclerotic plaque;; 1hyaluronic acid;; 2chondroitin sulphate;; 3dermatan sulphate;; 4heparin

The International HapMap Project is an organization whose goal is to develop a haplotype map (HapMap) of the human researchers to find genetic variants affecting health, disease and responses to drugs and environmental factors Entrez Gene is NCBI's repository for gene-specific information

genome, which will describe the common patterns of human genetic variation. The HapMap is expected to be a key res

Blastx Use the BLAST algorithm to compare the six-frame conceptual translation products of a nucleotide query sequen

strands) against a protein sequence database. The BLAST (Basic Local Alignment Search Tool) programs have been de

for speed to find high scoring local alignments. BLAST uses a heuristic algorithm which seeks local as opposed to globa alignments and is therefore able to detect relationships among sequences which share only isolated regions of similarit (Altschul et al., 1990). Because of its design for speed, there may be a minimal loss of sensitivity to distant sequence relationships.

Hyaluronic acid may be important in permitting tumor cells to migrate through the ECM. Tumor cells can induce fibrobl

synthesize greatly increased amounts of this GAG, thereby perhaps facilitating their own spread. Some tumor cells hav

heparan sulfate at their surfaces, and this may play a role in the lack of adhesiveness that these cells display. The intim arterial wall contains hyaluronic acid and chondroitin sulfate, dermatan sulfate, and heparan sulfate proteoglycans. Of t

proteoglycans, dermatan sulfate binds plasma low-density lipoproteins. In addition, dermatan sulfate appears to be the dermatan sulfate may play an important role in development of the atherosclerotic plaque.

GAG synthesized by arterial smooth muscle cells. Because it is these cells that proliferate in atherosclerotic lesions in a

Q150. the genetic factors that contribute to susceptibility to inf are elucidated by ;; 1HAPMAP;; 2ENTREZ GENE;; 3BLASTX;; 4RFLP;; Q151. dr milind has ansered q no 150 correctly;; which of the following diseaqse worsens during pregnancy;; 1ulcerative colitis;; 2thyrotoxicosis;; 3rheumatoid arthritis;; 4bronchial asthma

Rheumatoid arthritis: RA may begin during pregnancy or, even more often, during the postpartum period. Preexisting R generally abates temporarily during pregnancy. The fetus is not specifically affected, but delivery may be difficult if the woman's hip joints or lumbar spine is affected. If a woman develops an RA flare during pregnancy, first-line treatment begins with prednisone other immunosuppressive agents may be required

****.Myasthenia gravis: Myasthenia gravis varies in its course during pregnancy. Frequent acute myasthenic episodes

require increasing doses of anticholinesterase drugs (eg, neostigmine ), which may cause symptoms of cholinergic exce

abdominal pain, diarrhea, vomiting, increasing weakness); atropine may then be required. Sometimes myasthenia bec refractory to standard therapy and requires corticosteroids or immunosuppressants. During labor, women may need as

ventilation and are extremely sensitive to drugs that depress respiration (eg, sedatives, opioids, Mg sulfate). Because t not had a thymectomy

responsible for myasthenia crosses the placenta, transient myasthenia occurs in 20% of neonates, even more if mothe

Immune thrombocytopenic purpura (ITP): ITP , mediated by maternal antiplatelet IgG, tends to worsen during pregnan

increases risk of maternal morbidity. Corticosteroids reduce IgG levels and cause remission in most women, but improv

sustained in only 50%. Immunosuppressive therapy and plasmapheresis further reduce IgG, increasing platelet counts splenectomy is required for refractory cases; it is best done during the 2nd trimester, when it causes sustained remissi about 80% Q152. the following are genetic risk factors for rheumatic fever, except ;; 1HLA CLASS 1 ALLELES;; 2HIGH LEVEL EXPRESSION OF D8/17'' 3HIGH LEVELS OF MANNOSE BINDING LECTIN;; 4POLYMORPHISM OF THE TGF BETA ONE GENE Genetic host risk factors include the D8/17 B-cell antigen and certain class II histocompatibility antigens

N-Acetylglucosamine (GlcNAc) is the major immunoepitope of group A streptococcal cell wall carbohydrates. Antistrept antibodies cross-reactive with anti-GlcNAc and laminin are present in sera of patients with rheumatic fever. The cross-r

of these antibodies with human heart valvular endothelium and the underlying basement membrane has been suggeste

a possible cause of immune-mediated valve lesion. Mannose-binding lectin (MBL) encoded by the MBL2 gene, a soluble

pathogen recognition receptor, has high affinity for GlcNAc. We postulated that mutations in exon 1 of the MBL2 gene associated with a deficient serum level of MBL may contribute to chronic severe aortic regurgitation (AR) of rheumatic

Molecular mimicry between streptococcal and human proteins is considered as the triggering factor leading to autoimm rheumatic fever (RF) and rheumatic heart disease (RHD). Here, we present a review of the genetic susceptibility marke Several human leucocyte antigen (HLA) class II alleles are associated with the disease. Among these alleles, HLA-DR7

involved in the development of RF/RHD and the major immunopathological events underlying the pathogenesis of RF a

predominantly observed in different ethnicities and is associated with the development of valvular lesions in RHD patien Cardiac myosin is one of the major autoantigens involved in rheumatic heart lesions and several peptides from the LMM TNF- and IFN- from heart-infiltrating mononuclear cells suggests that Th-1 type cytokines are the mediators of RHD lesions while the presence of few interleukin-4 producing cells in the valve tissue contributes to the maintenance and progression of the valvular lesions. POLYMORPHISM OF THE TGF BETA ONE GENE...is associated wth rheumatoid arthritis

meromyosin) region were recognized by peripheral and intralesional T-cell clones from RF and RHD patients. The produ

Association of transforming growth factor-1 gene polymorphism in the development of Epstein-Barr virus-related hem diseases

Back to top vinay.


Guest Posted: Fri Jul 16, 2010 12:13 pm Post subject:

Q153. the most common organism implicated in interleukin -1 receptor associated kinase defect is ;; 1n.meningitidis;; 2n.gonorrhoea;; 3s;pneumoniae;; 4proteus vulgaris susceptibility to pyogenic bacteria such as staphylococci, streptococci, clostridia; resistant to mycobacteria; autosomal recessive Q154. the following drug has maximum propensity to cause -peripheral neuropathy;; 1.DIDANOSINE;; 2ZIDOVUDENE;; 3STAVUDINE;; 4LAMIVUDINE STAVUDINE;;

The main severe adverse effect is peripheral neuropathy, which can be corrected by reducing dosage. Stavudine has be

shown in laboratory test to be genotoxic, but with clinical doses its carcinogenic effects are non-existent. It is also one most likely antiviral drugs to cause lipodystrophy, and for this reason it is no longer considered an appropriate treatme most patients in developed countries.

It is still used as first choice in first line therapy in resource poor settings such as in India. Only in case of development peripheral neuropathy or pregnancy is it changed to the next choice - Zidovudine. DIDANOSINE;

The most common adverse events with didanosine are diarrhea, nausea, vomiting, abdominal pain, fever, headache an Peripheral neuropathy occurred in 21-26% of participants in key didanosine trials.

Pancreatitis is rarely observed but has caused occasional fatalities, and has black box warning status. Other reported s

adverse events are retinal changes, optic neuritis and alterations of liver functions. The risk of some of these serious ad events is increased by drinking alcohol.

In February of 2010, the United States Food and Drug Administration issued a statement that patients using Didanosin

are at risk for a rare but potentially fatal liver disorder, non-cirrhotic portal hypertension

Didanosine (ddI; 2',3'-dideoxyinosine) was the second drug licensed for the treatment of HIV infection, followed shortly

thereafter by zalcitabine. Didanosine is metabolized to dideoxyadenosine in vivo. It is best absorbed on an empty stom high pH. The toxicity profile of didanosine is quite different from that of zidovudine. The most common toxicity is a pain

sensory peripheral neuropathy that occurs in ~30% of patients receiving >400 mg/d. 'It generally resolves with discon

of the drug and may not recur if the drug is resumed at a reduced dose. At higher doses than are currently used one m

pancreatitis in ~10% of patients. Pancreatitis associated with didanosine therapy can be fatal. Didanosine should be discontinued if a patient experiences abdominal pain consistent with pancreatitis or if an elevated serum amylase or lip is found in association with an edematous pancreas on ultrasound.

Zalcitabine (ddC; 2',3'-dideoxycytidine) is rarely used today in the management of patients with HIV infectionand was discontinued from the US market in 2006. Among the nucleoside analogues licensed for the treatment of HIV infection, probably the weakest. The main toxicities of ddC are peripheral neuropathy and pancreatitis. no % given

Stavudine (d4T; 2',3'-didehydro-3'-deoxythymidine) was the fourth drug licensed for the treatment of HIV infection an

discontinued from the US market in 2006. Like zidovudine, stavudine is a thymidine analogue. These two drugs are ant

in vitro and in vivo and should not be given together. Stavudine has been associated with a higher incidence of mitocho toxicity than the other licensed nucleoside analogues. Peripheral neuropathy, lipoatrophy, lactic acidosis, and hepatic s are the main toxicities of stavudine.

DIDANOSINE... The most serious toxicities associated with didanosine include peripheral neuropathy and pancreatitis, both of which ar

to be a consequence of mitochondrial toxicity. Up to 20% of patients reported peripheral neuropathy in early clinical tri

(Perry and Noble, 1999). As with other dideoxynucleosides, peripheral neuropathy is more common with higher doses o

concentrations of didanosine and is more prevalent in patients with underlying HIV-related neuropathy or in those rece other neurotoxic drugs. Typically, this is a symmetrical distal sensory neuropathy that begins in the feet and lower extr

but may involve the hands as it progresses (stocking/glove distribution). Patients complain of pain, numbness, and ting

the affected extremities. If the drug is stopped as soon as symptoms appear, the neuropathy will stabilize and should i

or resolve. Retinal changes and optic neuritis also have been reported with didanosine, and patients should undergo pe retinal examinations. ****STAVUDINE

The most common serious toxicity of stavudine is peripheral neuropathy. Neuropathy occurred in up to 71% of patients

monotherapy trials with a dose of 4 mg/kg per day. With the current recommended dose of 40 mg twice daily, the neu potent inhibitor of DNA polymerase-g than either didanosine or zalcitabine, suggesting that other mechanisms may be (Cui et al., 1997). As with other dideoxynucleosides, peripheral neuropathy is more common with higher doses or

incidence is about 12% (Hurst and Noble, 1999). Although this is thought to reflect mitochondrial toxicity, stavudine is

concentrations of stavudine and is more prevalent in patients with underlying HIV-related neuropathy or in those receiv

other neurotoxic drugs. Stavudine is also associated with a progressive motor neuropathy characterized by weakness a

some cases respiratory failure, similar to Guillain-Barre syndrome (HIV Neuromuscular Syndrome Study Group, 2004).

ZALCITABINE Zalcitabine toxicities are similar to those of the other dideoxynucleoside analogs didanosine and stavudine. common with preexisting HIV-associated neuropathy and advanced HIV disease. This is a symmetrical distal sensory

Severe peripheral neuropathy has been reported in up to 15% of patients. Peripheral neuropathy is dose-related and m

neuropathy that begins in the feet but may progress to a stocking/glove distribution. Other specific risk factors for neu include alcohol consumption, diabetes, and low vitamin B12 concentrations (Fichtenbaum et al., 1995). If the drug is s zalcitabine therapy and appears to be less frequent than with didanosine (Saravolatz et al., 1996).

as soon as symptoms appear, the neuropathy usually stabilizes and should improve or resolve. Pancreatitis occurs rare

One toxicity unique to zalcitabine is oral ulceration and stomatitis, suggesting that this drug may have toxicity in rapidl dividing mucosal cells. Ulcerations of the buccal mucosa, soft palate, tongue, or pharynx occur in up to 4% of patients

et al., 1997) but may resolve with continued therapy. An erythematous maculopapular rash is reported commonly duri first 14 days of therapy but generally is self-limited and mild. Other reported toxicities include cardiomyopathy, arthral myalgias, and elevated hepatic transaminases.

CONCLUSION------DIDANOSINE(20%)>ZALCITABINE(15%)>STAVUDINE(12%)

STAVUDINE IN VERY HIGH DOSE CAN PER NEUROPATHY UPTO 71% MAY B TATS Y SOME INSTITUTE HAS GIVEN THAT ANSWER BUT I FEEL IN AIIMS THIS WILL B THE SEQUENCE

Q155. non-invasive diarrhoea is caused by all, except ;; 1shigella;; 2typhoid;; 3bacillus cereus;; 4y.enterocolitica; ANSWER SHIGELLA ITS JUST A STRAIGHT FOWARD QUESTION AS IT CAUSE BLOODY DIARRHOEA,,OUGHT TO B INVASIVE..... Q156. PELLINGRI-STEIDA LESION IN KNEE JOINT SIGNIFIES;; 1patellofemoral pathology;; 2acute acl tear;; 3chronic acl tear;; 4chronic mcl injury

A Pellegrini-Stieda lesion refers to a post traumatic ossification in or near the medial collateral ligament near the margi medial femoral condyle. One presumed mechanism of injury is a Stieda fracture (avulsion injury of the medial collatera ligament at the medial femoral condyle). Etymology Q157. thimble pitting of nails is seen in ;; 1alopecia areata;; 2lichen planus;; 3pityriasis lubra pilaris;; 4pityryais rosea;; Q158. all of the following are branches of the basilar artery, except ;; 1labyrhinthine;; 2ant inf cerebellar;; 3post nf cerebellar;; 4sup cerebellar;;

Back to top vinay.


Guest Posted: Sat Jul 17, 2010 12:43 pm Post subject:

Q159. haemodialysis amyloid is deposited in ;; 1knee joint;; 2large intestine;;

3liver;; 4 tongue;

Nuffield Department of Pathology, University of Oxford John Radcliffe Hospital, Oxford *Nuffield Department of Orthopa Surgery, Nuffield Orthopaedic Centre Oxford Renal Unit, Churchill Hospital Oxford Accepted for publication 24 July 1990.

Deposition of 2-microglobulin amyloid in the joints of dialysis patients is common and begins early in the course of tre but its pathogenic significance in the production of dialysis arthropathy is uncertain.

The joints (hip, knee, shoulder, elbow, wrist, cervical and lumbar spine, sacroiliac joint) and systemic tissues of 19 aut patients who had undergone haemodialysis for between 6 and 231 months were examined histopathologically for the p of 2-microglobulin amyloid;

it was present in all joints examined, including those unassociated with radiological changes and those of patients who on haemodialysis alone for only 24 months. Osteoarticular 2-microglobulin amyloid deposits were also found in patien who had been haemodialysed for more than 13 years and consisted of sparse tiny deposits in blood vessel walls. Q160. charge syndrome is associated with ;; 1cystic fibrosis;; 2oesophageal atresia;; 3choanall atresia;; 4cleft palate;;

had been treated mainly by continuous ambulatory peritoneal dialysis. Systemic amyloid deposition was only seen in p

Although genetic testing positively identifies nearly 2/3 of the total number of children with CHARGE, diagnosis is still la

clinical. The acronym CHARGE was coined in 1981 to describe a cluster of features identified in a number of children. T following are the signs that were originally identified in children with this syndrome, but these features alone are no lon in official diagnosis. * C - Coloboma of the eye, central nervous system anomalies * H - Heart defects * A - Atresia of the choanae * R - Retardation of growth and/or development * G - Genital and/or urinary defects (Hypogonadism) * E - Ear anomalies and/or deafness Q161. the excimer laser operates in which of the following spectral wavelenghths;; 1infrared;; 2microwave;; 3near ultraviolet;; 4far ultraviolet;;

An excimer laser (sometimes, and more correctly, called an exciplex laser) is a form of ultraviolet laser which is commo in eye surgery and semiconductor manufacturing. The term excimer is short for 'excited dimer', while exciplex is short

'excited complex'. An excimer laser typically uses a combination of an inert gas (argon, krypton, or xenon) and a react

(fluorine or chlorine). Under the appropriate conditions of electrical stimulation, a pseudo-molecule called an excimer ( of noble gas halides, exciplex) is created, which can only exist in an energized state and can give rise to laser light in t ultraviolet range. The wavelength of an excimer laser depends on the molecules used, and is usually in the ultraviolet.

The high-power ultraviolet output of excimer lasers makes them useful for surgery (particularly eye surgery), for lithog

semiconductor manufacturing, and for dermatological treatment. Excimer laser light is typically absorbed within the firs

billionth of a meter (nanometer) of tissue. Q162. welcome dr palm;;pl put the q no, while you anser;; polyprenoid, which canot be synthesises by human body is ;; 1dolichol;; 2ubiquinone;; 3retinoids;; 4menaquinone;

Chain-like structures are also synthesized from isoprenoid units and include ubiquinone, dolichol and the fat-soluble vit

Ubiquinone is an electron carrier in the mitochondrial respiratory chain while dolichol is a long-chain alcohol which help

glycoprotein synthesis. V itamins A, D, E and K are absorbed in the intestine and transported in the blood in the lipopro complexes. Vitamin A activity is present in retinoids and carotenoids. They function as visual pigment and as regulators

tissue growth and development. Vit E is represented by a group of closely related, hydrophobic lipids called the tocophe

They are powerful antioxidants which prevent oxidative damage to membrane lipids. Vit K is a blood-clotting co-factor. Plant-derived isoprenoid compounds include rubber, camphor, the fat-soluble vitamins A, D, E, and K, and -carotene (provitamin A) pg 118 harper old

Originary from vitamin A or synthesized by human body, retinoids can modulate cell proliferation, differentiation, and a decreasing carcinogenesis, effects controlled by six different human nuclear retinoid receptors answer menoquinone Q163. which of the following is not part of a plan;; 1schedules;; 2policies;; 3programmes;; 4manpower; Q164. time take for any project is estimated by ;; 1work sampling;; 2input-output analysis;; 3network analysis;; 4 system analysis

Back to top vinay.


Guest Posted: Mon Jul 19, 2010 12:46 pm Post subject:

Q165. xemphysema aquosum s seen in ;; 1wet drowning;; 2dry drowning;; 3immersion syndrome;; 4sec drowning;; Q166. THURSTON-HOLLAND SIGN IS SEEN IN ;; 1OBLIQUE FRACTURE OF LOWER 1/3 OF HUMERUS;; 2REVERSE OBLIQUE FRACTURE OF INTERTROCHANTERIC FEMUR;; 3SALTER -HARRIS TYPE 2 FRACTURE;; 4CORONAL FRACTURE F FEMORAL CONDYLES;; Q167. what is the mechanism of action of levosimendon;; 1it is a potassium channel pener;;

2it inhibits sodium entry into the cell;; 3it is a calcium sensitiser;; 4it inhibits na-k atp pump;; answer 3

Levosimendan is a calcium sensitiser it increases the sensitivity of the heart to calcium, thus increasing cardiac contr

without a rise in intracellular calcium. Levosimendan exerts its positive inotropic effect by increasing calcium sensitivity myocytes by binding to cardiac troponin C in a calcium-dependent manner. It also has a vasodilatory effect, by opening

adenosine triphosphate (ATP)-sensitive potassium channels in vascular smooth muscle to cause smooth muscle relaxat

combined inotropic and vasodilatory actions result in an increased force of contraction, decreased preload and decrease

afterload. Moreover, by opening also the mitochondrial (ATP)-sensitive potassium channels in cardiomyocytes, the drug cardioprotective effect. Q168. maculae cerulae are caused by ;; 1pubic lice;; 2head lice;; 3body lice;; 4mites;; pubic lice

small gray-blue macules found on the chest, abdomen, pages , or upper arms in pediculosis pubis, which are especially noticeable in light-skinned individuals. Q169. the type of filter, which has silver catalyst is ;; 1chamberland filter;; 2candle filter;; 3berkefield;; 4katadyn filter;; answer 4

In the Katadyn filter, the surface of the filter is coated with a silver catalyst so that bacteria coming in contact with the are killed by the 'oligodynamic' action of the silver ions, which are liberated into the water.

silver is unique in its behavior with oxygen. For example,when pure silver is melted in ambient air it absorbs about ten volume, or 0.3% of its weight of oxygen. On cooling to a few degrees above solidification, it abruptly releases most of oxygen in a dramatic phenomenon which the industry calls a spit." Q170. which of the following color code is not used for classifying the patient under the imnci strategy;; 1pink;; 2 red;; 3green;; 4yellow;; classifications are colour coded: pink suggests hospital referral or admission, yellow indicates initiation of treatment, and green calls for home treatment. imnci

During the mid-1990s, the World Health Organization (WHO), in collaboration with UNICEF and many other agencies, institutions and individuals, responded to this challenge by developing a strategy known as the Integrated Managemen

Childhood Illness (IMCI). Although the major reason for developing the IMCI strategy stemmed from the needs of cura

the strategy also addresses aspects of nutrition, immunization, and other important elements of disease prevention an

promotion. The objectives of the strategy are to reduce death and the frequency and severity of illness and disability, a contribute to improved growth and development. This strategy has been adapted for India as Integrated Management Neonatal and Childhood Illness (IMNCI).

The IMNCI clinical guidelines target children less than 5 years old the age group that bears the highest burden of de supports the rational, effective and affordable use of drugs and diagnostic tools An evidence-based syndromic approach can be used to determine the: Health problem(s) the child may have; Severity of the childs condition; Actions that can be taken to care for the child (e.g. refer the child immediately, manage with available resources, or at home). In addition, IMNCI promotes: Adjustment of interventions to the capacity and functions of the health system; and Active involvement of family members and the community in the health care process. Q171. association is best implicated by ;; 1case control study;; 2prospective study;; 3cross sectional study;; 4experimental epidemiology;; Q172. partial lipodystrophy is seen with ;; 1MGN;; 2MPGN;; 3FSGS;; 4MCD;;

from common childhood diseases. The guidelines take an evidence-based, syndromic approach to case management th

Q173. ALL OF THE FOLLOWING DRUGS ARE RECOMMENDED IN LIFE threatening fungal infections in neutropaenic pts , receiving chemotherapy, except ;; 1voricanazole;; 2caspofungin;; 3amphotericin;; 4itraconazole;; ANSWER ITRACONAZOLE

FOR CASPOFUNGINS In mainly HIV-positive patients with oesophageal and oropharyngeal candidiasis, the new antifun

caspofungin in a dosage of 35 or 50 mg/day showed superior efficacy compared with low-dose D-AmB (0.5 mg/kg/day

Remarkably, this compound, representing the first clinically studied agent from a new class of antifungals inhibiting the

synthesis of -(1,3)-d-glucan in the fungal cell wall, was extremely well tolerated. A dose-limiting toxicity has not yet b defined. Beyond that, resistance to this and other echinocandin antifungals has so far been experimentally induced in C species only at extremely low rates in vitro.12 Caspofungin has also shown significant efficacy as salvage antifungal in patients with documented invasive aspergillosis refractory or intolerant to conventional antifungals.13 Based upon the

documented response to caspofungin in 41% of these patients, the drug was approved in early 2001 by the Food and D Administration (FDA) for the treatment of patients with aspergillosis refractory or intolerant to other licensed antifunga Itraconazole, the only triazole presently licensed for the treatment of aspergillosis, has recently become available for

intravenous administration. In neutropenic patients with haematological malignancies and refractory fever, intravenous itraconazole has shown at least equivalent efficacy (47% versus 38%) to D-AmB. Not surprisingly, it was significantly b

tolerated (withdrawal due to toxicity 19% versus 38%) and showed less nephrotoxicity than D-AmB. Beyond that, abou

third of patients given intravenous itraconazole could be switched to the oral formulation after a median of 9 days of tr

As with voriconazole, an important potential for interactions with a large number of other drugs must be considered for itraconazole. Q174. which one is a robovirus;; 1buniya virus;; 2phlebovirus;; 3nairovirus;; 4hantavirus;; Q175. WHICH OF THE FOLLOWING ARE NOT CONTINUOUS CELL LINES;; 1hela;; 2hep-2'' 3mc coy;; 4wl-38

Back to top vinay.


Guest Posted: Tue Jul 20, 2010 12:48 pm Post subject:

Q176. this disease is not associated with endogenous intracellular accumulations;; 1steatosis;; 2ochronosis;; 3anthracosis;; 4melanoma;; ans anthracosis? it is an exogenous pigment deposition disease...

Black lung disease, also known as Coal workers' pneumoconiosis (CWP), is caused by long exposure to coal dust. It is a common affliction of coal miners and others who work with coal, similar to both silicosis from inhaling silica dust, and t body; that leads to inflammation, fibrosis, and in the worst case, necrosis.

long-term effects of tobacco smoking. Inhaled coal dust progressively builds up in the lungs and is unable to be remove

Coal workers' pneumoconiosis, in its most severe state, develops after the initial, milder form of the disease known as anthracosis (anthrac - coal, carbon). This is often asymptomatic and is found to at least some extent in all urban dwelle due to air pollution. Prolonged exposure to large amounts of coal dust can result in more serious forms of the disease, coal workers' pneumoconiosis and complicated coal workers' pneumoconiosis (or Progressive massive fibrosis,) All the other options are causes of endogenous pigment depositions... steatosis-lipid deposit ochranosis-HOMOGENTISIC ACID OXI DEFI in children with homogentisic acid deposition in joints cartilages melanosis-inc melanin deposi in skin eyes etc... Q178. in leucocyte adhesion def type 1 -the defect lies in the synthesis of ;; 1alpha interferon;; 2beta 2 microglobulin;; 3beta2 integrins;; 4 leukotrienes;; ans B2 integrins...

Leukocyte adhesion deficiency type I (LAD I) is a failure to express CD18, which composes the common 2 subunit of L family (2 integrins). CD11a/CD18 (LFA-1) expressed on lymphocytes is known to play an important role in lymphocyt

trafficking (adhesion to vascular endothelium), as well as interactions to antigen presenting cells (APC). LFA-1 also play

of cytotoxic killing by T cells. Another member of this family is CD11bCD18 (MacA or CR3) and CD11cCD18(CR4). Thes

members mediate leukocyte adhesions to endothelial cells but they also serve as receptors for iC3b (inactivated C3b). patients succumb to life-threatening infection, usually within 2 years of life in severe cases of leukocyte adhesion defici survive to adulthood.

(<1% expression of CD18). In milder forms of leukocyte adhesion deficiency I (1-30% expression of CD8), patients ma

Leukocyte adhesion deficiency type II is extremely rare; only a handful cases have been reported and most of them ar

Middle Eastern decent. It is a defect in the expression of ligands for selectins due to lack of enzymes required for expre

selectin ligands. Patients have leukocytosis, recurrent infections (more prominent in infants and toddlers), and severe g and mental retardation. This disease is a defect in fucose metabolism (lack of fucosylation of the carbohydrate selectin cells. The patients are unable to fucosylate other glycoproteins, including the H blood group polysaccharide.

that results in failure to express the ligand for E and P selectin, sialyl Lewis-X (CD15s) expressed on leukocytes and en

Patients with leukocyte adhesion deficiency II manifest the Bombay phenotype (ie, negative for O and H blood group a

with potential production of anti-H antibody). The immunoglobulin M (IgM) and immunoglobulin G (IgG) heavy chains a deficiency II. Q179. PRINGLE MANOVEUR REFERS TO ;; 1clamo over ivc;; 2clamp over hepatic vein;; 3clamp across of foramen of winslow;; 4clamp across splenic artery ans-clamp across of foramen of winslow ref. BAILEY 25TH,SCHWARTZ'S 9TH

not fucosylated. However, IgM and IgG serum levels are within the reference range in patients with leukocyte adhesion

The Pringle manoeuvre is a surgical manoeuvre used in for controlling traumatic liver injury n elective hepatic resection is accomplished by tourniquet or vascular clamp through the foramen of Winslow. Q180. which one of the following activities is simultaneously stimulated by epinephrine in muscle and and inhibited by epinephrine in the liver;; 1fatty acid oxidation;; 2glycogenolysis;; 3cyclic amp synthesis;; 4glycolysis;; Q181. all the following are risk factors for thromboembolic events in the perioperative period, except ;; 1morbid obesity;; 2adenocarcinoma;; 3nephrotic syndrome;; 4 chronic renal failure;

minimize blood loss during surgery.Pringle maneuver (occlu- sion of hepatic artery, portal vein, and bile duct)ie hepatic

Back to top vinay.


Guest Posted: Wed Jul 21, 2010 12:46 pm Post subject:

Q183. anencephaly is the result of a defect in which of the following;; 1closure of the caudal neuropore;; 2closure of the rostral neuropore;; 3formation of the first bbranchial arch;; 4formation of the somites;; Q184. CASTELLANIS PAINT contains all of the following except ;; 1basic fuchsin;; 2phenol;; 3benzene hexachloride;; 4boric acid; ans-benzene hexachloride ref.internet

CASTELLANI'S PAINT Basic fuchsin 0.3 Ethyl alcohol 95% 10.0 Boric acid 1.0 Phenol liquef. 4.0 Acetone 5.0 Resorcinol 10.0 Water to 100.0

Castellani's paint was perfected in 1905 by Aldo Castellani (1878-1971), an Italian physician and a specialist in tropical

diseases. Castellani's paint is an excellent preparation for tinea cruris and moniliasis of intertriginous areas.It is effectiv

pustular dermatoses of the hands and feet, and has also been recommended for pruritus ani and pruritus vulvae. Color Castellani's paint may be used to reduce secondary bacterial contamination in onycholysis and in chronic paronychia. Q185. LAURINS/ MERCHANTS VIEW is the x-ray view of ;; 1talus;; 2spine in a patient with scoliosis;; 3hip joint;; 4patella;; answer 4 patella Merchant Technique mainly for subluxation - Discussion: - for evaluation of both the patella and trochlea surfaces of femur; - disadvantages: - this view does not necessarily predict which patients will show functional improvements w/ surgical realignment; Laurin technique: - lateral patellofemoral angle is index of tilt but not of subluxation; - patellar alignment assessed using lateral patellofemoral angle on axial views, made w/ knee in 20 deg of flexion; - angle formed by lateral patellar facet & line drawn across most prominent aspects of anterior portion of femoral trochlea should be open laterally in normal patellofemoral joint; other views Stress Axilla View: - in some cases, there will be impressive differences between static and dynamic axilla knee views; - made with the knee flexed 35 deg off the end of the x-ray table; - a constant lateral pressure is exerted on to the patella in an attempt to displace the patella laterally; - comparisons should be made between the symptomatic and asymptomatic knees; Sunrise View: - used to image a tangential view of the patella;

- the patient is prone with the knee flexed 115 deg; - central beam is directed toward the patella with 15 deg cephalic tilt;

Q186. which structure most effectively prevents toxic molecules from penetrating an epithelium by passing between ad epithelial cells;; 1desmosome;; 2gap iunction;; 3 hemidesmosome;; 4tight junction;; ans tight junction..?

An important property of epithelial and endothelial cells is their assembly into a physical and ion- and size-selective bar

separating tissues. Intercellular junctions, such as adherens and tight junctions, play a crucial role in the formation and

maintenance of epithelial and endothelial barriers. Adherens and tight junctions were first identified on the ultrastructu

as part of the terminal bar, a tripartite junctional complex bordering the apico-basolateral membrane in a variety of po simple epithelia and implicated in barrier function . Desmosomes form the third structure of this complex.. Tight junctio intercellular membrane space of tight junctions is almost completely obliterated, hence their alternative name zonulae

the most apical structure of the apical complex demarcating the border between apical and basolateral membrane dom

occludens. Adherens junctions are positioned immediately below tight junctions and characterized by two apposing me which are separated by 20 nm, that run parallel over a distance of 0.20.5 m. Both adherens and tight junctions are cl associated with a circumferential belt of actin. molecular composition (reviewed in Anderson et al., 2004). They also restrict the basolateral n memb components, so "fence functions".... HEMIdeSMOSOMES connect cell to ECM.. GAP junc - free mobility of moleculecus/ions cross cell- functional syncitium.. DESMOSOMES ...??they too are intrecellular transmemb connections but prvide structural support mainly by attaching themselves to intermediate filaments in cytoplasm..AB against them l/t bullous skin disorders.....? Q187. the systematic distorsion of retrospective studies that can be eliminated by a prospective design is ;; 1confounding;; 2effect modification;; 3recall bias;; 4measurement bias;; ans recall bias or memory bias.. since in cohort studies we're proceeding from cause to effect there's no need for cohorts to recall events of past... Q188. LEVI-LORAIN DWARFISM IS DUE TO A LACK OF WHICH HORMONE;; 1SOMATOSTATIN;; 2SOMATOTROPIN;; 3 SOMATOMEDINC;; 4TRANSFORMING GROWTH FACTOR -ALPHA;; answer somatomedin c

Tight junctions do provide epithelia with a semipermeable size- and ion-specific barrier, which varies depending on the

Dwarfism can also result from a lack of somatomedin C (also called insulin like growth factor, IGF-1) production. Soma C is a hormone produced in the liver that increases bone growth when growth hormone is present. The African pygmy proportionate little person. Q189. THE VISUAL FIELD DEFECT SEEN IN VIT B 12 DEFICIENCY;; 1arcuate;;

Levi-Lorain dwarfs lack the ability to produce somatomedin C in response to growth hormone. All causes of dwarfism le

2 paracentral scotoma;; 3nasal step;; 4centrocecal scotoma

Back to top vinay.


Guest Posted: Thu Jul 22, 2010 1:21 pm Post subject:

Q190. all the following uraemic manifestations improve with dialysis, except ;; 1metabolic acidosis;; 2osteodystrophy;; 3asterixis;; 4bleeding; ANS-OSTEODYSTROPHY REF.HARRISON 17,CMDT 10 READ TABLE 274-3 METABOLIC ACIDOSIS N ASTERIXIS IMPROVES WITH DIALYSIS

BLEEDING TIME IMPROVES WITH DIALYSIS ALTHOUGH IT DOESNT BECOME NORMAL-REQUIRES CRYO PPT TRANSFUS

RENAL OSTEODYSTROPHY CAN IMPROVE OR WORSEN WITH DIALYSIS..OFTEN REQUIRES OTHER MEASURES LIKE VIT SUPPL. ,PARATHYROIDECTOMY OR RENAL TRANSPLANT. Q191. which culture method is used for the preparation of bacterial antigens and vaccines;; 1lawn culture;; 2stroke culture;; 3pour plate culture;; 4liquid cultyre;; Q192. which area of the mandible is most commonly fractured;; 1 angle;; 2condyle;; 3molar ;; 4 mental; Q193. during lumbar synpathectomy-open - all can be confused for the lumbar sypathetic chain, except ;; 1genitofemoral nerve;; 2iliohypogastric nerve;; 3lymph nodes and lymphatics;; 4tendinous strip of psoas minor;;

ANS-ILIOHYPOGASTRIC NERVE REF-DIRECT PICK FROM BAILEY 24 PG 953 NOT GIVEN IN 25TH EDITION AS SURGERY BECOME OBSELETE.

''care should b taken not to mistake genitofemoral nerve,small lymph nodes and lymphatics or occasional tendinous str psoas minorfor sympathetic chain"

Lumbar sympathectomy (LS) has been used in the treatment of various vascular and neurological disorders of lower ex

for close to 80 years. LS is acknowledged to have a role in the treatment of patients with reflex symptomatic dystrophy

(causalgia), vasospastic disorders like acrocyanosis and Raynaud's syndrome, hyperhydrosis of the feet, symptomatic vasospasm, and nonreconstructable arterial occlusive disease. A literature survey finds its miscellaneous uses for frostb desiccation of chronically moist ulcerations between the toes, chronic renal pain, rectal tenesmus, and sympathetically

maintained intractable pain due to malignant reasons. Although it is the most commonly performed operation in the de

countries for peripheral ischemic diseases (PID), there is considerable controversy about its usefulness Q194. perforin molecules are found in ;; 1primary granules of neutrophils;; 2sec granules of neutrophils;; 3cytotoxic t cells;; 4macrophages;; ANS-3)CYTOTOXIC T CELLS REF-ROBBINS 8TH PG 31 -GIVEN IN MECH OF APOPTOSIS VERY IMP TOPIC PERFORINS -A TRANSMEMBRANE PORE FORMING MOLECULE WHICH PROMOTES ENTRY OF GRANZYMES Q195. MEYERS -KOUVENAAR bodies are found in ;; 1filariasis;; 2falciparum malaria;; 3schistosomiasis;; 4l epromatous leprosy;; ANS-FILARIASIS REF ROBBINS 8TH PG 395

dead microfilariae surrounded by stellate, hyaline, eosinophilic precipitates embedded in small epithelioid granulomas ( Kouvenaar bodies)

Back to top Guest


Posted: Fri Jul 23, 2010 12:50 pm Post subject:

A study was made of the Meyers-Kouwenaar (MK) body in the livers of experimentally infected ferrets. Meyers-Kouwen bodies, the carcasses of microfilariae (mff) covered by deposits of Splendore-Hoeppli (SH) material, were found in sma

abscesses of eosinophils and in granulomas. The SH deposits varied from an eosinophilic, hyaline fringe around intact m

multilayered deposits surrounding an unrecognizable granular remnant. In abscesses, peroxidase activity was intense i

deposits and the surrounding eosinophils. The presence and localization of IgG were variable in MK bodies, as detected enzyme-linked immunohistologic assay; and antigens of mff were not detected in the SH deposits. Electron microscopy MK body demonstrated a layered, radial deposition of amorphous and granular material on the mff and a structural

heterogeneity which apparently included leukocyte granules and other cell organelles. Leukocytes surrounding MK bodi

abscesses were often degranulated and degenerate; incorporation of lysosomes of eosinophils and cellular debris into t deposits at the periphery of the MK bodies was indicated.

Back to top vinay.


Guest Posted: Fri Jul 23, 2010 12:53 pm Post subject:

Q196. all are actions of endothelin 1 except ;; 1bronchodilatation;; 2vasoconstriction;; 3devreased gfr;; 4vasodilatation;;

Endothelial cells also produce endothelin-1, one of the most potent vasoconstrictor agents yet isolated. Endothelin-1 (E endothelin-2 (ET-2), and endothelin-3 (ET-3) are the members of a family of three similar 21-amino-acid polypeptides

this ques is lil tricky in sense ,vasoconstriction is well known feature of endothelin 1..so we tend to think vasodil will no but vasodil is due to increased NO..

decreased gfr too seen it becoz contraction of mesangial cell n capillaries... answer here is bronchodilation as wth endothelin 1 broncho constriction which is commonly seen in asthma..

Q197. splenenunculi can be found in all of the following areas, except ;; 1splenic hilum;; 2post abdominal wall;; 3tail of pancreas;; 4left spermatic cord;; Splenunculi are small nodules of spleen that are detached from the rest of the organ. They are common, seen in up to need to distinguish them from more sinister pathology.

CTs of the abdomen and up to 30% of autopsies. They are benign and asymptomatic, their importance mainly related t

Splenunculi are typically a few centimetres in diameter when identified, well circumscribed rounded or ovoid nodules. A most are located near the spleen, they have been identified elsewhere in the abdominal cavity including 2,3. near the spleen (most common) 1.gastrosplenic ligament 2.splenorenal ligament 3.pancreatic tail 4.greater omentum 5.small bowel mesentery 6.stomach or bowel wall 7.scrotum CT They have density and enhancing characteristics similar to the rest of the spleen. Small (<1cm) may appear hypodens spleen, most likely as an artefact of partial voluming 2.

The converse of that is that if a mass is seen at the splenic hilum, which does not enhance in the same fashion as the a spleen, then alternative diagnoses should be entertained. Scintigraphy Tc 99m sulphur colloid scan long as the splenunculus is at [b]least 2cm/b] in diameter. splenunculi adherent to anterior abdominal wall not posterior apart from that its also seen in spermatic cord n scrotum answer here choice 2 post abd wall Q198. which one is the least commn type of high arch foot;; 1calaneocavus;; 2cavovarus;; 3plantaris;; 4calaeaneus; CAVOVARUS Neuromuscular imbalance due to weak intrinsics

In rare instances the diagnosis can be confirmed with Tc99m sulphur colloid scan which will demontrate increased upta

Conditions associated with cavus or cavo-varus feet : Central nervous system Brain Cerebral palsy Friedreich's Ataxia Spinal Cord Spinal cord tumour Spinal dysraphism Tethered cord Spinal bifida Diastematomyelia Poliomyelitis Spinal muscular atrophy Peripheral Nervous system Hereditary peripheral neuropathies, e.g. CMT Traumatic peripheral nerve lesions Muscle Disease Duchenne's muscular dystrophy Post-ischaemic (Volkmans contracture) Common component of residual club feet Idiopathic Clinically

Often complains of fatigue and discomfort in the foot or ankle with the severity of symptoms paralleling the degree and of the deformity The presenting abnormality may be representing the early signs of a progressive neurologic degenerative disease The idiopathic variety usually noticed at about 10 years of age and it is usually bilateral Ankle usually has reduced dorsiflexion Hindfoot : There is inversion of the heel (calcaneo varus) and inversion of the subtalar joint Forefoot is plantar flexed (plantaris deformity) with pronation of the forefoot in relation to the hind foot MTP joints are hyperextended Interphalangeal joints are flexed becomes fixed Pressure is taken over a small area under the metatarsal heads, causing pain and callosities Callosities also develop over interphalangeal joints due to pressure from shoe-wear Coleman's block test

Weight bearing is via a tripod of the heel, the first and fifth metatarsal heads and associated with hind foot varus which

Establishes whether the hind foot is fixed or flexible and demonstrates the type of surgical correction required Place the foot obliquely on a 1" - 1.5" block so that the first metatarsal hangs free to allow correction of the hind foot d weight bearing If the hind foot returns to neutral or valgus the deformity is flexible Treatment Is based on the age of the patient and the flexibility of the deformity Tendon lengthenings and transfers are used for flexible feet Bony procedures added for fixed deformities Young children Strengthening exercises and stretching Ensure footwear has enough room for the toes Operative

Flexor to extensor transfers and interphalangeal joint arthrodesis will straighten the toes, if there is much cavus

Steindlers and short flexor / abductor hallucis release may also be needed to release plantar fascia, followed by a BK P

the foot held in valgus and supination for 6 weeks with frequent changes of POP to correct deformity (children aged 4 years) Transfer of Tibialis Posterior to the calcaneum may result in correction of calcaneus deformity Adolescents

If toe deformities become fixed, arthrodese all the PIP joints; so the long flexors no longer bunch the toes up and the l extensors are reinserted into the metatarsal necks to elevate the forefoot

If hind foot varus is fixed in Coleman's test and the first ray deformity is severe, calcaneal osteotomy may be indicated Dwyer type medial opening wedge or modified Dwyer with a lateral closing wedge osteotomy of the calcaneum For children aged 10 or older

Can be combined with Steindler operation Wedge excision of the tarso-metatarsal joints may be needed for severe deformity in the absence of neurological abnor Done after maturity Adults Increased arch support and contact area in appropriate fitting shoes is the best treatment Severe painful deformities may require operation

CALCANEOCAVUS Due to major muscle imbalance, usually weak or absent gastrocnemius/soleus Usually tibialis anterior, tibialis posterior and peronei are preserved No conservative treatment Progressive Clinically Complains of shoe fitting problems Pain over prominences Prominent heel Upward pitch of calcaneus High arch (pistol grip deformity) Treatment Before 5 years old No treatment in genera,l as no bony deformity is usually present Good muscle testing in this age group is difficult Patient is not likely to be disabled by the problem 5 - 12 years old Stabilise subtalar joint by extra-articular arthrodesis Management of bony deformity of calcaneus and ankle Release of plantar soft tissue contractures with or without calcaneal posterior displacement osteotomy Tendon transfers to improve plantar flexion of os calcis

Older than 12 years old These patients have fixed deformity Triple arthrodese to stabilise and correct hindfoot +/- tendon transfer to improve plantar flexion wheelers ortho says - cavus deformity of the foot (elevated longitudinal arch) due to fixed plantar flexion of the forefoot; - main type is the cavovarus and less frequent is the cavovarus; - associated with spinal cerebellar degenerative dz;

Q199. the nerve fibres mostresistant to mechanical trauma ;; 1motor;; 2sensory;; 3sympathetic;; 4parasympathetic;; Q200. lipedema is not associated with ;; 1woman predominance;; 2no pitting;; 3involvement of trunk;; 4stammers sign;;

Lipedema, which is also known as painful fat syndrome, is abnormal symmetrical swelling due to accumulations of fat a feet. There are also fatty bulges on the outer surfaces of the pages . [1]

located in the tissues just under the skin of the hips and legs. It extends downward toward the ankles but does not affe

This condition is inherited, occurs almost exclusively in women, and most cases gradually develop during puberty. Lipedema involves the extra deposit and expansion of fat cells in a distinct pattern on the body from just below the wa above the ankles. Unlike the fat associated with obesity, the fat associated with lipedema cannot be exercised away. Also, this fat tissue does not respond or reduce with diet. The Symptoms of Lipedema Pain is present particularly along the shin. This is a major problem . In the early stages of lipedema the upper part of the body may be slender. Despite having a slim upper body, fat accumulates from the tops of the hips to the ankles.

When weight is gained, it accumulates in the areas of the hips and legs. When weight is lost, the fat decrease occurs in areas other than those affected by lipedema. Weight loss does not occu area between the waist and ankles. Fat that extends down the legs creates a ring of fatty tissue that overlaps the tops of the feet. Swelling develops in the legs during the second half of the day; however, this swelling decreases during sleep. In the early stages, nodules develop. Nodules are small fatty lymps within the tissues. In the later stages, lobules develop. Lobules are rounded fat deposits that aer largr than nodules. diff b/w lipedema n lymphedema 1.Lipedema is symmetrical because the excess fat involves both legs equally. Lymphedema is not symmetrical and often occurs in only one leg. 2.Lipedema swelling has a characteristic ring at the base of the ankle where the swelling stops. Lymphedema swelling often begins at the foot and there is not ring at the base of the ankle.

3.Lipedema does not include swelling of the foot. Lymphedema swelling affects the foot, ankle, and leg. 4.Lipedema patients often complain of pain when touched. Lymphedema pain is rarely this severe. 5.Lipedema often causes bruising and subcutaneous bleeding. Lymphedema symptoms do not include bruising and subcutaneous bleeding. 6.Lipedema does not have pitting edema as a symptom. See How Lymphedema is Diagnosed Lymphedema does have pitting edea as a diagnostic indicator. 7.Lipedema does not have Stemmers sign as a symptom. Lymphedema does have Stemmers sign as a diagnostic indicator. answer choce d stemmer sign

Back to top vinay.


Guest Posted: Sat Jul 24, 2010 12:47 pm Post subject:

Q202. in a neuron, graded electrogenesis occurs at ;; 1dendritic zone;; 2initial segment;; 3axon;; 4nerve ending Q203. middle cardiac vein opening into coronary sinus is related to ;; 11 ant interventricular groove;; 2inferior interventricular groove;; 3lower border of the heart;; 4post atrioventricular groove ans-posterior(inferior) interventricular septum REF.GRAYS 40,BDC VIMP TOPIC BLOOD SUPPLY OF HEART Q204. BLOODLESS FOLD OF TREVES IS RELATED TO ;; 1meckles diverticulum;; 2ischiorectal fossae;; 3dononvilliers fascia;; 4iliocaecal fold ans-ileocaecal fold ref -grays 40,BDC

Cecal Foss (pericecal folds or foss).There are three principal pouches or recesses in the neighborhood of the cecu The superior ileocecal fossa is formed by a fold of peritoneum, arching over the branch of the ileocolic artery which sup

ileocolic junction. The fossa is a narrow chink situated between the mesentery of the small intestine, the ileum, and the

portion of the cecum behind. (b) The inferior ileocecal fossa is situated behind the angle of junction of the ileum and ce

is formed by the ileocecal fold of peritoneum (bloodless fold of Treves), the upper border of which is fixed to the ileum, its mesenteric attachment, while the lower border, passing over the ileocecal junction, joins the mesenteriole of the ve

process, and sometimes the process itself. Between this fold and the mesenteriole of the vermiform process is the infer

ileocecal fossa. It is bounded above by the posterior surface of the ileum and the mesentery; in front and below by the

fold, and behind by the upper part of the mesenteriole of the vermiform process. (c) The cecal fossa is situated immed behind the cecum, which has to be raised to bring it into view. It varies much in size and extent. In some cases it is su

large to admit the index finger, and extends upward behind the ascending colon in the direction of the kidney; in other

merely a shallow depression. It is bounded on the right by the cecal fold, which is attached by one edge to the abdomin

from the lower border of the kidney to the iliac

Q205. the most widespread rickettsial infection is ;; 1epidemic typhus;; 2indian tick typhus;; 3murine typhus;; 4scrub typhus;; ans-scrub typhus ref-parks 20 th-imp topic in psm

ALTHOUGH EPIDEMIC TYPHUS DISTRIBUTED THROUGH OUT THE WORLD PREVALENCE HAS DRASTICALLY REDUCED.. SCRUB TYPHUS MC RICKETTSIAL DISEASE-PARKS

Tsutsugamushi disease (ie, scrub typhus): Cases are usually seen in rural south and southeast Asia, limited to the geog area bound by Japan, the Solomon Islands, and Pakistan. It is estimated that 1 million cases occur each year.

Rickettsialpox: This may be more prevalent worldwide than is reported. It has been identified in large cities in Russia, S Africa, and Korea. mc in india is scrub typhus {bhatia micro prof} Q206. human live vaccine is available against which species of brucella;; 1b.melitensis;; 2b.abortus;; 3b.suis;; 4b canis ;; ans-b.abortus REF.HARRISONS PG 976,ANANTH 7TH

In many developing nations, immunizations derived from the S19 vaccine have been evaluated in humans. In the form

Union, the administration of live S19 preparations were immunogenic, and protection was achieved and considered to l year but caused a modest but notable incidence of clinical cases, as well as a hypersensitivity reaction. As such, S19 is safe vaccine candidate for human use Q207. anaesthetic agent with least effect on myocardial contractility is ;; 1halothane;; 2trilene;; 3isoflurane;; 4ether;; Q208. HEERFORDTS DISEAQSE consists of all, except ;; 1facial paralysis;; 2parotid lymphoma;; 3iridocyclitis;; 4 fever;; ans-parotid lymphoma ref- harr 16 not given in 17 A peculiar course of sarcoidosis with enlargement of the parotid glands, mild fever, and uveitis, and facial nerve palsy.

may be associated hyperalgesia, papilloedema, meningism and pleocytosis on the cerebrospinal fluid due to sarcoidosis

clinical features may include hilar adenopathy, generalized lymphadenopathy, visceral lesion and skin rash. Young adul usually affected, males more often than females Also known as:

Heerfordt-Mylius syndrome Heerfordt-Waldenstrm syndrome Waldenstrms uveoparotitis Q209. the drug of chice for POSNER-SCHLAUSUMANN SYNDROME IS ;; 1TIMOLOL;; 2TOPICAL STEROIDS;; 3PILOCARPINE;; 4BRIMONIDINE; ANS-TOPICAL STEROIDS REF KANSKI 6TH

POSNER-Schlossman Syndrome (Glaucomatocyclitic crisis) is a condition with self-limited recurrent episodes of marked inflammatory glaucoma Pathophysiology

elevated intraocular pressure (IOP) with mild idiopathic anterior chamber inflammation. It is most often classified as se

Episodic changes in the trabecular meshwork lead to impairment of outflow facility and result in an elevation of IOP. Th

changes are accompanied by mild intraocular inflammation. In the acute phase of PSS, optic nerve head parameters an flow rates were altered; however, all returned to normal without any permanent damage after resolution of the elevate Electroretinogram studies in the acute phase demonstrate a selective reduction in the S-cone b-wave.

Associations with immunogenetic factors also exist; in one study, the presence of human leukocyte antigen Bw54 (HLA was found in 41% of patients # Treatment recommendations include the following: * Topical steroids - Prednisolone acetate 1% 1 gtt qid, followed by taper blockers should be avoided in patients with asthma.) * Systemic carbonic anhydrase inhibitors - Acetazolamide 250 mg PO qid * Topical NSAIDs - Diclofenac 0.1% 1 gtt tid/qid or equivalent * Oral NSAIDs - Indomethacin 75-150 mg/d PO

* Topical antiglaucoma drops - Timolol 0.25-0.5% 1 gtt bid or equivalent, or dorzolamide 2% 1 gtt bid/tid or equivalen

# Miotics and mydriatic agents seldom are used because they may have further deleterious effects on the blood-aqueo barrier, and long-acting periocular steroids are frowned upon because of lingering IOP effects.

# NSAIDs reduce the inflammatory component by inhibiting the production of prostaglandins, and antiglaucoma medic elevations caused by steroids in steroid-responsive patients. Q210. the main action of trabeculectomy is ;; 1external drainage of aqueous;; 2hyposecretion of aqueous;; 3cyclodialysis;; 4increased uveoscleral outflow;;

reduce the influx of new aqueous; both these effects rapidly control the IOP. This combination also avoids potential IOP

During a trabeculectomy, the patient's eye is held open with a speculum. The outer layer, or conjunctiva, and the white eye, or sclera, are cut open (A). A superficial scleral flap is created and a plug of sclera and underlying trabecular netw removed (B). This allows the fluid in the eye to circulate, relieving pressure. The scleral flap is closed and sutured (C). conjunctiva is closed (D)

Intraocular pressure may be lowered by allowing drainage of aqueous humor from within the eye to the following route

filtration through the sclerostomy around the margins of the scleral flap into the filtering bleb that forms underneath th

conjunctiva, (2) filtration through outlet channels in the scleral flap to underneath the conjunctiva, (3) filtration throug

connective tissue of the scleral flap to underneath the conjunctiva. into cut ends of Schlemm's canal, (4) aqueous flow and the sclera if tissue is dissected posterior to the scleral spur

ends of Schlemm's canal into collector channels and episcleral veins and (5) into a cyclodialysis cleft between the ciliar

Back to top kollin.


Guest Posted: Mon Jul 26, 2010 12:05 pm Post subject:

Q211. CHASSAIGNAC TUBERCLE IS IS A LAND MARK FOR ;; 1stellate ganglion block;; 2trigeminal ganglion block;; 3celiac plexus block;; 4brachial plexus block;;

The Chassaignac tubercle is used as a landmark by anesthesiologists for local anesthesia of the brachial plexus and the plexus using the supraclavicular approach.

it is the name given to the anterior tubercle of the transverse process of the sixth cervical vertebra against which the c artery may be compressed by the finger.

The patient is placed in the supine position with the neck slightly extended, the head rotated slightly to the side opposi

block, and the jaw open. The point of needle puncture is located between the trachea and the carotid sheath at the lev

cricoid cartilage and Chassaignac's tubercle. Although the ganglion lies at the level of the C7 vertebral body, the needle

inserted at the level of C6 to avoid the piercing the pleura. Cutaneous anaesthesia is obtained with a skin wheal of loca anaesthetic.

The sternocleidomastoid and carotid artery are retracted laterally as the index and middle fingers palpate Chassaignac'

tubercle. The skin and subcutaneous tissue are pressed firmly onto the tubercle to reduce the distance between the ski and bone, and in an attempt to push the dome of the lung out of the path of the needle. When properly performed, thi manoeuvre is uncomfortable for the patient.

The needle is directed onto the tubercle, and then redirected medially and inferiorly toward the body of C6. After the b contacted, the needle is withdrawn 1-2 mm. This brings the needle out of the belly of the longus colli muscle, which sit immobile. Needle position is confirmed by fluoroscopy. Spread of radiocontrast is confirmed by both anteroposterior and lateral v colli muscle. Immediate dissipation of the solution indicates intravascular placement of the needle orifice.

posterior to the ganglion and runs along the anterolateral surface of the cervical vertebral bodies. The needle is then h

Failure of the solution to spread cephalad and caudad between tissue planes suggests intramuscular injection into the l

A 10 ml control syringe charged with local anaesthetic is attached to the needle and aspiration is performed to rule out usefulness of this test dose to provide early warning of intra-arterial injection is questionable, however, since seizures

intravascular placement. A 0.5 ml test dose is performed to rule out intravascular injection into the vertebral artery. Th

immediately, even with very small volumes of local anaesthetic. This test dose is followed by a 3 ml epinephrine-contai dose to rule out intravenous placement. The remainder of the anaesthetic (10-15 ml) is injected in divided doses of 3 m stellate ganglion. The onset of Horner's syndrome indicates a successful block. Q212. ect is least effective in ;; 1obsessions;; 2neuroleptic malignant syndrome;; 3depression;; 4delirium;;

intermittent aspiration. The patient is placed in the sitting position to facilitate the spread of anaesthesia inferiorly to th

the problem in psych is any damn thing which is not responding to drug therapy they will give ECT...so ect will b menti most of the psych conditions..as GMAX SAID approach by exclusion is best ..NMS N DEPRESSION definitely used..now

obsession severe enough to cause severe depression ECT USED WHEN ALL OTHER THERAPY FAILS...in delirium -an acu confusional state occuring in various medicl,surgical,metabolic ,toxic or post op conditions-ECT IS HARDLY OF USE CAU MORE HARM THAN BENIFIT EVEN IN RESISTANT STOPOUROUS CASES... SO IT VL GO WITH GMAX'S ANS-DELIRIUM >OBSESSION Q213. ALEXITHYMIA IS ;; 1.a feeling of intense rapture;; 2pathological sadness;; 3affective flattening;; 4inability to recognise and desribe feelings;; Alexithymia is a relatively new term which means the inability to express feelings with words. Q214. which of the following skin tuberculosis show -highest immunity;; 1tbvc;; 2lupus vulgaris;; 3lupus miliary;; 4tb chancre;; Primary Inoculation Tuberculosis (Tuberculous Chancre; Tuberculous Primary Complex)

Primary inoculation tuberculosis results from the inoculation of mycobacteria into the skin of a host not previously infec skin. INCIDENCE

tuberculosis. The tuberculous chancre and the affected regional lymph nodes constitute the tuberculous primary compl

Primary inoculation tuberculosis is estimated to constitute a very small proportion of all primary tuberculous lesions; ho

may be not quite as rare as generally believed. In some regions, particularly in Asia where the incidence of tuberculosi high and where living conditions and hygiene are poor, primary inoculation tuberculosis of the skin is not unusual. Most

individuals are children, but the lesions may also occur in adolescents and young adults. All parts of the body may be a on the mucous membranes of the conjunctiva and oral cavity. PATHOGENESIS

but sites of predilection are the face, hands, and lower extremities, which are readily injured. One-third of the lesions a

Because tubercle bacilli cannot actively penetrate intact skin, they are introduced into the tissue at the site of minor ab

or wounds. Rarely, infection may occur in healthy individuals after sexual contact with patients suffering from genitouri tuberculosis. Oral lesions may be due to bovine bacilli from nonpasteurized milk and occur after mucosal trauma or too

extraction. The skin lesion appears 2 to 4 weeks after inoculation. Infection spreads to the regional lymph nodes, produ

tuberculous lymphadenitis; with increasing acquired immunity, the process is localized to the particular region involved

inoculation tuberculosis is initially a multibacillary type of skin tuberculosis that becomes paucibacillary as immunity de CLINICAL MANIFESTATIONS

The tuberculous chancre initially presents as a small papule, scab, or wound with little tendency to heal. A painless ulce

develops, which may be quite insignificant or may enlarge to a diameter >5 cm ( Fig. 200-1). The lesion is shallow with granular or hemorrhagic base studded with miliary abscesses or covered by necrotic tissue. The ragged edges are unde and of a reddish-blue hue; as the lesions grow older, they become more indurated, with thick adherent crusts.

Wounds inoculated with tubercle bacilli may heal temporarily but break down later, giving rise to granulating ulcers. Mu infections result in painless ulcers or fungating granulomas.

Inoculation tuberculosis of the finger may present as a painless paronychia; inoculations of mycobacteria in puncture w

have resulted in subcutaneous abscesses. Slowly progressing, regional lymphadenopathy develops 3 to 8 weeks after t

infection (see Fig. 200-1) and may rarely be the only clinical symptom. After weeks or months, cold abscesses may dev

that perforate to the surface of the skin and form sinuses; the lymph nodes draining the primary glands may also be in

Body temperature may be slightly elevated; occasionally, lymph node enlargement, abscess formation, and perforation take a more acute course. Fever, pain, and inflammatory swelling of the surrounding tissues simulating a pyogenic infe present in half the cases. HISTOPATHOLOGY

In the early phase, there is an acute nonspecific inflammatory reaction in both skin and lymph nodes, and mycobacteri may occur. DIAGNOSIS AND DIFFERENTIAL DIAGNOSIS

easily detected. After 3 to 6 weeks, the infiltrate and the regional lymph nodes acquire a tuberculoid appearance and c

Lack of awareness of the condition is probably the most common reason for diagnostic error. Any ulcer with little or no tendency to heal and unilateral regional lymphadenopathy in a child should always arouse suspicion. Acid-fast organism

demonstrated in histologic sections or in smears obtained from the primary ulcer and draining nodes in the initial stage

disease, but may be difficult to find in older lesions. The diagnosis is verified by bacterial culture. The reaction to intrad

PPD is negative initially and later converts to positive (see Fig. 200-1). A previous tuberculous infection should be exclu differential diagnosis includes the primary complexes of syphilis, tularemia, cat-scratch fever, sporotrichosis, and ulcera lesions of other mycobacterioses as well as other forms of skin tuberculosis. COURSE

Without treatment, the condition may last up to 12 months. Lesions heal by scarring, but in rare cases, lupus vulgaris d

at the site of a healed tuberculous chancre; in more than 50 percent of patients the regional lymph nodes calcify. Usua

primary tuberculous complex yields satisfactory immunity, but reactivation of the disease may occur. Hematogenous sp may give rise to tuberculosis of other organs, particularly of the bones and joints. Depending on the size of the inoculu Erythema nodosum is a feature in approximately 10 percent of the cases. Tuberculosis Verrucosa Cutis (Warty Tuberculosis) This disorder is verrucous skin tuberculosis caused by exogenous reinfection in previously sensitized individuals. INCIDENCE

the age and resistance of the host, primary inoculation tuberculosis may progress to acute miliary disease with fatal ou

In Western countries, tuberculosis verrucosa cutis is rare. It is the most frequent form of tuberculous skin disease in H Kong, accounting for more than 40 percent of cases. PATHOGENESIS

Tuberculosis verrucosa cutis is inoculation tuberculosis occurring in persons who have acquired a certain degree of imm

thus, skin tests are highly positive and it is a paucibacillary type of skin tuberculosis. Inoculation occurs at sites of mino wounds or abrasions or, rarely, from the patient's own sputum. In the past, certain professional groups, particularly ph patients or from autopsy material (verruca necrogenica, anatomist's wart, postmortem wart). Farmers, butchers,

pathologists, medical students, and laboratory attendants, were most liable to acquire warty tuberculosis from tubercu

knackers contracted the disease from tuberculous cattle, and in these cases, M. bovis was responsible. Children can be infected by playing on contaminated ground, a frequent occurrence in underdeveloped countries. CLINICAL MANIFESTATIONS

Tuberculosis verrucosa cutis usually occurs on the hands. In children, the sites of predilection are the lower extremities lesions are asymptomatic and start as a small papule or papulopustule with a purple inflammatory halo; they become

hyperkeratotic and are often mistaken for a common wart. Slow growth and peripheral expansion lead to the developm

verrucous plaque with an irregular outline and a papillomatous horny surface. Clefts and fissures discharging pus exten

the underlying brownish-red to purplish infiltrated base. The lesion usually is solitary, but multiple lesions may occur. T regional lymph nodes are rarely affected but may become enlarged from secondary bacterial infection. HISTOPATHOLOGY The most prominent histopathologic features are pseudoepitheliomatous hyperplasia with marked hyperkeratosis, a de

inflammatory infiltrate, and abscesses in the superficial dermis or within the pseudoepitheliomatous rete pegs. Epithelio

and giant cells are found in the upper and middle dermis; typical tubercles are uncommon. Mycobacteria can occasiona seen. At times, the dermal infiltrate may be nonspecific. DIAGNOSIS AND DIFFERENTIAL DIAGNOSIS Early lesions resemble warts or keratoses. Hyperkeratotic lupus vulgaris exhibits apple-jelly nodules at the periphery

occurs in sites where tuberculosis verrucosa cutis is rare (see below). Blastomycosis, chromomycosis, and bromoderma

similar clinically and histopathologically but can be excluded by special stains and by culture. Negative fungal cultures a

tuberculoid foci are diagnostic aids. Chronic vegetating pyoderma and hyperkeratotic lesions due to other mycobacteria difficult to exclude. Hypertrophic lichen planus is pruritic and more disseminated, and usually other cutaneous and muc lesions are found. Tertiary syphilis is not quite as verrucous and is accompanied by diagnostic serology.

COURSE The lesions progress slowly, and, if untreated, persist for many years. Secondary pyogenic infection may temporarily le results in sunken atrophic scars. Occasionally, ulcerative and sclerotic lesions or fungating granulomas are seen. Tuberculous Cellulitis

more acute inflammatory changes, with lymphangitis and regional lymphadenitis. Spontaneous involution does occur a

Two cases of cellulitis caused by M. tuberculosis were recently described in patients on long-term systemic glucocorti

therapy. 12 A mode of infection was not determined, although one of the patients had been treated previously for pulm tuberculosis. Histopathologically, multinucleated giant cells, granulomas, and acid-fast bacilli were present. Both patien responded to antituberculous antibiotics Lupus Vulgaris

Lupus vulgaris is an extremely chronic and progressive form of tuberculosis of the skin occurring in individuals with mo immunity and a high degree of tuberculin sensitivity. INCIDENCE

Although the incidence of lupus vulgaris has steadily declined during the past decades, it was estimated in 1960 that so Females appear to be affected two to three times as often as males; all age groups are affected equally. PATHOGENESIS

50,000 new cases occur throughout the world every year. It has always been less common in the United States than in

Lupus vulgaris is a postprimary, paucibacillary form of skin tuberculosis arising in previously sensitized individuals with

moderate immunity. I t originates by hematogenous, lymphatic, or contiguous spread from tuberculosis elsewhere in the body, most often fro cervical adenitis or pulmonary tuberculosis, but sometimes from an old, apparently quiescent primary complex.

Rarely, it follows primary inoculation tuberculosis or BCG vaccination 13 (see below). The lesions progress steadily. Spontaneous involution may occur and new lesions may arise within old scars. Complete healing occurs only rarely with therapy.

CLINICAL MANIFESTATIONS The lesions are usually solitary, but two or more sites may be involved simultaneously; in patients with active pulmona

tuberculosis, multiple foci may develop. In approximately 90 percent of patients, the head and neck are involved. Lupu

vulgaris usually starts on the nose or cheek and slowly extends onto adjacent areas. The earlobes are often affected, a

solitary patches may be encountered on the scalp. Only a small percentage of the lesions occur on the extremities, and for patients with disseminated lupus vulgaris, the trunk is rarely involved.

In general, lupus vulgaris is asymptomatic. The initial lesion is a brownish-red, soft or friable lupus macule or papule w

smooth surface or covered by a scale. On diascopy the infiltrate exhibits a typical apple-jelly color. Progression is chara by elevation, a deeper brownish color ( Fig. 200-4), and formation of a plaque. Involution in one area and simultaneou expansion in another result in a gyrate outline.

Lesions may become flat plaques with a serpiginous or polycyclic outline and a smooth surface or psoriasiform scaling; may be erosions, ulceration, and scarring. Hypertrophic forms appear as a soft mass with a nodular, hyperkeratotic sur

Edema, lymphatic stasis, recurrent erysipelas, elephantiasic thickening, and vascular dilatation may lead to gross defor results in extensive destruction and disfigurement. Atrophic scarring, with or without prior ulceration, is a prominent feature of lupus vulgaris. Recurrence within a scar is characteristic. Sometimes, fibrosis is very pronounced and leads to deformations, mutilations, and contractures. Lupus vulgaris of mucous membranes:-

ulcerative forms, the underlying tissue may be affected by progressive necrosis; involvement of the nasal or auricular c

The mucosae may be primarily involved or become affected by the extension of skin lesions. They show small, soft, gra

papules, ulcers, or granulating masses that bleed easily. A dry rhinitis is often the only symptom of early nasal lupus, b progressive lesions destroy the cartilage of the nasal septum. Scarring of the soft palate and stenosis of the larynx may result. Lupus postexanthematicus:-

After a transient impairment of immunity, particularly after measles (thus the term lupus postexanthematicus), multipl

disseminated lesions may arise simultaneously in different regions of the body as a consequence of hematogenous spre

a latent tuberculous focus. During and after the eruption, a previously positive tuberculin test may become negative bu usually revert to positive as the general condition of the patient improves. Clinically and histopathologically, the lesions tuberculosis of the skin. HISTOPATHOLOGY

postexanthematic lupus are typical for lupus vulgaris, and their appearance distinguishes the condition from acute milia

The most prominent histopathologic feature is the formation of typical tubercles ( Fig. 200-9). Secondary changes may

superimposed: epidermal thinning and atrophy or acanthosis with excessive hyperkeratosis or pseudoepitheliomatous hyperplasia; acid-fast bacilli are usually not found. Nonspecific inflammatory reactions may partially conceal the tuberc

structures. Old lesions are composed chiefly of epithelioid cells and may be impossible to distinguish from sarcoidal infi DIAGNOSIS AND DIFFERENTIAL DIAGNOSIS

Typical lupus vulgaris plaques do not present diagnostic problems; they have to be distinguished from lesions of sarcoi

lymphocytoma, discoid lupus erythematosus, tertiary syphilis, leprosy, blastomycosis or other deep mycotic infections,

leishmaniasis, and chronic vegetating pyodermas. Criteria helpful in the diagnosis are the softness of the lesions, the b red color, and the slow evolution. The apple-jelly nodules revealed by diascopy are highly characteristic; finding them m tuberculosis/bovis confirm the diagnosis. The tuberculin test is strongly positive except for the early phases of postexanthematic lupus. COURSE Lupus vulgaris is extremely chronic, and without therapy its course usually extends over many years or even decades.

decisive, especially in ulcerated, crusted, or hyperkeratotic lesions. Histologic examination and a positive culture for M.

adults or older patients tend to have more extensive lesions than children. The disease is progressive and leads to cons the cartilages of the face and scarring lead to disfigurement (see Fig. 200, to ectropion with its complications, to .

impairment of function and to disfiguration. Contractions result in a reduction of joint mobility, and ulceration and dest microstomia with impairment of speech and food intake, and to other severe mutilations. ****The most serious complication of long-standing lupus vulgaris is the development of carcinoma (see Fig. 200-

early 1900s, carcinoma was estimated to occur in almost 10 percent of patients. Squamous cell carcinomas outnumber cell carcinomas by far, and the incidence of metastases is surprisingly high.

RELATIONSHIP OF LUPUS VULGARIS TO TUBERCULOSIS OF OTHER ORGANS In 40 percent of patients there is associat

tuberculous lymphadenitis, and 10 to 20 percent have pulmonary tuberculosis or tuberculosis of the bones and joints. T of occurrence of pulmonary tuberculosis is 4 to 10 times higher in patients with lupus vulgaris than in the general popu

some cases, lupus vulgaris may be regarded as a symptom of another tuberculous disease running a more serious cou Scrofuloderma (Tuberculosis Colliquativa Cutis)

Scrofuloderma is a subcutaneous tuberculosis leading to cold abscess formation and a secondary breakdown of the ove skin.

PATHOGENESIS Scrofuloderma results from contiguous involvement of the skin overlying another tuberculous process, commonly tuberculous lymphadenitis, tuberculosis of bones and joints, or tuberculous epididymitis. It may affect all ag groups, although there is a higher prevalence among children, adolescents, and the aged. It may be either a multibacillary or a paucibacillary form of skin tuberculosis.

CLINICAL MANIFESTATIONS Tuberculosis colliquativa most often occurs in the parotidal, submandibular, and supraclavicular regions, as well as on t

lateral aspects of the neck; lesions may be bilateral. Lesions on the extremities or on the trunk accompany tuberculous

of the phalangeal bones, the joints, the sternum, and the ribs. Skin lesions first present as firm, subcutaneous nodules

well-defined, freely movable asymptomatic infiltrate. As the infiltrate enlarges it softens, but it may take months before liquefaction with subsequent perforation ( Fig. 200-10). Ulcers and sinuses develop and discharge watery and purulent caseous material. The ulcers are linear or serpiginous with undermined, inverted, bluish edges and uneven, soft, and

granulating floors. Sinusoidal tracts undermine the skin, and clefts and dissecting subcutaneous pockets alternate with

nodules; scar tracts develop and bridge ulcerative areas or even stretches of normal skin. Tuberculin sensitivity is usua pronounced. other types tub gumma : poorly nourished children, hgous spread ,firm subcut nodules,undermined ulcer oroficial tub: immunocompromised lips tongue genitalia reddish painful ulcer due to auto inoculation from advanced internal git tb remaining are tuberculids lichen scruflosorum papulonecrotic tuberculid eryhthema nodosum Q215. which of the following is least likely to present as an eccentric osteolytic lesion;; 1aneurysmal bone cyst;; 2giant cell tumor;; 3fibrous cortical defect;; 4simple bone cyst;; ANS-SIMPLE BONE CYST REF-APLEY 8TH PG 178 READ THE TABLE V.IMP TOPIC ECCENTRIC EXPANSILE LESION 1)aneurysmal bone cyst

2)fibrous cortical defect 3)giant cell tumor 4)non ossifying fibroma 5)chondromyxoid fibroma CENTRAL NON EXPANSILE 1)simple (unicameral bone cyst 2)enchondroma 3)chondroblastoma Q216. MERCEDEZ BENZ SIGN IS SEEN IN ;; 1porcelain gallbladder;; 2limely bile;; 3gall stones;; 4emphysematous cholecystitis; MERCEDEZ BENZ SIGN * + "Mercedes Benz" sign is a triradiate collection of nitrogen gas # Fills crevices created by shrinkage of cholesterol crystals in the stone # Radiolucent fissures usually widest centrally radiating like points of star ANS-GALL STONE REF-BAILEY 25 PG 1115

The Mercedes-Benz sign describes a star-shaped pattern of gas-fissuring within gallstones initially described on an abd

radiograph . Fissures, usually fluid-filled, are present in close to 50% of gallstones. Less than half of these fissured gall

contain some amount of gas . The radiolucency caused by the gas usually appears in a triradiate pattern, mimicking th Mercedes-Benz logo. IT IS ALSO CALLED SEA-GULL SIGN Q217. the term lead time is used in ;; 1pollution studies;; 2inventory management;; 3entomology;; 4rehabilitation;; answer inventory management

goin logically answer v wud tempt to go for pollution studies as pollution will hav effect over a period of time nt immed there s some time taken leadin to its effect... n it cud hav been answer if inventory management nt in choice..... because its specifically use with it... Q218. which of the following is not an example of non-random sampling;; 1quota sampling;; 2purposive sampling;; 3convenient sampling;; 4cluster sampling;; Q219. which of the following is not a charecteristic of indian census;; 1international simultaneity;; 2delimited territory;; 3universality;;

4dejure nature;; Q220. which of the folowing is not an example of latent infection;; 1yaws;; 2plasmodium vivax;; 3tuberculosis;; 4brill-zinser disease;;

A lingering infection that may lie dormant in the body for a period of time but may become active under certain conditi

Back to top dilshan.


Guest Posted: Tue Jul 27, 2010 12:23 pm Post subject:

Q219. answer is dejure nature int simultaneity... that its along wth int census every 1o years>> census movement..starts wth sunrise of 1st march delimited territory..no territorial limit ..reacf far flung places... every area covered

indian census is de facto..not de jure... de jure census tallies ppl accord to their regular or legal residence... where as de facto census allocates them to place ehere they prsnt at time of enumration,,,,,,,,, FROM CENSUS OF INDIA WEB SITE Some of the essential features of the census are as follows:

Sponsorship: To conduct a census, vast organisation and considerable resources are needed. The census organisation has to mobilise cooperation of States and Local Governments. Defined Territory: changes in its area in successive censuses should be clearly and explicitly stated. Universality ensure completeness and accuracy of census data.

extensive administrative machinery with adequate legislative authority. This can be done only by National Government

The population figures have no meaning unless they refer to a well defined territory. The territory covered, alongwith a

:Each person present and/or residing within its scope, without omission or duplication should be included in the census

Simultaneity:The total population enumerated should refer to one well defined point of time and the data collected sho

refer to a well defined point or period of time. This is essential to an accurate count of the total population and relation

facts about the population to a specified period of time. As a general rule, a day is fixed for the census and also a parti moment which is called the "census moment". In India, the "Census moment" now is the sunrise of 1st March, of the c

year. The fixing of the "census moment" helps the enumerators to decide which persons are to be included in the censu

persons born after the census moment or the persons dying before the census moment are to be excluded from the ce

Some of the characteristics of the population like age, marital status, occupation, literacy and birthplace etc., are refer period of time usually from 10th February to 28th February of the census year to ensure simultaneity.

Defined Periodicity: Censuses should be taken at regular intervals so that comparable information is made available in sequence. A series of censuses makes it possible to appraise the past, accurately describe the present and estimate th

Individual Units: Census data must be collected separately for each individual so that detailed classifications may be pr

all the required combinations. A procedure of "group enumeration" is not a census in the strict sense of the term becau

recording of aggregated or summarized information on the characteristics of a group of person usually precludes the cr

tabulation of data on several characteristics. Even though a well-designed "group enumeration" can produce crossclassifications of certain individual characteristics, such as sex and age, the possibilities in this respect are so limited th procedure is not recommended for general use, particularly since it tends to result in under-enumeration of the popula provided that the sample design is consistent with the size of the areas for which the data are to be tabulated and the detail in the cross-tabulations to be made.

Individual enumeration does not preclude the use of sampling techniques for obtaining data on specified characteristics

Compilation and Publications: No census is complete unless the data collected are compiled and published by geograph

and by basic demographic variables. The unpublished data is of no use to the potential users of the census data. That i the census should be limited to very important items which can be tabulated and published in time.

International Simultaneity: The census of any country is of greater value nationally, regionally and internationally if it c compared with the censuses of other countries which are taken at approximately the same time. Censuses in most cou International comparability is thus maintained. The Indian census accompanies all the above features.

the world are conducted in years ending in 0 or 1. In India, the decennial censuses are conducted in years ending in 1.

Back to top kollin.


Guest Posted: Tue Jul 27, 2010 12:27 pm Post subject:

Q221. osteomalacia and painful bone fractures are commonly seen in chronic poisoning by ;; 1arsenic;; 2lead;; 3cadmium;; 4mercury;; answer cadmium

Cadmium has no constructive purpose in the human body. Cadmium is extremely toxic even in low concentrations, and bioaccumulate in organisms and ecosystems.

Acute exposure to cadmium fumes may cause flu like symptoms including chills, fever, and muscle ache sometimes ref as "the cadmium blues." Symptoms may resolve after a week if there is no respiratory damage. More severe exposures

cause tracheo-bronchitis, pneumonitis, and pulmonary edema. Symptoms of inflammation may start hours after the ex

and include cough, dryness and irritation of the nose and throat, headache, dizziness, weakness, fever, chills, and ches

Inhaling cadmium-laden dust quickly leads to respiratory tract and kidney problems which can be fatal (often from rena failure). Ingestion of any significant amount of cadmium causes immediate poisoning and damage to the liver and the Compounds containing cadmium are also carcinogenic. The bones become soft (osteomalacia), lose bone mineral density (osteoporosis) and become weaker. This causes the causes a fracture.

the joints and the back, and also increases the risk of fractures. In extreme cases of cadmium poisoning, mere body w

Q222. a pregnant 35 year old patient isat highest risk for the concurrent development of which of the following maligna

1cervix;; 2ovary;; 3breast;; 4vagina;; answer cervix ques direct pick from pre test seroes of obg ..ques no. 281..pg 231 answer given

Cervical cancer is a more common gynecologic malignancy in pregnancy than ovarian or breast cancer due to the fact t

a disease of younger women. Management of cervical intraepithelial lesions is complicated in pregnancy because of inc

vascularity of the cervix and because of the concern that manipulation of and trauma to the cervix can compromise con of the pregnancy. A traditional cone biopsy is only indicated in the presence of apparent microinvasive disease on a

colposcopically directed cervical biopsy. Otherwise, more limited procedures such as shallow coin biopsies are more app age at which the diagnosis is made and the severity of the disorder. Survival is decreased for malignancies discovered pregnancy. Radiation therapy almost always results in spontaneous abortion, in part because the fetus is particularly radiosensitive. Chemotherapy is associated with higher than expected rates of fetal malformations consistent with the exposure occurs.

If invasive cancer is diagnosed, the decision to treat immediately or wait until fetal viability depends in part on the gest

antimetabolite effects of agents used. Specific malformations depend on the agent used and the time in pregnancy at w

* Late pregnancy or never being pregnant: Because estrogen levels are lower during pregnancy, breast tissue is expos more estrogen in women who become pregnant for the first time after age 35 or who never become pregnant. Q223. RETRACTILE MESENTERY MAY BE SEEN IN ;; 1ormonds disease;; 2gardners syndrome;; 3turners syndrome;; 4down syndrome;; answer ormonds disease also known as sclerosing retroperitonitis,characterised by diffuse fibrous overgrowth and chronic inflammation.may be idiopathic,the etiologic role of ergot derivative drugs and autoimmune reaction has been suggested

similar process in the mediastinum,sclerosing cholangitis and riedel's thyroiditis and termed multifocal fibrosclerosis.tho

Circulating antibodies to ceroid, a lipoproteinaceous by-product of vascular atheromatous plaque oxidation, are present than 90% of patients with retroperitoneal fibrosis. The relationship of this finding to the occurrence of fibrosis remains uncertain. The early inflammatory reaction is predominated by T-helper cells, plasma cells, and macrophages, but is subsequently replaced by collagen-synthesizing fibroblasts.

Medical therapy is initiated with prednisone (60 mg every other day, for 2 months). Following this initial therapy, predn

gradually tapered off over the next 2 months. Therapeutic efficacy is assessed on the basis of patient symptomatology,

erythrocyte sedimentation rate, and diagnostic imaging. Cyclosporine, tamoxifen, or azathioprine may be used to treat who are recalcitrant to the above regimen. Q224. two parents are both affected with albinism;;but have a normal child;;which of the following terms best applies situation;; 1allelic heterogeneity;; 2locus heterogeneity;; 3variable expressivity;; 4incomplete penetrance;; Q225. agent of choice for obese patients;; 1desflurane;;

2isoflurane;; 3sevoflurane;; 4halothane Desflurane, sevoflurane,

and isoflurane are minimally metabolized and are therefore useful agents in the obese patient, with desflurane possibly

providing better hemodynamic stability. Sevoflurane provides rapid recovery, good hemodynamic control, infrequent in

of nausea and vomiting, and prompt regaining of psychological and physical functioning when compared to isoflurane. Metabolism of volatile anesthetics is greater in obese than in normal-weight patients, which is reflected by a greater inc

serum inorganic fluoride, including with sevoflurane, whose biotransformation has not been shown to result in significa

differences in plasma fluoride levels nor differences in pre- and postoperative liver function and renal function tests bet

obese and nonobese patients. A potentially hepatotoxic reductive pathway metabolizes halothane in obese patients, res an increased incidence of halothane hepatitis. Fortunately, halothane is rarely FIGURE 36-8. A. The preformed head e

pillow. B. Proper head-elevated laryngoscopy position (HELP) placement with the head elevation pillow combined with a

standard intubating pillow as described in the text. needed in modern-day practice. Evidence does not support the sugg

that significant delayed recovery and awakening from volatile anesthetic agents occurs in obese patients when compar nonobese. Rapid elimination and analgesic properties make nitrous oxide an attractive choice for anesthesia in obese p

but high oxygen demand in this patient population limits its use. Short-acting opioids at the lowest possible dose, comb Cisatracurium possesses an organ-independent elimination profile and is a favorable nondepolarizing muscle relaxant f during maintenance of anesthesia. Vecuronium and rocuronium are also useful choices. Dexmedetomidine, an alpha-2 agonist with sedative and analgesic properties, provides hemodynamic stability without

with a low solubility inhalation anesthetic, facilitate a more rapid emergence without increasing opioid-related side effe

myocardial depression. It has no clinically significant adverse effects on respiration, which makes it an attractive agent as an anesthetic adjunct in obese patients.55 Furthermore, it reduces the postoperative opioid analgesic requirements subsequent detrimental respiratory depressant effects.

Ventilatory tidal volumes greater than 13 mL/kg offer no added advantages during ventilation of anesthetized morbidly

patients. Increasing tidal volumes further only increases the peak inspiratory airway pressure, end-expiratory (plateau) pressure, and lung compliance without significantly improving arterial oxygen tension.57 Arterial oxygenation during

laparoscopy in morbidly obese patients is affected mainly by body weight and not body position, pneumoperitoneum, o

of ventilation, and oxygenation is not significantly improved by increasing either the respiratory rate or tidal volume.58

end-expiratory pressure is the only ventilatory parameter that has consistently been shown to improve respiratory func obese subjects.59 PEEP may, however, decrease venous return and cardiac output.

Back to top

Posted: Wed Jul 28, 2010 12:30 pm

Post subject:

Q224. ANS-incomplete penetrance ref-ROBBINS8, HARRISON 17 PG 396 VERY nicely explained

1)allelic heterogenity-I different mutations occurring within the same gene produce disorders, it is said to manifest allelic hetero term has been used when a number of different alleles cause a similar phenotype or different phenotypes. Eg- Different FBN1 mutations causing Marfan's syndrome Cystic fibrosis is caused by mutations in 136 different alleles 2)LOCUS HETEROGENITY -If mutations in unrelated gene loci cause a single disorder, it is referred to as locus heterogeneity

3)VARIABLE EXPRESSIVITY-This differs from penetrance, which refers to the likelihood of the gene generating any phenotype at

expressivity refers to the influence of an expressed gene in individuals. Variable expressivity occurs when a phenotype is expres different degree among individuals with the same genotype. For example, individuals with the same allele for gene involved in a

trait like body height might have large variance some are taller than others, making prediction of the phenotype from a particula

alone difficult

4)incomplete penetrance or reduced penetrance. Penetrance is said to be reduced or incomplete when some individuals fail to ex trait, even though they carry the allele. Q226. MAX PROPENSITY TO FOR RELEASING HISTAMINE;; 1dtc;; 2pancuronium;; 3gallamine;; 4atrracurium;; ans-dtc ref-MORGANS 4 TH,LEE12

MC CAUSE-Tubocurarine, the first muscle relaxant used clinically, often produced hypotension and tachycardia through histamin

ability to block autonomic ganglia was of secondary importance. Histamine release could also produce or exacerbate bronchospa its no more used now

OTHER LIKELY CAUSES -Histamine release from mast cells can result in bronchospasm, skin flushing, and hypotension from per

vasodilation. Both atracurium and mivacurium are capable of triggering histamine release, particularly at higher doses. Slow inje and H1 and H2 antihistamine pretreatment ameliorate these side effects also mus.relax. least likely to cause histamin release-vecuronium Q227. epidural blood patch is used in the treatment of ;; 1spinal headache;; 2cauda equino syndrome;; 3chronic back ache;; 4 none of the above;; ans-spinal headache

Any breach of the dura may result in a postdural puncture headache (PDPH) also called SPINAL HEADACHE. This may follow a d

lumbar puncture, a myelogram, a spinal anesthetic, or an epidural "wet tap" in which the epidural needle passed through the ep tap is usually immediately recognized as CSF dripping from the needle or aspirated from an epidural catheter. . Typically, PDPH is bilateral, frontal or retroorbital, and occipital and extends into the neck. It may be throbbing or constant and relieved or decreased by lying down flat. The onset of headache is usually 1272 h following the procedure; however, it may be immediately. Untreated, the pain may last weeks, and in rare instances has required surgical repair

and entered the subarachnoid space. Similarly, an epidural catheter might puncture the dura at any time and result in PDPH. An

with photophobia and nausea. The hallmark of PDPH is its association with body position. The pain is aggravated by sitting or st

An epidural blood patch is a very effective treatment for PDPH. It involves injecting 1520 mL of autologous blood into the epidu

or one interspace below, the level of the dural puncture. It is believed to stop further leakage of CSF by either mass effect or co The effects may be immediate or may take some hours as CSF production slowly builds intracranial pressure. Approximately 90%

will respond to a single blood patch, and 90% of initial nonresponders will obtain relief from a second injection. Prophylactic bloo

has been advocated by injecting blood through an epidural catheter that was placed after a wet tap. However, not all patients w

PDPH, and the tip of the catheter may be many levels away from the dural defect. Alternatively, a saline bolus can be injected th epidural catheter but does not appear to be as effective as blood patching. Most practitioners either offer the epidural blood patc PDPH becomes apparent or allow conservative therapy a trial of 1224 h. Q228. what is a straddle fracture;; 1fracture of the skull;; 2fracture of the c5 vertebra;; 3multiple rib fractures both sides of midline;; 4pelvic fractures;;

ans-pelvic # ref-APLEYS 8TH,MAHESHWARI straddle fracture-Bilateral fractures of the superior and inferior public rami List of fracture eponyms

Aviators astragalus:Implies a variety of fractures of the talus; described after World War I as rudder bar is driven into foot durin crash.

Bartons fracture:Displaced articular lip fracture of the distal radius; may be associated with carpal subluxatioin. Fracture configu be in a dorsal or volar direction. Bennetts fracture: Oblique fracture of the first metacarpal base separating a small triangular fragment of the volar lip from the displaced metacarpal shaft.

Bosworth fracture: Fracture of the distal fibula with fixed displacement of the proximal fragment posteriorly behind the posterola ridge. Boxers fracture: Fracture of the fifth metacarpalneck with volar displacement of the metacarpal head.

Burst fracture: Fracture of the vertebral body from axial load, usually with outward displacement of the fragments. May occur in thoracic, or lumbar spine.

Chance fracture: Distraction fracture of the thoracolumbar vertebral body with horizontal disruption of the spinous process, neur vertebral body.

Chauffeurs fracture (Hutchinsons fracture): Oblique fracture of the radial styloid, initially attributed to the starting crank of an e forcibly reversed by a backfire.

Choparts fracture and dislocation: Fracture and/or dislocation involving Choparts joints (talonavicular nd calcaneocuboid) of the

Clay-shovelers (coal-shovelers) fracture: Spinous process fracture of the lower cervical or upper thoracic vertebrae. Injury initi force opposite to the neck musculature.

attributed to workers attempting to throw upward a full shovel of clay, but the clay, adhering to the shovel, would cause a sudde

Collesfracture: General term for fractures of the distal radius with dorsal displacememnt, with or without an ulnar styloid fractur Cottons fractures: Trimalleolar ankle fracture with fractures of both malleoli and the posterior lip of the tibia. Die-punch fracture: Intraarticular fracture of the distal radius with impaction of the dorsal aspect of the lunate fossa.

Dupuytrens fracture: Fracture of the distal fibula with rupture of the distal tibiofibular ligaments and lateral displacement of the Duverneys fracture: Fracture of the iliac wing without disruption of the pelvic ring. Essex-Loprestis fracture: Fracture of the radial head with associated dislocation of the distal radioulnar joint. Galezzis fracture: Fracture of the radius in the distal third associated with subluxation of the distal ulna. Greenstick fracture: Incompletely fractured bone in a child, with a portion of the cortex and periosteum remaining intact on the side of the fracture. Hangmans fracture: Fracture through the neural arch of the second cervical vertebra (axis).

Hill-Sachs fracture: Posterolateral humeral head compression fracture caused by anterior glenohumeral dislocation and impactio humeral head against the anterior glenoid rim.

Holstein-Lewis fracture: Fracture of the distal third of the humerus with entrapment of the radial nerve.

Jeffersons fracture: Comminuted fracture of the ring of the atlas due to axial compressive forces. Fractures usually occur anteri posterior to the lateral facet joints. Jones fracture: Diaphyseal fracture of the base of the fifth metatarsal.

Lisfrancs fracture dislocation: Fracture and/or dislocation involving Lisfrancs (tarsometatarsal) joint of the foot. Lisfranc was on Napoleons surgeons and described traumatic foot amputation through the level of the tarsometatarsal joint.

Maisonneuves fracture: Fracture of the proximal fibula with syndesmosis rupture and associated fracture of the medial malleolu of the deltoid ligament. Malgaignes fracture: Unstable pelvic fracture with vertical fractures anterior and posterior to the hip joint.

Mallet finger: Flexion deformity of the distal interphalangeal joint caused by separation of the extensor tendon from the distal ph tendon inserts. Monteggias fracture: Fracture of the proximal third of the ulna with associated dislocation of the radial head. Nightstick fracture: Isolated fracture of the ulna secondary to direct trauma.

deformity may be secondary to direct injury of the extensor tendon or an avulsion fracture from the dorsum of the distal phalan

Posadas fracture: Transcondylar humeral fracture with displacement of the distal fragment anteriorly and dislocation of the radi from the bicondylar fragment.

Potts fracture: Fracture of the fibula 2 to 3 in above the lateral malleolus with rupture of hte deltoid ligament and lateral sublux talus. Rolandos fracture: Y-shaped intraarticular fracture of the thumb metacarpal. Segonds fracture: Avulsion fracture of the lateral tibial condyle from the bony insertion of the iliotibial band. Shepherds fracture: Fracture of the lateral tubercle of hte posterior talar process.

Smiths fracture: Fracture of the distal radius with palmar displacement of the distal fragment. Also referred to as a reverse Coll Stiedas fracture: Avulsion fracture of the medial femoral condyle at the origin of the medial collateral ligament. Straddle fracture: Bilateral fractures of the superior and inferior public rami.

Teardrop fracture: Flexion fracture/dislocation of the cervical spine with associated triangular anterior fragment of the involved v Injury complex in unstable, with posterior ligamentous disruption.

Tillauxs fracture: Fracture of the lateral half of the distal tibial physis during differential closure of the physis. The medial part o physis has already fused. Torus fracture: Impaction fracture of childhood as the bone buckles instead of fracturing completely.

Walthers fracture: Inschioacetabular fracture that passes through the pubic rami and extends towards the sacroiliac joint. The m the acetabulum is displaced inward Q229. which bone takes part in the formation of lateral longitudinal arch;; 1cuboid;; 2talus;;

3medial cuneiform;; 4lateral cuneiform;; answer cuboid ref keith l moore

longitudinal arch of the foot is composed of medial and lateral parts. Functionally, both parts act as a unit with the transverse ar foot, spreading the weight in all directions.

The medial longitudinal arch is higher and more important than the lateral longitudinal arch (Fig. 5.69A & D). The medial longitu

composed of the calcaneus, talus, navicular, three cuneiforms, and three metatarsals. The talar head is the keystone of the med longitudinal arch. The tibialis anterior, attaching to the 1st metatarsal and medial cuneiform, helps strengthen the medial longitu The fibularis longus tendon, passing from lateral to medial, also helps support this arch

The lateral longitudinal arch is much flatter than the medial part of the arch and rests on the ground during standing. It is made calcaneus, cuboid, and lateral two metatarsals.

Q230. korners septum , is formed by which of the following;; 1sqamous tympanic fissure;; 2lateral margin of tegmen tympani;; 3petrosqamous suture;; 4petrosqamous fissure;;

CT scan, temporal bone. An axial view through the superior portion of the temporal bone, which demonstrates the anterior (a) a

(b) crura of the superior semicircular canal. The surrounding otic capsule is observed as dense white bone. The mastoid air cells

lateral to the otic capsule. The squamous air cells are separated from the petrous air cells by the Koerner septum (d). The poste of the temporal bone, or cerebellar plate, (e) forms the anterior margin of the posterior cranial fossa.

Posted: Mon Aug 02, 2010 12:42 pm

Post subject:

Q241. most of surface of pinnae is supplied by 1greater auricular;; 2lesser occipital;; 3auricotemporal;; 4auricular branch of vagus;; ans auriculotemporal pinna: Sensory The auriculotemporal nerve (from cranial nerve V) to the tragus,crus of helix & adjacent part of helix. The lesser occipital nerve (C2) to the upper 1/3 of the medial surface. The greater auricular nerve (C2,3) to the posterior part of the lateral and most(lower 2/3rd) of the

medial surfaces. Arnold's nerve(auricular branch of Vagus) to the area around the concha at the external meatus & corresponding area on medial surface. facial nerve -distribute with arnold's nerve,supplies concha & retroauricular groove. external auditory canal: Nerve supply: Sensory only Anterior wall & roof:auriculotemporal nerve. Posterior wall & floor: Arnolds nerve(auricular branch of vagus nerve) posterior wall of auditory canal also receives sensory fibres of facial nerve through auricular branch of vagus tympanic membrane: Nerve supply: Sensory only anterior of the lateral surface: The auriculotemporal nerve. posterior of the lateral surface: Arnolds nerve Medial surface: Jacobsons nerve.(Tympanic branch of glossopharyngeal nerve) note:Arnold's cough reflex-due to irritation of auricular branch of vagus nerve.also known as Ear cough. Q242. wave sign of mulvay or sail sign is seen in ;; 1craniopharyngioma;; 2fourth ventricular cyst;; 3thymic hyperplasia;; 4hydatid in lung;; ANS-THYMIC HYPERPLASIA REF-sumer sethi 4th On radiographs of children, the normal thymus is seen as a triangular structure with well-defined borders. It may blend with the heart, or have a notch between its inferior border and heart. Sometimes, it may produce an acute angle between its inferior border and cardiac outline, producing the so-called sail sign The borders may be wavy in outline due to indentation by the costal cartilages; this has been named the wave sign of Mulvey or the thymic wave sign. Q243. gobiff campaign in india is supported by which agency;; 1care;; 2rockfeller foundation;; 3bill and melenda gates foundation;; 4unicef;; ans unicef... the concept of "selective primary health care" was proposed to allow for the scarcity of resources available to achieve health for all. It involved defining strategies focusing on priority health problems (including infant and child mortality), using interventions that were feasible to implement, of low cost, and with proven efficacy(25,26). UNICEF's GOBI strategy of 1982 emerged from this. At its foundation were four child health interventions which met the above criteria and which were considered to be synergistic -- growth monitoring (G), oral rehydration therapy for diarrhoea (O), the promotion of breastfeeding (B) and childhood immunizations (I). Birth spacing/family planning (F), food supplementation (F) and the promotion of female literacy (F) were added subsequently (GOBI-FFF) Q244. which of the following is not located within the abductor canal;; 1 saphenous nerve;; 2 nerve to vastus medialis;; 3 femoral vein;; 4 deep femoral artery;;

Q245. which one of the following human genes lacks introns;; 1 dystrophins;; 2 immunoglobuling;; 3 histone;; 4 ldl receptor;; Q246. which type of mri demonstrates best anatomica; detail;; 1 t1 weighed;; 2 t2 weighed;; 3 spin-echo sequences;; 4 fat suppressed images;; Q247. in the treatment of a burn ;;which of the following is a hypertonic formula;; 1haila formula;; 2warden formula;; 3brooke formula;; 4demling formula;; Q248. stokes gritti operation is performed through the ;; 1ankle joint;; 2knee joint;; 3cervical spine;; 4 elbow; Rocco Gritti, (10) a surgeon at the Ospedale Maggiore in Milan, Italy, described a through-knee procedure in 1857 in which the patella and surrounding tissues were used as an osteo-plastic flap to provide an end-bearing stump. Thirteen years later, the procedure was modified by Sir William Stokes, (11) and the operation is now known as the Gritti-Stokes amputation just above the level of the knee, to be covered by the skin in front of the patella (Stokes-Gritti amputation). Both these end-bearing stumps are excellent. Properly fashioned, they will bear the body weight well, without distress to the patient, and without the development of secondary troubles, due to the artificial manner in which weight must be borne on any amputation stump. For both these stumps, excellent prostheses have been devised, which fit and function well. If circumstances preclude amputation at the level of the Syme or the Stokes-Gritti stump, it is necessary to fashion a stump which will carry the weight of the body at its upper end,-below the knee, on the flaring condyles of the tibia; and above the knee, on the ischial tuberosity. Stumps, therefore, are divided into two classes: (1) those which bear weight upon the lower end; and (2) those which bear weight at or near the upper end. In the latter group, with which this paragraph is concerned, the length of the stump is of importance only for the purpose of motivating the artificial limb. Its useful length, therefore, is more or less constant. From the foregoing, it is evident that there are four types of amplitations of the lower ext,remity which are of value. 249. people with impaired dna ligase activity are immunodeficient;;dna ligase joins;; 1parallel and antiparallel strands;; 2okazaki fragments;; 3primer and template strands;; 4major and minor grooves;; answer 2 Seals the single strand nick between the nascent chain and Okazaki fragments on lagging strand

Q250. one of the following is an example of atavistic epiphysis;; 1head of first metacarpal;; 2head of femur;; 3coracoid process of scapula;; 4trochanters of femur;;

vinay.
Guest

Back to top
Posted: Tue Aug 03, 2010 12:44 pm Post subject:

Q251. which of the following drug is most likely to cause impaired glucose tolerance;; 1sulfasalazine;; 2azathioprine;; 3methotrexate;; 4tacrolimus;; ans-tacrolimus ref kdt 5th Side effects can be severe and include infection, cardiac damage, hypertension, blurred vision, liver and kidney problems (tacrolimus nephrotoxicity), hyperkalemia, hypomagnesemia, hyperglycemia, diabetes mellitus, itching, and various neuropsychiatric problems such as loss of appetite, insomnia, Posterior reversible encephalopathy syndrome, confusion, weakness, depression, cramps, neuropathy, seizures, tremors, and catatonia Q252. all of the following are projective personality tests, except ;; 1rorschach inkblot test;; 2thematic appreciation test;; 3sentence completion test;; 4bender gesalt test;; The Bender Gestalt Test, or the Bender Visual Motor Gestalt Test, is a psychological assessment instrument used to evaluate visual-motor functioning and visual perception skills in both children and adults. Scores on the test are used to identify possible organic brain damage and the degree maturation of the nervous system. The Bender Gestalt was developed by psychiatrist Lauretta Bender in the late nineteenth century. Q253. in which of the following condition -snow banking is seen ;; 1leprotic uveitis;; 2candidiasis;; 3para planitis;; 4fuchs iridocyclitis;; ANS-PARS PLANITIS REF KANSKI 6TH PG456 SNOW BANKING SEEN IN INTERMEDIATE UVETIS (pars planitis) In early cases cells are seen in anterior vitreous. Later they assume a sheet-like configuration & small gelatinous exudates ( 'snowballs' or 'cotton-balls' ) appear.

Posterior vitreous detachment is common.. The hallmark is the presence of a grey-white fibro vascularplaque involving the inferior pars plana. The plaque which is referred to as 'snowbanking.

vinay.
Guest

Back to top
Posted: Wed Aug 04, 2010 1:03 pm Post subject:

Q253. most important toll like receptors in human is ;; 1tlr-2 2tlr -4;; 3tlr-5;; 4tlr-9;; Q254. which of the following is not a component of moros reflex;; 1extension of elbow;; 2flexion of shoulder;; 3opening of hand;; 4abduction of shoulder;; Q255. leucocyte adhesion def type 2 diseae is due to a defect in ;; 1beta chain of cd11/cd18 integrins;; 2fucosyl transferase;; 3mpo-hp2 system;; 4protein involved in organelle membrane docking and fusion;; ans-fucosyl transferase REF-ROBBINS 8TH pg50 Leukocyte adhesion deficiency type II is extremely rare; only a handful cases have been reported and most of them are of Middle Eastern decent. It is a defect in the expression of ligands for selectins due to lack of enzymes required for expression of selectin ligands. Patients have leukocytosis, recurrent infections (more prominent in infants and toddlers), and severe growth and mental retardation. This disease is a defect in fucose metabolism (lack of fucosylation of the carbohydrate selectin ligands) that results in failure to express the ligand for E and P selectin, sialyl Lewis-X (CD15s) expressed on leukocytes and endothelial cells. The patients are unable to fucosylate other glycoproteins, including the H blood group polysaccharide. fucosyl transferase

Leukocyte adhesion deficiency type I (LAD I) is a failure to express CD18, which composes the common 2 subunit of LFA1 family (2 integrins). CD11a/CD18 (LFA-1) expressed on lymphocytes is known to play an important role in lymphocyte trafficking (adhesion to vascular endothelium), as well as interactions to antigen presenting cells (APC). LFA-1 also plays a role of cytotoxic killing by T cells. Another member of this family is CD11bCD18 (MacA or CR3) and CD11cCD18(CR4). These 2 members mediate leukocyte adhesions to endothelial cells but they also serve as receptors for iC3b (inactivated C3b). These patients succumb to life-

threatening infection, usually within 2 years of life in severe cases of leukocyte adhesion deficiency I (<1% expression of CD18). In milder forms of leukocyte adhesion deficiency I (1-30% expression of CD8), patients may survive to adulthood.

Leukocyte adhesion deficiency type II is extremely rare; only a handful cases have been reported and most of them are of Middle Eastern decent. It is a defect in the expression of ligands for selectins due to lack of enzymes required for expression of selectin ligands. Patients have leukocytosis, recurrent infections (more prominent in infants and toddlers), and severe growth and mental retardation. This disease is a defect in fucose metabolism (lack of fucosylation of the carbohydrate selectin ligands) that results in failure to express the ligand for E and P selectin, sialyl Lewis-X (CD15s) expressed on leukocytes and endothelial cells. The patients are unable to fucosylate other glycoproteins, including the H blood group polysaccharide. Patients with leukocyte adhesion deficiency II manifest the Bombay phenotype (ie, negative for O and H blood group antigens with potential production of anti-H antibody). The immunoglobulin M (IgM) and immunoglobulin G (IgG) heavy chains are also not fucosylated. However, IgM and IgG serum levels are within the reference range in patients with leukocyte adhesion deficiency II. Leukocyte adhesion deficiency II may be classified as one of the congenital disorders of glycosylation (CDG), a rapidly expanding group of metabolic syndromes with a wide symptomatology and severity. All stem from dysfunctional N -glycosylation of proteins. Currently, 18 subtypes have been reported: 12 are type I (dysfunctional lipid-linked oligosaccharide precursor synthesis), and 6 are type II (dysfunctional trimming/processing of the protein-bound oligosaccharide), including leukocyte adhesion deficiency II (CDG-IIc).

vinay.
Guest

Back to top
Posted: Mon Aug 09, 2010 11:46 am Post subject:

Q254. A pregnant woman positive for hepatitis B surface antigen, anticore antibody IgM, e-antigen, and hepatitis B DNA. After delivery, appropriate treatment of the baby would be: A. Checking LFT in a week before initiating treatment B. Administration of I/V serum immune globulin C. Administration of hepatitis B immune globulin followed by hepatitis B vaccination D. Administration of hepatitis B vaccination alone

aayush.
Guest

Back to top
Posted: Tue Aug 17, 2010 12:47 pm Post subject:

254. Charecteristic radiological feature of transient tachypnoea of newborn is ;;

1. reticulogranular appearance. 2. low volume lungs. 3. prominent horizontalfissure. 4. air bronchogram. The characteristic findings include prominent perihilar streaking, which correlates with the engorgement of the lymphatic system with retained lung fluid, and fluid in the fissures. Small pleural effusions may be seen. Patchy infiltrates have also been described. 255. X-ray films are least sensitive to which coloured light;; 1. violet. 2. blue. 3. yellow. 4. red. 256. The most common finding on chest radiograph done early during the course of p.jirovici pneumonia is ;; 1. normal film. 2. ground glass appearance. 3. perihilar infiltrates. 4. pleural effusion. 257. On MRI-the differential dignosis of spinal cord oedema is; 1. myelodysplasia. 2. myelomalacia. 3. myeloschisis. 4. cord tumors. 258. mould therapy is used in the treatment of ;; 1. pelvic cancer. 2. lung cancer. 3. prostate cancer. 4. skin cancer

aayush.
Guest

Back to top
Posted: Tue Aug 17, 2010 12:52 pm Post subject:

Q:: Hot water Bag for abdomen colic works by inhibiting:: a. cold receptors on skin b. Adrenergic receptors c. Cholinergic receptors D. Peritoneal sensory supply Q:: Transfer of carbon diaoxide is said to be diffusion limited. because a. the binding of co to hb is of high avidity b. the diffusion of co across blood gas barrier is slow

c. alveoli basement memb is less permeable to co d. the partial pressure of co rises rapidly in blood the moment it is exposed to it. Q:: corneal tatooing is C/I a. traumatic loss of iris b. adherant leukoma c. iris coloboma d. leucoma Q:: Essential Atrophy of choroid is due to inborn error of metabolism of:: a. cysTine b. lySine c. aRginine d. oRnithine e. Cysteine

aamir.
Guest

Back to top
Posted: Tue Aug 17, 2010 12:53 pm Post subject:

Q259. preg induced changes accountnfor about 9 to 10 kgnof wt gain in normal cond.wt gain blood vol is a. 3.4 kg b. 0.9 kg c. 1.6 kg d. 0.45 kg Q260: Hot water Bag for abdomen colic works by inhibiting:: a. cold receptors on skin b. Adrenergic receptors c. Cholinergic receptors D. Peritoneal sensory supply Q261: Transfer of carbon diaoxide is said to be diffusion limited. because a. the binding of co to hb is of high avidity b. the diffusion of co across blood gas barrier is slow c. alveoli basement memb is less permeable to co d. the partial pressure of co rises rapidly in blood the moment it is exposed to it. Q262: Intravascular hemolysis in hemolytic anemia is associated decreased serum haptoglobulin which condition maskes this a. pregnancy b. parencymal liver disease c. bile duct obs d. malnutrition Q263. a pt wth hereditary blood clotting problems presents wth pain in back of her knee.ateriogram reveals bld clot in popliteal artery at its prox end.which of the arteries allow blood to reach foot?? a. post tibial b. peroneal

c. lat circumflex femoral d. superior medial genicular artery Q264. corneal tatooing is C/I a. traumatic loss of iris b. adherant leukoma c. iris coloboma d. leucoma Q265. Essential Atrophy of choroid is due to inborn error of metabolism of:: a. cysTine b. lySine c. aRginine d. oRnithine e. Cysteine

aayush.
Guest

Back to top
Posted: Wed Aug 18, 2010 12:49 pm Post subject:

q 259. preg induced changes account for about 9 to 10 kg of wt gain in normal cond. wt gain in blood vol is a.3.4 kg b.0.9 kg c.1.6 kg d.0.45 kg ans-c)1.6 kg ref.dutta5th, williams23rd # The average weight gain in pregnancy is 10-12 kg. # This increase occurs mainly in the second and third trimesters at a rate of 350-400 gm/week. # Six kg of the average 11 kg weight gain is composed of maternal tissues (breast 0.5 kg, fat-3.5kg, blood-1.5 kg and uterine tissue-1kg) and 5 kg of foetus, placenta and amniotic fluid. # Of this 11 kg, 7 kg are water, 3 kg fat and 1 kg protein Blood Volume * The total blood volume increases steadily from early pregnancy to reach a maximum of 35-45% above the non-pregnant level at 32 weeks. * Plasma volume increases by 40% whereas red cell mass increases by 20% leading to haemodilution (Physiological anaemia)

Q:260: Hot water Bag for abdomen colic works by inhibiting::

a. cold receptors on skin b. Adrenergic receptors c. Cholinergic receptors D. Peritoneal sensory supply ans- REF.GANONG 21ST PG 143 ans is -ADRENERGIC receptors visceral pain is carried by UNMYELINATED C adrenergic fibres. it works by GATE CONTROL HYPOTHESIS SEEN IN COUNTER IRRTANTS USED ON SKIN Q:262: Intravascular hemolysis in hemolytic anemia is associated decreased serum haptoglobulin which condition maskes this a. pregnancy b. parencymal liver disease c. bile duct obs d. malnutrition ANS-c) bile duct obstruction ref-robbins 8th pg 642, Cecilmedicine23 Haptoglobin is a protein that in humans is encoded by the HP gene. In blood plasma, haptoglobin binds free hemoglobin (Hb) released from erythrocytes with high affinity and thereby inhibits its oxidative activity. The haptoglobin-hemoglobin complex will then be removed by the reticuloendothelial system (mostly the spleen). In clinical settings, the haptoglobulin assay is used to screen for and monitor intravascular hemolytic anemia. In intravascular hemolysis free hemoglobin will be released into circulation and hence haptoglobin will bind the Hb. This causes a decline in Hp levels. Conversely, in extravascular hemolysis the reticuloendothelial system, especially splenic monocytes, phagocytose the erythrocytes and hemoglobin is not released into circulation and hence haptoglobin levels are normal. Higher-than-normal levels may be due to: * Acute rheumatic disease * Biliary obstruction * Peptic ulcer * Ulcerative colitis * Other inflammatory conditions Lower-than-normal levels may be due to: * Chronic liver disease * Drug-induced immune hemolytic anemia * Erythroblastosis fetalis * Hematoma * Hemolytic anemia due to G6PD deficiency * Idiopathic autoimmune hemolytic anemia * Immune hemolytic anemia * Primary liver disease * Transfusion reaction Q. 264. corneal tatooing is C/I a. traumatic loss of iris

b. adherant leukoma c. iris coloboma d. leucoma ans-b)adherant leucoma ref-internet The leading reason for corneal tattooing is to cosmetically alter the appearance of the eye. Usually, the need for this alteration stems from corneal opacities. Corneal opacities, the scarring of the cornea which creates an opaque or semi-transparent area on the eye, can be caused by leucoma, keratitis, and cataracts. These opacities can be cosmetically disruptive for patients in their everyday lives. Tattooing the cornea can cosmetically alter the discoloration and therefore blend the opacity to a normal eye color. Occasionally, corneal tattooing can be performed in cases of albinism, aniridia, coloboma, iridodialysis, keratoconus, or diffused nebulae of the cornea. Corneal tattooing is also occasionally performed on eyes that still have vision in order to reduce the symptomatic glare associated with large iridectomies or traumatic iris loss Suggested contraindications to corneal tattooing procedures are adherent leucoma, keratectasia, anterior staphyloma, neurotrophic cornea. Q. 265. Essential Atrophy of choroid is due to inborn error of metabolism of:: a. cysTine b. lySine c. aRginine d. oRnithine e. Cysteine ans-ornithine ref-KANSKI 6TH pg 692 GYRATE ATROPHY (GA) of the choroid and retina is a rare, autosomal recessive disease causing progressive chorioretinal degeneration resulting in blindness. It is caused by a deficiency of ornithine aminotransferase (OAT)

divakar.
Guest

Back to top
Posted: Wed Aug 18, 2010 12:52 pm Post subject:

:: In artificial rupture of membranes fetal heart rate shows:: a. acceleration b. deceleration c. variable deceleration d. any of the above. ans-acceleration or variable deceleration RCOG STUDY(1980)-most frequently observed patterns were short-lived FHR acceleration and increased band-width. Short-lived bradycardia was recorded in only few patients after amniotomy. The majority of the alterations observed were not indicative of fetal distress as a direct result of artificial membrane rupture.

pub med study(1977)-This study reports the effects of spontaneous rupture of membranes and artificial rupture of membranes on fetal heart rate patterns during labour in 87 normal and 25 complicated pregnancies. The incidence of early deceleration patterns following membrane rupture was 6.25 per cent during the first 15 minutes and 1.78 per cent after 45 minutes, regardless of whether the pregnancy was normal or complicated. It is concluded that rupture of membranes does not significantly increase the incidence of early deceleration patterns. However, the occurrence of other heart rate alterations such as late decelerations, loss of beat-to-beat variability, changes in base line and tachycardia, need to be investigated further in larger collaborative studies. Q. Splenic macrophages in gauchers disease differ from those in ceroid histiocytosis by staining positive for:: a. lipids b. phospholipids c. acid fast stain d. iron ans-lipids ref-ROBBINS 8TH pg 153, Gauchers disease Autosomal recessive disease, due to accumulation of glucocerebroside (a sphingolipid) in reticuloendothelial cells in liver, spleen and bone marrow, due to a defect in lysosomal betaglucocerebrosidase Increased risk (14x) of hematologic malignancies and 4x for other malignancies Type 1 - chronic nonneuronopathic form - often completely asymptomatic; disease discovered incidentally; does not involve the nervous system, high prevalence among Ashkenazi Jews (1/12 are carriers) Type 2 - fatal neurodegenerative disorder of infancy, similar to Tay-Sachs disease Type 3 - slowly progressive neurologic disease with survival into adulthood Treatment: glucocerebrosidase (enzyme replacement therapy) Gross: massively enlarged spleens up to 10 kg m in diameter and have one or more dark eccentrically placed nuclei. Periodic Acid Schiff (PAS) Staining is usually intensely positive. With the electro microscope, the fibrillary cytoplasm can be resolved as elongated distended lysosomes, containing the stored lipid in Stacks of BilayerMicro images: Gaucher cells rarely appear vacuolated but instead have a fibrillary type of cytoplasm likened to crumpled tissue paper. Gaucher cells are often enlarged, sometimes up to 100 Positive stains: iron, PAS (but weak) Negative stains: phospholipids stains, acid-fast stains DD: chronic myelogenous leukemia (similar looking cells) Ceroid-containing histiocytes are described for the first time in the spleen and marrow of a patient with ITP. Ceroid results from the oxidation and polymerization of unsaturated lipids and may appear in histiocytes in a variety of diseases. With the Wright or Giemsa methods, ceroid granules are stained seablue, and ceroid-containing macrophages appear as "sea-blue histiocytes." This sea-blue color is not

pathognomonic for ceroid. Therefore, the finding of seablue histiocytes in the bone marrow should be followed by histochemical studies to confirm the presence of ceroid. The spleen from a case of the recently described "syndrome of the sea-blue histiocyte" showed numerous ceroid-containing macrophages. In view of the lack of specificity of the sea-blue histiocyte, it is suggested that the syndrome be renamed "idiopathic ceroid histiocytosis of spleen and marrow."

divakar.
Guest

Back to top
Posted: Wed Aug 18, 2010 12:58 pm Post subject:

Q.268:: Which of the following is not seen in interphotoreceptor matrix:: (MK) a. Metalloproteinase b. Sialoprotein Associated with Rods and Cones c. Tissue Inhibitors of Metaloproteinases (TIMP) d. Memicane.....ans!! a. Glycoproteins B. Metalloproteinase....ans!! c. Proteoglycans d. Glycolipids For the interphotoreceptor matrix, the answer is memicane as memicane if given in the option is not at all found in the interphotoreceptor matrix but if the options of Across are taken into account, then the concentration of metalloprotinase's is least in the retina with respect to the other 3 so both the books are right. Q269:: all show miliary mottling except:: a. HMD b. Klebsiella C. Mitral Stenosis d. Sarcoidosis e. Stap. Pneumonia D/d for miliary mottling: Miliary tuberculosis Varicella pneumonia(and other viral pneumonia's) Acute hypersensitivity pneumonitis Fungal pneumonia Thyroid cancer metastases Renal cell carcinoma metastases Staphylococcal pneumonia Tropical eosinophilia Aspiration pnemonia Pnemocystis carinni pneumonia Sarcoidosis Lymphoma Leukemia

Methotrexate lung disease Uremic lung disease Pulmonary alveolar proteinosis Pulmonary alveolar microlithiasis Idiopathic pulmonary hemosidirosis Ref : Differential diagnosis in pediatrics by Suraj Gupte.4/e. Q270:: chalky white optic disc on fundus A/E:: a. syhilis b. lebers c. Post papilledema Optic neuritis (ans:: across) d. trauma to optic nerve (ans:: MK) Across is right in c/o chalky white optic disc on ophthalmoscopy. The reason is : In conditions with primary optic atrophy (eg, pituitary tumor, optic nerve tumor, traumatic optic neuropathy, multiple sclerosis), optic nerve fibers degenerate in an orderly manner and are replaced by columns of glial cells without alteration in the architecture of the optic nerve head. The disc is chalky white and sharply demarcated, and the retinal vessels are normal. Lamina cribrosa is well defined.

Guest

Back to top

Posted: Wed Aug 18, 2010 12:59 pm

Post subject:

271. the chief surgical risk to which pt wth polycyathemia vera are exposed is that due to a. anemic disturb b. hge c. infection d. renal dysfunc 272. posner schlossmann synd typically affects young adults..40% of them positive for a. hla bw 54 b. hla b 27 c. hla b 5 d. hla dr4 273. which of the following will decrease the oxygen consumption of resp muscles a. a dec in lung compliance b. a dec in airway resistance c. a dec in diff capacity of lung d. a inc in rate of resp

aayush.
Guest

Back to top
Posted: Thu Aug 19, 2010 1:12 pm Post subject:

Glaucomatocyclitic crisis is a condition with self-limited recurrent episodes of markedly elevated intraocular pressure (IOP) with mild idiopathic anterior chamber inflammation. It is most often classified as secondary inflammatory glaucoma. In 1948, Posner and Schlossman first recognized glaucomatocyclitic crisis and described the features of this syndrome.1 For this reason, the entity is often termed Posner-Schlossman syndrome (PSS). Posner and Schlossman identified the following features2 : Recurrent episodes of mild cyclitis Uniocular involvement Duration of attack varying from a few hours to several weeks Signs of a slight decrease in vision, elevated IOP with open angles, corneal edema with a few keratic precipitates, heterochromia with anisocoria, and a large pupil in the affected eye Normal visual fields Normal optic disc Normal IOP and outflow facility, and all provocative tests normal between episodes Associations with immunogenetic factors also exist; in one study, the presence of human leukocyte antigen Bw54 (HLA-Bw54) was found in 41% of patients Differential Diagnoses Anisocoria Glaucoma, Uveitic Glaucoma, Angle Closure, Acute Ocular Hypertension

Glaucoma, Angle Closure, Chronic Glaucoma, Primary Open Angle Glaucoma, Unilateral

aayan.
Guest

Back to top
Posted: Thu Aug 19, 2010 1:13 pm Post subject:

Q269:: all show miliary mottling except:: a. HMD b. Klebsiella C. Mitral Stenosis d. Sarcoidosis e. Stap. Pneumonia D/d for miliary mottling: Ref: harrison/ Grainger bacterial:: Miliary tuberculosis, brucellosis, broncho pneumonia fungal:: histoplasma, coccidiodomy, blastomycosis viral:: Varicella pneumonia(and other viral pneumonia's) allergic:: Loefflers, tropical eosinophilia, drug reaction neoplastic:: secondaries, leukemia, lymhoma cardiac:: mitral stenosis, pulmonary edema, multi pulm infract Pneumoconiosis Misc:: sarcoiosis, RA, Hemosiderosis, HMD, Interstitial Pulm fibrosis ans: staph. pneumonia and klebsiella, b+e

aayan.
Guest

Back to top
Posted: Thu Aug 19, 2010 1:14 pm Post subject:

Q 275:: In Oguchi s disease :: a. ERG is abnormal

b. associated with vita A def c. visual acuity is under normal photopic conditions d. abnormality of visual purple e. cellular differentiation of retina is normal Q:: 276 Most severe Papillodema will be caused by:: a. Pituitary tumor b. Frontal lobe tumor c. post cranial fossa tumor d. medulloblastoma

aayan.
Guest

Back to top
Posted: Fri Aug 20, 2010 12:41 pm Post subject:

Q.276:: Bispectral Index is used to :: a. measure Fio2 b. predict difficult intubation c. measure obesity d. measure depth of anesthesia (ans) Bispectral index (BIS) is one of several recently developed technologies which purport to monitor depth of anesthesia. BIS monitors can replace or supplement Guedel's classification system for determining depth of anesthesia. Titrating anesthetic agents to a specific bispectral index during general anesthesia in adults (and children over 1 year old) allows the anesthetist to adjust the amount of anesthetic agent to the needs of the patient, possibly resulting in a more rapid emergence from anesthesia. Use of the BIS monitor may reduce the incidence of intraoperative awareness in high risk procedures or patients and may also have a role in predicting recovery from severe brain injury.[citation needed] The introduction of BIS to the intensive care environment allows physicians and nurses to titrate sedative drugs safely, and to monitor the patient (together with measurement of intracranial pressure) during therapeutic burst suppression bispectral index is a statistically based, empirically derived complex parameter. It is a weighted sum of electroencephalographic subparameters, including a time domain, frequency domain, and high order spectral subparameters. The BIS monitor provides a single dimensionless number, which ranges from 0 (equivalent to EEG silence)to 100 (equivalent to fully awake and alert). A BIS value between 40 and 60 indicates an appropriate level for general anesthesia, as recommended by the manufacturer. Q 277:: Prognosis in a newborn with congenital diaphragmatic hernia is most closely related to :: a. size of hernial defect b. extent of herniation c. degree of pulmonary hypoplasia d. associated congenital anomalies ANSWER degree of pulm hypoplasia Ventilatory predictors of pulmonary hypoplasia in congenital diaphragmatic hernia, confirmed by morphologic assessment. Bohn D, Tamura M, Perrin D, Barker G, Rabinovitch M.

Abstract We carried out a prospective study in 66 infants with congenital diaphragmatic hernia within the first 6 hours of life to determine whether outcome is related to the degree of underlying pulmonary hypoplasia, as predicted by preoperative PaCO2, when correlated with an index of ventilation (VI = mean airway pressure X respiratory rate) and confirmed by postmortem analysis of the lung. Those infants with PaCO2 greater than 40 mm Hg before surgery had a 77% mortality; when PaCO2 reduction could be achieved only with VI greater than 1000, the mortality was still greater than 50%. After repair, however, the ability to hyperventilate to PaCO2 less than 40 mm Hg proved to be an important determinant of survival; only one of 31 infants in this group died, whereas only two of 27 infants with PaCO2 greater than 40 mm Hg survived. In 16 infants with PaCO2 greater than 40 mm Hg despite hyperventilation, high-frequency oscillatory ventilation was started. This resulted in a rapid fall in PaCO2, but 14 of the 16 infants had only temporary improvement in oxygenation, and died. In five of the infants who died, alveolar number was assessed by postmortem morphometric analysis; there was a severe reduction to less than 10% of published normal neonatal values. Pulmonary vascular changes of increased muscularization were less remarkable than those observed in infants with persistent pulmonary hypertension. Our findings suggest that the degree of pulmonary hypoplasia (which would not be influenced by surgical repair), rather than the pulmonary vascular abnormality, mainly determines survival. Consideration could therefore be given to an initial nonsurgical approach to congenital diaphragmatic hernia, with the expectation that pulmonary function might improve and pulmonary vascular resistance decrease.

Q 278. Von Brun's nest is seen in ? A) Normal urothelium B) Transitional cell carcinoma C) Squamous cell carcinoma D) Adeno carcinoma Von Brunns nest refers to nests of transitional cells within the lamina propria of the urinary bladder that arise from invagination of the overlying urothelium. It is the most common reactive proliferative change within the urothelium and is common enough to be considered a normal feature of the bladder mucosa by some investigators. Cystitis glandularis is a common change that occurs most often in the trigone. It is an inflammatory process that arises within and merges imperceptibility with von Brunns nests. With cystic dilatation of the glands, a progressive flattening of the lining cells occurs, and the lesion takes on the form of cystitis cystica. 3, 4 Cystitis glandularis is microscopic or occurs as small mucosal nodules in most occasions but it may occur as polypoid mass that suggests a neoplastic process macroscopically and endoscopically.

Q 279. floating teeth is seen in{delhi pg 2008} a. hyperparathyroidism b. hypoparathyroidism c. struge weber d. vhl syndrome Q280. ALL of the following are features of eisenmergers syndrome, except 1. triangular heart. 2. large main and central pulmonary aretry. 3. large peripheral vessels in the lung parenchyma. 4. ipulmonary arterial calcification

Q281. mechanism of action of pertusis toxin a. adp ribosylation b. adenyl cyclase c. open ca channel d. dec affinity of g protein Q282. which of the following skin lesion has an 'almost pathognomic ' assoc with hodgkins disease? a. acanthosis nigricans b. dermatomyositis c. gen erythroderma d. ichthyosis Q283. which of the following condition least likely assoc with platelet disorder? a. epistaxis b. retroperitoneal bleed c. ecchymoses d. petechiae Q284.: risk factors for local recurrence in extremity soft tissue sarcoma include all except:: a. High histologic grade b. Deep location c. Fibrosarcoma histopath d. Previous recurrence e. positive microscopic margins Q285. Lemierre's disease MC cause :: a. Staph aureus b. Fusobacterium Necrophorum c. H. Influenza d. Beta hemolytic Streptococci Lemierre's syndrome (or Lemierre's disease, also known as postanginal sepsis and human necrobacillosis) is a form of thrombophlebitis usually caused by the bacterium Fusobacterium necrophorum, and occasionally by other members of the genus Fusobacterium (F. nucleatum, F. mortiferum and F. varium etc.) and usually affects young, healthy adults. Lemierre's syndrome develops most often after a sore throat caused by some bacterium of the Streptococcus genus, has created a peritonsillar abscess, a crater filled with pus and bacteria near the tonsils. Deep in the abscess, anaerobic bacteria (microbes that do not require oxygen) like Fusobacterium necrophorum can flourish. The bacteria penetrate from the abscess into the neighboring jugular vein in the neck and there they cause an infected clot (thrombosis) to form, from which bacteria are seeded throughout the body by the bloodstream (bacteremia). Pieces of the infected clot break off and travel to the lungs as emboli blocking branches of the pulmonary artery bringing the heart's blood to the lungs. This causes shortness of breath, chest pain and severe pneumonia. Fusobacteria are normal inhabitants of the oropharyngeal flora answer fusobacteria Q286:: Low dose spiral ct is evolving as a screening tool for a. ca lung b. ca breast c. ca prostate d. ca oesophagus

Q287:: Skeletal metastases are rare in a .epithelial ovarian cancers and sarcomas b. malanomas and rcc c. lung cancers and breast cancer d. GI cancers and urothelial tumors Metastases involve bone by means of 3 main mechanisms: (1) direct extension, (2) retrograde venous flow, and (3) seeding with tumor emboli via the blood circulation. Seeding occurs initially in the red marrow; this process accounts for the predominant distribution of metastatic lesions in the red marrowcontaining areas in adults. In contrast, bone metastases are usually widespread in children. Retrograde venous embolism is probably the major mechanism when spread from intra-abdominal cancer involves the vertebrae. Increased intraabdominal pressure causes blood to be diverted from the systemic caval system to the valveless vertebral venous plexus of Batson; this diversion allows the caudal and cranial flow of blood. Metastases from certain primary sites (eg, renal cell or thyroid carcinomas) are almost always osteolytic, whereas those from other sites (eg, prostatic carcinoma) are predominantly sclerotic Other malignancies associated with sclerotic metastases include breast carcinoma, colonic carcinoma, melanoma, bladder carcinoma, and soft-tissue sarcoma. The findings of sclerotic metastases virtually exclude an untreated renal tumor or hepatocellular carcinoma

Q288:: Bitter taste is mediated by action of a. guanyl cyclase b. g protein c. tyrosine kinase d. epithelial sodium ion channels Q289:: Pseudoexotropia is a feature of a. high hypermetropia b. high myopia c. epicanthus d. high astigmatism

divakar.
Guest

Back to top
Posted: Sat Aug 21, 2010 1:12 pm Post subject:

Q288:: Bitter taste is mediated by action of a. guanyl cyclase b. g protein-answer c. tyrosine kinase d. epithelial sodium ion channels Q290: Transferance-Counter transferance is important in

a. hypnosis b. psychoanalysis c. counselling d. behaviour therapy Q291:: ROKITANSKY system of Postmortem : a. organs are removed one by one b. in situ -dissection is done c. thoracic abnd cervical organs,abdominal organs and the urogenital system is removed in separate organ blocks d. thoracic abnd cervical organs,abdominal organs and the urogenital system is removed as one large block Rokitansky is associated with : Superior Mesenteric Artery Syndrome Mayer-Rokitansky-Kster-Hauser syndrome Rokitansky's diverticulum Rokitansky's triad (pulmonary stenosis) Rokitansky-Aschoff sinuses (in the gallbladder) Rokitansky-Cushing ulcer Rokitansky-Maude Abbott syndrome Von Rokitansky's syndrome Rokitansky nodule - teratomas Rokitansky's diverticulum Associated persons: Karl Freiherr von Rokitansky Description: A traction diverticulum resulting from shrinking of a bifurcator gland coalesced with the oesophagus. Q.292 Generally atropinization+beta blockade cause HR of:: a. 110-100/min b. 60-70/min c. 160-150/min d. 140-150/min Q. 293:: Benda's Sign ( A dolichocephalic head with bulging Occiput) is diagnostic of:: a. Communication Hyrdocephalus b. Otitic Hydrocephalus c. Obs at Aqueduct site d. Obs at cisterna magna answer c seen in dandy walker malfrm Patients with Dandy-Walker malformation present with developmental delay, enlarged head circumference, or signs and symptoms of hydrocephalus. The clinical presentation depends to some extent on the particular combination of developmental anomalies in each infant Macrocrania is usually the consequence of hydrocephalus but, in some patients, it results from massive enlargement of the posterior fossa by the posterior fossa cyst. In this situation, macrocrania precedes the development of hydrocephalus, giving the skull a characteristic

dolichocephalic shape, with bulging of the occiput. Dandy-Walker malformation as consisting of (1) complete or partial agenesis of the vermis, (2) cystic dilatation of the fourth ventricle, and (3) an enlarged posterior fossa with upward displacement of lateral sinuses, tentorium, and torcular herophili. This triad is typically found in association with supratentorial hydrocephalus, which should be considered a complication rather than part of the malformation complex. Common findings of Dandy-Walker malformation include the following: Enlarged posterior fossa Varying degrees of cerebellar and vermian hypoplasia or complete vermian absence Cyst formation in the posterior fossa Vermian remnant everted above the posterior fossa cyst Hypoplastic cerebellar hemispheres winged anterolaterally (outward) in front of the cyst Absence of the foramina of Luschka and Magendie Obstructive hydrocephalus secondary to cystic dilatation of the fourth ventricle Abnormally high position of the straight sinus, torcular herophili, and tentorium Sinus confluence and lateral sinuses elevated above the lambdoid sutures (high tentorial insertion, also called lambdoid-torcular inversion; Aqueductal obstruction, which may affect the need for supratentorial decompression (and is, therefore, an important component) If callosal agenesis coexists (20-25%), development of dilatation of the occipital horns ) Possible brainstem compression and hypoplasia; the degree of pontine hypoplasia is directly related to the degree of cerebellar hypoplasia Thinning and bulging of the occipital bones Q. 294: Spinal Tap is for:: a. Spinal Fluid Pressure b. Whether block exists c. Types and number of cells d. Protein and sugar levels e. all Q. 295: Csf Pressure in sitting :: a. 4-6 mm hg b. 7-10 mm hg c 12-14 mm hg

d. 18-20 mm hg e. 21-24 mm hg Q. 296:: Foster s sign :: a. spastic CP b. hypotonic CP c. Choreoathetotic CP d. Myaesthenia gravis
remi.
Guest

Q. 297: Which Of the following BEST predicts the PROGNOSIS of patients with recent diagnosis of cutaneous melanoma and no cl metastasis? a. Breslow Thickness b. Clark level c. Ulceration d. Gender e. Celtic Complexion

Breslow's depth is one of the cornerstones of the current AJCC TNM staging of malignant melanoma. A large study validated the i of Breslow's depth as one of the three most important prognostic factors in melanoma (the others being T stage and ulceration). depth also accurately predicted the risk for lymph node metastasis, with deeper tumors being more likely to involve the nodes.

The above studies showed that depth was a continuous variable correlating with prognosis. However, for staging purposes, the m AJCC guidelines use cutoffs of 1 mm, 2 mm, and 4 mm to divide patients into stages. Q. 298:: Which bone tumors are radiographic diagnosis and do not require biopsy? a. Chondrosarcoma b. Metastasis c. Giant cell tumor d. Osteochondroma Q. 299:: Which of the following concerning HTN induced Choroidopathy is false:: a. Elschnig spots characteristic b. Exudative RD may develop c. Hypertensive choroidopathy may be associated with chronic elevation in BP d. Hypertensive choroidopathy may be associated with acute elevation in BP Q. 300.OSHA GUIDLINESfor needle stick injury and other sharp injury requires all except. 1. record maintaining of all injuries 2. information and training. 3. pre-exposure prophylaxis. 4. post-exposure prophylaxis

Occupational Health and Safety Administration (OSHA) regulations and the Needlestick Safety and Prevention Act. In 1992 the OSHA issued the Bloodborne Pathogen regulations (29 CFR 1910). In November of 2000, the Needlestick Safety and Prevention Act was signed into law and took effect on April 18th, 2001. This new act required that OSHA revise the Bloodborne Pa standard to add the following components:

1.Provide new examples in the definition of engineering controls. 2.Require that exposure control plans reflect how employers implement a needleless/safety and needle stick prevention program. 3.Requires the employer to solicit input from direct patient care employees in the identification, evaluation and selection of safer

devices and work practices. 4.Require employers to establish and maintain a log of sharps related injuries. answer choice 3 pre exp prophylaxis OSHA Standards

This section highlights the OSHA standard requirements, preambles to final rules (background to final rules), directives (instructio compliance officers), and standard interpretations (official letters of interpretation of the standards) related to bloodborne pathog needlestick prevention.

:For the most part, these States adopt standards that are identical to Federal OSHA. However, some States have adopted differen standards applicable to this topic or may have different enforcement policies. Bloodborne pathogens. Revisions to as a result of the Needlestick Safety and Prevention Act: (i) requires the use of engineering and work practice controls to eliminate or minimize employee exposure to bloodborne pathoge Employers must keep a Sharps Injury Log for the recording of percutaneous injuries from contaminated sharps The Exposure Control Plan shall: Reflect changes in technology that eliminate or reduce exposure to bloodborne pathogens Document annually consideration and implementation of appropriate commercially available and effective safer medical devices d eliminate or minimize occupational exposure . Solicit input from non-managerial employees responsible for direct patient care, who are potentially exposed to injuries from cont sharps, in the identification, evaluation, and selection of effective engineering and work practice controls and shall document the in the Exposure Control Plan Appendix A, Hepatitis B vaccine declination (Mandatory). Q. 301. PHOTOCOAGULATION IS done with all, except 1. argon. 2. krypton. 3. diode. 4. aluminium garnet Q. 302. Ureteral duplication is commonly associated with a) VUR. b) PUJO. c) Ureterocele. d) Hypospadias.

Duplicated ureter is a congenital condition in which the ureteric bud, the embryological origin of the ureter, splits (or arises twice) in two ureters draining a single kidney. It is the most common renal abnormality, occurring in approximately 1% of the population additional ureter may result in a ureterocele, or an ectopic ureter. Pathophysiology:

Ureteral development begins in the human fetus around the 4th week of embryonic development. A ureteric bud, arising from the mesonephric (or Wolffian) duct, gives rise to the ureter, as well as other parts of the collective system. In the case of a duplicated the ureteric bud either splits or arises twice. In most cases, the kidney is divided into two parts, an upper and lower lobe, with so overlap due to intermingling of collecting tubules. However, in some cases the division is so complete as to give rise to two separ each with its own renal pelvis and ureter. Classification:

Ureteral duplication is either: Partial - i.e. the two ureters drain into the bladder via a single common ureter. Partial, or incomplete, ureteral duplication is rarely significant . or Complete - in which the two ureters drain separately. Complete ureteral duplication may result in one ureter opening normally int bladder, and the other being ectopic, ending in the vagina, the urethra or the vulval vestibule. These cases occur when the ureter arises twice (rather than splitting)

Prevalence:

Duplicated ureter is the most common renal abnormality, occurring in approximately 1% of the population [2]. Race: Duplicated u more common in Caucasians than in African-Americans. Sex: Duplicated ureter is more common in females. However, this may b the higher frequency of [urinary tract infection]s in females, leading to a higher rate of diagnosis of duplicated ureter. Clinical Presentation

Prenatally diagnosed hydronephrosis suggest post-natal follow-up examination. The strongest neo-natal presentation is urinary tr infection. A hydronephrotic kidney may present as a palpable abdominal mass in the newborn, and may suggest an ectopic ureter ureterocele. In older children, ureteral duplication may present as: Urinary tract infection - most commonly due to vesicoureteral reflux (flow of urine from the bladder into the ureter, rather than v versa). Urinary incontinence in females occurs in cases of ectopic ureter entering the vagina, urethra or vestibule.

Although, urterocoele is also seen in Duplicated ureter, but, since the most common presentation is UTI which itself is due to VUR duplicated ureter is mostly associated with VUR. Q. 303. Gonococcal stricture involves a) Bulbar urethra. b) Penile urethra. c) Prostatic urethra. d) External meatus.

trictures can be of any length from 5 mm to 10 cm. The commonest sites for gonococcal ones are: (1) the bulbous urethra, (2) at junction of the penis and scrotum, and (3) in the glans penis[md]in this order. Gonococcal strictures are the result of fibrosis in th spongiosum Q304. Which vitamin in milk is extremely sensitive to light & requires storage of milk in a container that protects against photo degradation a. riboflavin b. thiamin c. niacin d. vit.A

BACKGROUND & AIMS: Vitamins A and E are the most light-sensitive vitamins. Vitamin A is degraded by photolysis, while vitamin degrades by photo-oxidation. The composition of the parenteral nutrition mixture and the container could therefore influence deg during daylight administration. The aim of this study was therefore to determine the influence of fat emulsion and the type of bag photo-degradation of vitamins A and E in Parenteral Nutrition (PN) mixtures during simulated infusion in daylight. Q305. the following are associated with fibular hemimelia, except; 1. short tibia. 2. anteritr bowing of the leg. 3. equinovalgus deformity of the foot and ankle; 4. presence of polydactyly. Q306. Pathological findings encountered in the frozen shoulder, commonly include all the following, except; 1. osteopenia of the underlying bone. 2. acute inflammatory chenges in the capsule. 3. fibrosis between the capsule and rotator cuff. 4. calcim deposits in the rotator cuff. Q307. All are true about Testicular atrophy Except:: a. Leyding cell Hypoplasia b. lymphocyte infiltrate c. increased interstitial stroma d. normal sertoli cell population Q308. System for diabetic foot::

a. Robson s Classification b. Modified Wagener c. Mullikens And Glowacki d. Columbias.

Robsons Classification- it is for renal cell carcinoma--Two staging symptoms commonly used are the Robson classification and the American Joint Committee on Cancer (AJCC) staging The following table is taken from the Robson classification system. The prognosis generally depends on stage and there are four m stages (I to IV): Treatment

Localized Tumors - The standard management for stage I or II tumors and selected cases of stage III disease is radical nephrecto

Advanced Disease - Investigational therapy is the first-line treatment for metastatic disease as no immune approach or chemothe agent has shown significant anti-tumor activity. The prognosis is highly variable. Surgery has a limited role for patients with meta disease. One indication for nephrectomy is to alleviate pain or hemorrhage from a primary tumor. Interferon alpha and interleukin-2 produce regressions in 10 to 20% of patients but these are rarely durable. Wagner Classification of Diabetic Foot Ulcers Grade 0: No ulcer in a high risk foot. Grade 1: Superficial ulcer involving the full skin thickness but not underlying tissues. Grade 2: Deep ulcer, penetrating down to ligaments and muscle, but no bone involvement or abscess formation. Grade 3: Deep ulcer with cellulitis or abscess formation, often with osteomyelitis. Grade 4: Localized gangrene. Grade 5: Extensive gangrene involving the whole foot. University of Texas Wound Classification System of Diabetic Foot Ulcers Grade I-A: non-infected, non-ischemic superficial ulceration Grade I-B: infected, non-ischemic superficial ulceration Grade I-C: ischemic, non-infected superficial ulceration Grade I-D: ischemic and infected superficial ulceration

Grade II-A: non-infected, non-ischemic ulcer that penetrates to capsule or bone Grade II-B: infected, non-ischemic ulcer that penetrates to capsule or bone Grade II-C: ischemic, non-infected ulcer that penetrates to capsule or bone Grade II-D: ischemic and infected ulcer that penetrates to capsule or bone Grade III-A: non-infected, non-ischemic ulcer that penetrates to bone or a deep abscess Grade III-B: infected, non-ischemic ulcer that penetrates to bone or a deep abscess Grade III-C: ischemic, non-infected ulcer that penetrates to bone or a deep abscess Grade III-D: ischemic and infected ulcer that penetrates to bone or a deep abscess

In 1982, Mulliken and Glowacki described a clinically relevant classification of vascular anomalies in which these lesions were cate according to their endothelial cell characteristics. (1) This classification, which was adopted by the International Society for the St Vascular Anomalies in 1996, differentiates proliferating tumors (most of which are hemangiomas) from vascular malformations, w structural anomalies involving capillaries, venules, veins, lymphatic channels, and combinations of these structures. ---

* Tumors Juvenile hemangioma Rapidly involuting congenital hemangioma Noninvoluting congenital hemangioma Kaposiform hemangioendothelioma Tufted angioma * Vascular malformations High-flow Arteriovenous malformation Low-flow Venous malformation Lymphatic malformation Lymphatic-venous malformation Capillary (or venular) malformation Columbia Classification is a Algorithm of Suicide Assessment (C-CASA)

Q309. Which does not stabilize new formed vessels:: a. VEGF b. PDGF c. Ang1 d. TGF-beta Q310. Edavarone is used in Rx:: a. Cva b. Asthma c. Hiv d. Diabetes

Edaravone (3-methyl-1-phenyl-2-pyrazolin-5-one), a strong novel free radical scavenger, is used for treatment of patients with a infarction. Edaravone has preventive effects on myocardial injury following ischemia and reperfusion in patients with acute myoca infarction. Antioxidant actions of edaravone include:: enhancement of prostacyclin production, inhibition of lipoxygenase metabolism of arachidonic acid by trapping hydroxyl radicals, inhibition of alloxan-induced lipid peroxidation,

quenching of active oxygen, leading to protection of various cells, such as endothelial cells, against damage by reactive oxygen sp (ROS).

Q311. Lawrence Moon syn consists A/e:: a. RP b. AR c. Poast axial Polydactly d. Hypogonadism e. Spastic paraplegia

Posted: Tue Aug 24, 2010 12:13 pm

Post subject:

Q. 306. Pathological findings encountered in the frozen shoulder, commonly include all the following, except; 1. osteopenia of the underlying bone. 2. acute inflammatory chenges in the capsule. 3. fibrosis between the capsule and rotator cuff....hallmark 4. calcim deposits in the rotator cuff....ans

Adhesive capsulitis is characterized by pain and restricted motion of the shoulder. Usually this occurs in the absence of intrinsic s disease, including osteoarthritis and avascular necrosis.

It is, however, more common in patients who have had bursitis or tendinitis previously as well as patients with other systemic ill such as chronic pulmonary disease, ischemic heart disease, and diabetes mellitus. The etiology isnot clear, but adhesive capsulit

appears to develop in the setting of prolonged immobility. Reflex sympathetic dystrophy may also occur in the setting of adhesiv to years. On physical examination, the affected joint is tender to palpation, with a restricted range of motion.

capsulitis. Clinically, this disorder is more commonly seen in females over age 50. Pain and stiffness develop over the course of m

The gold standard for diagnosis is arthrography with limitation of the amount of injectable contrast to less than 15 mL. In most p adhesive capsulitis will regress spontaneously within 1 to 3 years. NSAIDs, glucocorticoid injections, physical therapy, and early mobilization of the arm are useful therapies. Q. 312:: Aldactone bodies seen in:: a. zona glomerulosa b. zona reticulosa c. zona fasiculata d. adrenal medulla Q. 313:: RCC worst prognosis:: a. vascular invasion b. hypercalcemia c. Hematuria d. > 5 cm size Temperature is the first sensation lost, followed by light touch, pain and then deep pressure. Sensory loss usually begins in the extremities, toes and fingertips Q. 314: x-ray appearance of HMD:: a. homogenous ground glass b. Reticulonodular c. normal d. air bronchogram Q. 315:: Moro Disappears at:: a. 3 months b. 9 months c. 7 months d. 6 months

Posted: Tue Aug 24, 2010 1:43 pm

Post subject:

Q. 311:: Aldactone bodies seen in::

a. zona glomerulosa b. zona reticulosa c. zona fasiculata d. adrenal medulla aldactone bodies a ka spirnolactone bodies

Spironolactone (aldactone) bodies: concentric lamellar formations in the adrenal cortices of patients treated with spironolactone.

Numerous spironolactone bodies have been detected in the zona glomerulosa cells of the adrenal cortex of a 36-year-old spirono treated woman whose non-tumorous right adrenal gland was removed surgically because of primary hyperaldosteronism.

Electron microscopy revealed spherical laminated whorls which consisted of a central core composed of an amorphous electron-d

material surrounded by numerous smooth-walled concentric membranes. Continuous with and deriving from the endoplasmic re

they were present in viable cells and were not associated with ultrastructural features indicating cellular injury. Cytoplasmic inclu

similar to spironolactone bodies can be detected in other organs after the administration of various compounds. Thus, they can b regarded as neither specific to spironolactone treatment nor exclusively inducible in the zona glomerulosa of the adrenal cortex.

Spironolactone bodies were observed in an adrenal cortical adenoma that was removed from a patient with primary hyperaldoste

(Conn syndrome) treated preoperatively with spironolactone. The electron microscopical evaluation of this adrenal cortical adeno shows origin of spironolactone bodies from whorls of endoplasmic reticulum in cells with the cytoplasmic features of those from t

glomerulosa of the adrenal gland. There was no evidence that the bodies were derived from mitochondria, which confirms recen who have received spironolactone, and the pharmacologic specificity of the bodies strongly suggests a direct mode of action by spironolactone on aldosterone production by cells of the adrenal zona glomerulosa. answer zona glomerulosa Q. 314: x-ray appearance of HMD:: a. homogenous ground glass b. Reticulonodular c. normal d. air bronchogram

ultrastructural findings in patients treated with spironolactone. These bodies have been described in the adrenal cortex only in p

On x-ray, the lungs may have a characteristic, but not pathognomonic appearance that includes a fine reticular granularity of th

parenchyma and air bronchograms, which are often more prominent early in the left lower lobe because of superimposition of th shadow ( Fig. 101-4 ). The initial roentgenogram is occasionally normal, with the typical pattern developing at 612 hr. Consider this variation often results in poor correlation between roentgenograms and the clinical course.

variation in films may be seen, depending on the phase of respiration and the use of CPAP or positive end-expiratory pressure (P

Q. 315:: Moro Disappears at:: a. 3 months b. 9 months c. 7 months d. 6 months (ans) Moro's till 4-5 months is considered normal. persistence beyond is a problem. 6 months is the best possible answer here. Q 316:: The following is of serious pathological significance in infants:: a. Loss of weight b. palpable left kidney c. palpable spleen d. deviation of trachea from midline

Q. 317:: Sneezing in a neonate :: a. nasal allergy b. syphilis c. normal d. Nasopharyngitis Q. 317:: Sneezing in a neonate :: a. nasal allergy b. syphilis c. normal d. Nasopharyngitis

Posted: Wed Aug 25, 2010 12:52 pm

Post subject:

Q312. RCC worst prognosis:: a. vascular invasion b. hypercalcemia c. Hematuria d. > 5 cm size its chance to combine our knowledge abt 1 of most imp topics rcc lil bit of copy paste from standard books for last min revisons .. Renal cell carcinoma accounts for approximately 3% of adult malignancies and 90-95% of neoplasms arising from the kidney.

While the optimal treatment strategy continues to evolve, three agents that target angiogenesis (sunitinib, bevacizumab, and pa and an mTOR-targeted therapy (temsirolimus) have been approved as front-line agents. These have largely replaced cytokines (immunotherapy) in treatment-naive patients The tissue of origin for renal cell carcinoma is the proximal renal tubular epithelium. Renal cancer occurs in both a sporadic

(nonhereditary) and a hereditary form, and both forms are associated with structural alterations of the short arm of chromosom formation. These genes are either tumor suppressors (VHL, TSC) or oncogenes (MET) syndromes associated with renal cell carcinoma are recognized: (1) von Hippel-Lindau (VHL) syndrome, (2) hereditary papillary renal carcinoma (HPRC), (3) familial renal oncocytoma (FRO) associated with Birt-Hogg-Dube syndrome (BHDS), and (4) hereditary renal carcinoma (HRC)

Genetic studies of the families at high risk for developing renal cancer led to the cloning of genes whose alteration results in tum

familial renal oncocytoma can develop bilateral, multifocal oncocytoma or oncocytic neoplasms in the kidney. Birt-Hogg-Dube sy

is a hereditary cutaneous syndrome. Patients with Birt-Hogg-Dube syndrome have a dominantly inherited predisposition to deve renal tumors, colonic polyps or tumors, and pulmonary cysts.

benign tumors of the hair follicle (ie, fibrofolliculomas), predominantly on the face, neck, and upper trunk, and are at risk of dev

median age at diagnosis was 64 years of age

Renal cell carcinoma has a male-to-female preponderance of 1.6:1.

classic triad of flank pain, hematuria, and flank mass is uncommon (10%) and is indicative of advanced disease Most common presentations Hematuria (40%) Flank pain (40%) Palpable mass in the flank or abdomen (25%) Cytokine release by tumor (eg, IL-6, erythropoietin, nitric oxide) causes these paraneoplastic conditions. risk factors

1.Cigarette smoking doubles the risk of renal cell carcinoma and contributes to as many as one third of all cases. The risk appea increase with the amount of cigarette smoking in a dose-dependent fashion. 2.Obesity is another risk factor, particularly in women; increasing body weight has a linear relationship with increasing risk. 3.Hypertension may be associated with an increased incidence of renal cell carcinoma. 4.Phenacetin-containing analgesia taken in large amounts may be associated with increased incidence of renal cell carcinoma. 5.There is an increased incidence of acquired cystic disease of the kidney in patients undergoing long-term renal dialysis; this predisposes to renal cell cancer. 6.Tuberous sclerosis

7.Renal transplantation: Acquired renal cystic disease of the native kidney also predisposes to renal cell cancer in renal transplan recipients. 8.VHL disease: This inherited disease is associated with renal cell carcino

The most common locations of metastasis are the lungs (more than 50%) and bones (33%), followed in frequency by the region nodes, liver, adrenal, and brain.

The average 5-year survival rate of persons with renal cell carcinoma is about 45% and as high as 70% in the absence of distan metastases. With renal vein invasion or extension into the perinephric fat, the figure is reduced to approximately 15% to 20%. frequency and similar outcome. ### designed to correlate stage at presentation with prognosis. The Robson staging system is as follows: Stage I - Tumor confined within capsule of kidney Stage II - Tumor invading perinephric fat but still contained within the Gerota fascia Stage III - Tumor invading the renal vein or inferior vena cava (A), or regional lymph-node involvement (B), or both (C) Stage IV - Tumor invading adjacent viscera (excluding ipsilateral adrenal) or distant metastases

Nephrectomy has been the treatment of choice, but partial nephrectomy to preserve renal function is being done with increasing

The Robson modification of the Flocks and Kadesky system is uncomplicated and is used commonly in clinical practice. This syste

treatment:::

Sorafenib (Nexavar), a small-molecule Raf kinase and vascular endothelial growth factor (VEGF) multireceptor kinase inhibitor, i approved by the U.S. Food and Drug Administration for the treatment of patients with advanced renal cell carcinoma. toxicities of sorafenib::

reversible skin rashes in 40% and hand-foot skin reaction in 30%. Diarrhea was reported in 43%, treatment-emergent hyperten

17%, and sensory neuropathic changes in 13%. Alopecia, oral mucositis, and hemorrhage also were reported more commonly o sorafenib arm. The incidence of treatment-emergent cardiac ischemia/infarction events was higher in the sorafenib group (2.9% compared with the placebo group (0.4%).

Sunitinib is another multikinase inhibitor approved by the FDA for the treatment of metastatic kidney cancer that has progressed trial of immunotherapy. (19%), and decline in cardiac ejection fraction (11%). Dermatitis occurred in 8%, and hypertension occurred in 5% of patients.

toxicities::Major toxicities (grade II or higher) include fatigue (38%), diarrhea (24%), nausea (19%), dyspepsia (16%), stomati

Temsirolimus inhibits mTOR (mammalian target of rapamycin), which is a serine/threonine kinase important in the regulation of growth and division

toxicities::::Common toxicities of temsirolimus include asthenia, rash, anemia, hypophosphatemia, and hyperlipidemia. Temsirolimus has FDA approval for the treatment of advanced renal cell carcinoma at a dose of 25 mg weekly IV until progressio

Everolimus (Afinitor) is a serine-threonine kinase inhibitor of mTOR, an important regulatory protein in cell signaling. Everolimus

approved by the US Food and Drug Administration in March 2009 for advanced renal cell carcinoma after failure of treatment wit sunitinib or sorafenib. ct:::

1.intravenous gemcitabine (600 mg/m2 on days 1, 8, and 15) with continuous infusion fluorouracil (150 mg/m2/d for 21 d in 28 in patients with metastatic renal cell cancer produced a partial response rate of 17%

2.Floxuridine (5-fluoro 2'-deoxyuridine [FUDR]), 5-fluorouracil (5-FU), and vinblastine, paclitaxel (Taxol), carboplatin, ifosfamide gemcitabine, and anthracycline (doxorubicin) all have been used. Floxuridine infusion has a mean response rate of 12%,

3.Renal cell carcinoma is refractory to most chemotherapeutic agents because of multidrug resistance mediated by p -glycoprote Normal renal proximal tubules and renal cell carcinoma both express high levels of p -glycoprotein. Calcium channel blockers or drugs that interfere with the function of p -glycoprotein can diminish resistance to vinblastine and anthracycline in human renal carcinoma cell lines. surgery:::

Radical nephrectomy, which remains the most commonly performed standard surgical procedure today for treatment of localized adrenal gland, with or without ipsilateral lymph node dissection

carcinoma, involves complete removal of the Gerota fascia and its contents, including a resection of kidney, perirenal fat, and ip

In patients with renal vein involvement without metastases, radical nephrectomy is performed with early ligation of the renal art

no manipulation of the renal vein. If the inferior vena cava is involved, then vascular control of the inferior vena cava is obtained actual invasion of the inferior vena caval wall have poor prognoses, despite aggressive surgical approaches. (1) the transperitoneal approach, (2) the flank approach, and (3) the thoracoabdominal approach.

above and below the tumor thrombus, and the thrombus is resected intact, with subsequent closure of the vena cava. Patients w

The thoracoabdominal approach offers the advantage of palpation of the ipsilateral lung cavity and mediastinum, as well as the a resect solitary pulmonary metastases.

2.Laparoscopic nephrectomy is a less invasive procedure, incurs less morbidity, and is associated with shorter recovery time and blood loss.

3.Palliative nephrectomy should be considered in patients with metastatic disease for alleviation of symptoms such as pain, hem

malaise, hypercalcemia, erythrocytosis, or hypertension. 4.Renal artery embolization with ethanol and gelatin sponge pledgets has been found effective for palliative treatment in patient are not candidates for surgery, or who refuse surgery.

rt::::: Radiation therapy may be considered as the primary therapy for palliation in patients whose clinical condition precludes surgery, because of extensive disease or poor overall condition. A dose of 4500 centigray (cGy) is delivered, with consideration of a boost up to 5500 cGy.

ref : emedicine robbins 8th ed harisson 17th is answer vascular invasion

couldnt find prognostic significnce of hypercalcemia in terms of percentages{High "corrected" serum calcium (>10 mg/dL) is a o prognostic indicators though} tumor 5cm :: comes in t1 stage which is wth good prognosis... Primary tumor (T) TX - Primary tumor cannot be assessed T0 - No evidence of primary tumor T1 - Tumor 7 cm or smaller in greatest dimension, limited to the kidney T2 - Tumor larger than 7 cm in greatest dimension, limited to the kidney T3 - Tumor extends into major veins or invades adrenal gland or perinephric tissues but not beyond the Gerota fascia T3a - Tumor invades adrenal gland or perinephric tissues but not beyond the Gerota fascia T3b - Tumor grossly extends into the renal vein(s) or vena cava below the diaphragm T3c - Tumor grossly extends into the renal vein(s) or vena cava above the diaphragm T4 - Tumor invading beyond the Gerota fascia Laterality does not affect the N classification NX - Regional lymph nodes cannot be assessed N0 - No regional lymph node metastasis N1 - Metastasis in a single regional lymph node N2 - Metastasis in more than 1 regional lymph nod

Posted: Wed Aug 25, 2010 1:02 pm

Post subject:

Q318. Erythroblastosis fetalis not involved:: a. Antigen Cw b. Antigen Cx c. Antigen Diego d. Anti Lewis answer anti lewis Other fetomaternal incompatibilities that can cause erythroblastosis fetalis involve the Kell, Duffy, Kidd, MNSs, Lutheran, Diego,

Ee, and Cc antigen systems, as well as other antigens. Incompatibilities of ABO blood types do not cause erythroblastosis fetalis. Q319. CYSTATIN C is elevated in:: a. CVD b. Alzheimer s c. chronic kidney disease d. liver diseases e. brain infraction ans-c)chronic renal disease,a)cardiovascular disease,b)alzheimers, e)brain infarction ref-Robbins 8th,wikepedia

CYSTATIN C-a protein encoded by the CST3 gene, is mainly used as a biomarker of kidney function. Recently, it has been studie specific type of protein deposition), such as Alzheimer's disease

role in predicting new-onset or deteriorating cardiovascular disease. It also seems to play a role in brain disorders involving amy

. Cystatin C levels are decreased in atherosclerotic (so-called 'hardening' of the arteries) and aneurysmal (saccular bulging) lesion aorta.

Since cystatin 3 also binds amyloid and reduces its aggregation and deposition, it is a potential target in Alzheimer's disease. A not all studies have confirmed this, the overall evidence is in favor of are role for CST3 as a susceptibility gene for Alzheimer's d Cystatin C levels have been reported to be higher in subjects with Alzheimer's disease Mutations in the cystatin 3 gene are responsible for the Icelandic type of hereditary cerebral amyloid angiopathy, a condition predisposing to intracerebral haemorrhage, stroke and dementia

A few studies have looked at the role of cystatin C or the CST3 gene in age-related macular degeneration. Cystatin C has also be investigated as a prognostic marker in several forms of cancer. Its role in pre-eclampsia remains to be confirmed.

For women, the average reference interval is 0.52 to 0.90 mg/L with a mean of 0.71 mg/L. For men, the average reference inter they increase again, especially beyond age 50

0.56 to 0.[snip] mg/L with a mean of 0.77 mg/L. The normal values decrease until the first year of life, remaining relatively stab

Diazymes Cystatin C assay is a convenient cost effective dual liquid stable PEIA method for this critically important emerging ma the early detection and diagnosis of renal disease.

Cystatin C in the sample binds to the specific anti-Cystatin C antibody, which is coated on latex particles, and causes agglutinatio

degree of the turbidity caused by agglutination is measured optically and is proportional to the amount of Cystatin C in the samp

Intended use The assay is for the quantitative in vitro determination of Cystatin C in human serum or plasma. There is a growing body of evid suggesting that Cystatin C can be used to detect kidney disease at earlier stages, which may help facilitate prevention efforts in elderly and those with diabetes, hypertension, or cardiovascular disease.

Cystatin C, a serum measure of renal function, is a stronger predictor of the risk of death and cardiovascular events in elderly pe than is creatinine.

Cystatin C or cystatin 3 (formerly gamma trace, post-gamma-globulin or neuroendocrine basic polypeptide),[1] a protein encode CST3 gene, is mainly used as a biomarker of kidney function. Recently, it has been studied for its role in predicting new-onset or deteriorating cardiovascular disease. It also seems to play a role in brain disorders involving amyloid (a specific type of protein deposition), such as Alzheimer's disease.

Q320. The average BP in a 1 year old child is:: a. 120/80 b. 120/90 c. 110/70 d. 95/55 e. 60/30

Normal blood pressure in children less than 1 year is usually 70/30 or greater. It can be higher or lower, depending on activity a circumstances.

When calculating a child's (greater than 1 year) normal blood pressure consider this formula: 90 + 2(age in years) is normal; 70 in years) is the lower end. For example: A 7-year-old child's systolic (top number) pressure would be between 70 + 2(7) =84 an should be from 84/39 to 104/59.

2(7) =104. The bottom number (diastolic) should be 30-45 less than the systolic number. So the healthy 7-year-old child's pres

normal blood pressure in adults is less than 120/80. 120-139/80-89 is indicative of pre-hypertension; 140-159/90-99 is Stage I hypertension; greater than 160/100 is Stage II hypertension.

. A newborn has a normal blood pressure of about 70 over 42, but when the infant reaches 1 month of age, the normal blood pr 94 over 56. answer d Q321. Cranipharyngioma contains? (1) HCG (2) AFP (3) BETA - GLOBULIN (4) CA 19.9 Q322. Picibanil all are true except:: a. ok-432 b. better suited for monocystic lesions c. for unilocular lesions d. useful for keloids A lyophilized preparation of a low-virulence strain (SU) of Streptococcus pyogenes (S. hemolyticus), inactivated by heating with G. It has been proposed as a noncytotoxic antineoplastic agent because of its immune system-stimulating activity

rises to approximately 85 over 45. At 6 months of age normal blood pressure is 93 over 55, rising again at 1 year to a normal av

Intralesional injection of OK-432 (lyophilized incubation mixture of group A Streptococcus pyogenes of human origin) is safe and

therapy for lymphangioma. The authors evaluated the mechanism of this therapy in 6 patients who had cystic lymphangioma. Th

intracystic fluid of the cystic lymphangioma was aspirated before and after (on days 1 and 4) the OK-432 therapy. Changes in ce populations and cytokine productions in each aspirated fluid were analyzed. White blood cells in the intracystic fluid increased m macrophages.

in number. Before OK-432 therapy, 96% of the intracystic white blood cells were lymphocytes, and the remaining were neutroph

predominantly macrocystic lymphangiomas had a successful outcome. CONCLUSIONS: OK-432 should be efficacious in the treat lymphangiomas. Our study design is well structured to clearly define the role of this treatment agent.

sclerotherapy using OK-432 is an effective and safe treatment modality for BCC branchial cleft cyst, especially for unilocular cyst Sclerosing of unilocular BCC with OK-432 should therefore be considered before surgical excision.

Q323. a child is brought to u with history of swallowing a coin, your next step is:: a. fibreoptic endoscopy b. rigid endoscopy c. laproscopy d. bacche key sath chor police khelo

In most cases,of a coin in the lower esophagus,intially x ray is done. the child may be watched closely over a day or two to see i not given hence we assume tat child is symptomatic

object passes on its own. If the child is in pain or has vomiting, this approach is not recommended. since option of xray n observ

Endoscopy is the definitive way to evaluate, diagnose and remove a foreign body in the stomach or esophagus. The child is place

anesthesia or sedation while a surgeon uses an endoscope. . A rigid endoscope is better for protecting the esophagus from dama

the object is removed but can only be advanced to the esophagus. A flexible endoscope can reach as far as the first part of the s intestines. A small clamp or net can be used to retrieve the object. Q324. a 25 yr old man had DM-1 for 10 yrs. now mild proteinuria. as per JNC-7 his target Bp::

a. <125/<75 b. <130/<80 c. <140/<90 d. <140/<80 ANS-b)<130/<80 ref. cmdt 2010,harrison 17th

Hypertension develops with progressive kidney involvement, and coronary and cerebral atherosclerosis seems to be accelerated. Approximately two-thirds of adult patients with diabetes have hypertension. Once diabetic nephropathy has progressed to the st hypertension, proteinuria, or early chronic kidney disease, glycemic control is not beneficial in influencing its course. In this

circumstance, antihypertensive medications, including ACE inhibitors, and restriction of dietary protein to 0.8 g/kg body weight p

are recommended. ACE inhibitors have been shown to protect against deterioration in kidney function in type 1 diabetic patients clinical nephropathy. This beneficial effect appears to be due to improved glomerular hemodynamics that cannot be explained on

antihypertensive action of these drugs. Captopril (25 mg three times daily) has shown a 50% reduction in the risk of the combin inhibitor therapy, an increment in serum creatinine greater than 2 mg/dL due to a rapid fall in intraglomerular pressureor the occurrence of persistent hyperkalemia (above 6 mEq/L) due to hyporeninemic hypoaldosteronismis an indication to stop this medication. Q. 325) D and L enantiomers occurs in which drug? a) amoxicillin b) diazepam c) verapamil d) losartan answer verapamil{ref :goodman pg590}

points of death, dialysis, and transplantation in type 1 subjects with diabetic nephropathy and clinical proteinuria. During initiatio

Verapamil (CALAN, ISOPTIN, VERELAN, COVERA-HS) is prescribed as a racemate. L-Verapamil is a more potent calcium channel than is D-verapamil. However, with oral therapy, the L-enantiomer undergoes more extensive first-pass hepatic metabolism. For concentrations of the L- and D-enantiomers are equivalent) than when administered orally Q326. polyhydramnios occurs in all except? a) Diabetes b) multiple pregnancy c) renal agenesis d) anencephaly answer c :: it results oligohydramnios Q327. commonest site of thyroglossal cyst is? a) supra hyoid b) sub hyoid c) infra thyroid d) foramen caecum Q328. LEMON ON MATCH STICK contour is seen in a) diabetes mellitus b) cushings syndrome c) myxoedema d) acromegaly answer :b:: central obesity manifests as this appearance Q329. DURCK granuloma is seen in ?

reason, a given concentration of verapamil prolongs the PR interval to a greater extent when administered intravenously (where

a) tuberculosis b) malaria c) hiv d) sarcoidosis Q330. Pancytopenia Gap for allogenic BMT:: a. 10-14 days b. 6-10 days c. 3-4 days d. 2-3 days

Posted: Wed Aug 25, 2010 1:03 pm

Post subject:

Q331. Machine Gun sneezing:: a. Pandas b. Vein of galen malformation c. Rhinoscleroma d. Psychiatric

chronic machine gun sneezing, which means the individual started to sneeze and continuously sneezes for weeks on end, there

arent complications associated with sneezing. Long term chronic, machine gun sneezing is very rare, and can complicate daily

interfering with the individuals ability to function normally. This type of sneezing can become very painful and can interfere with individuals ability to sleep, eat, or even speak.

Per an allergist interviewed for this story, the condition is extremely rare and "can be triggered by hundreds of causes, including allergies, sinus problems, or growths in the nasal passage. his girl now has a diagnosis called PANDAS (Pediatric Autoimmune Neuropsychiatric Disorder Associated with Streptococcus) Q332. Fiduciary markers:: a. gold b. silver c. Aluminium d. copper

Gold fiducial markers are tiny gold markers that are 2.0 to 3.0 mm in size. That's about 0.11 inch in size. The markers are inser the same method a prostate biopsy is performed. A transrectal ultrasound is performed in the urologist's office. The prostate is

measured. Depending on the patient, the prostate can be anesthetized with 1.0 % lidocaine. In general, three gold fiducial mark then inserted through a needle which is passed through the ultrasound probe. The ultrasound provides the urologist with the abi precisely place the markers in strategic positions. fiduciary marker or fiducial is an object used in the field of view of an imaging system which appears in the image produced, for point of reference or a measure. It may be either something placed into or on the imaging subject, or a mark or set of marks in reticle of an optical instrument answer ::gold:: used in imaging of prostate Q333. Cyberknife is:: a. frameless system b. shameless system c. with frame d. with rotating gantry

answer a

CyberKnife is a frameless robotic radiosurgery system invented by John R. Adler, a Stanford University Professor of Neurosurger Radiation Oncology, and Peter and Russell Schonberg of Schonberg Research Corporation. The two main elements of the CyberKnife are (1) the radiation produced from a small linear particle accelerator and (2) a robotic arm which allows the energy to be directed at any part of the body from any direction.

The original (and still utilized) method is called 6D or skull based tracking. The X-ray camera images are compared to a library o computer generated images of the patient anatomy. Digitally Reconstructed Radiographs (or DRR's) and a computer algorithm

determines what motion corrections have to be given to the robot because of patient movement. This imaging system allows the CyberKnife to deliver radiation with an accuracy of 0.5mm without using mechanical clamps attached to the patient's skull.[8] Th are made for the 3 translational motions (X,Y and Z) and three rotational motions

the image guided technique is referred to as frameless stereotactic radiosurgery. This method is referred to as 6D because corre

Small metal markers (fiducials) made out of gold for bio-compatibility and high density to give good contrast on X-ray images ar

surgically implanted in the patient. This is carried out by an interventional radiologist, or neurosurgeon. The placement of the fid out from each other it will not be possible to accurately deliver the radiation.

a critical step if the fiducial tracking is to be used. If the fiducials are too far from the location of the tumor, or are not sufficientl

One of the most widely known stereotactic radiosurgery systems is the Gamma Knife. The Gamma Knife was originally develope

Leksell, remains the gold standard method for delivery of stereotactic radiosurgery to the brain and is manufactured by Elekta. J

Adler, the inventor of the CyberKnife system spent time training with Lars Leksell in Stockholm at the Karolinska Institute in 198 GammaKnife system uses 201 Cobalt-60 sources located in a ring around a central treatment point ("isocenter"). The Gamma K

system is equipped with a series of 4 collimators of 4mm, 8mm, 12mm and 16mm diameter, and is capable of submillimeter acc

The Gamma Knife system does however require a head frame to be bolted onto the skull of the patient, and is only capable of tr

cranial lesions. As a result of frame placement, treatment with Gamma Knife does not require real time imaging capability as the does not allow movement during treatment. This is the reason that the Gamma Knife system is likely to be more accurate than C being more accurate than CyberKnife.

Knife. The Cyberknife Society and Accuray maintain that there are no peer-reviewed published papers that establish Gamma Kni

Q334. ALL are DATE RAPE drugs except:: a. Chloral hydrate b. Flunitrazepam c. Gamma hydroxybutyrate d. insulin e. Mickey finn Date rape drug refers to a drug that can be used to assist in the commission of a sexual assault, such as date rape. Drugs used victim's knowledge.

facilitate rape may have sedative, hypnotic, dissociative, and/or amnesiac effects, and can be added to a food or drink without t

The act of adding such substances to drinks is known as "drink spiking". The reasons for drink spiking range from personal amus or maliciousness to theft or (sexual) assault three most commonly used drugs for date rape are alcohol and two prescription-strength sleep aids. The two prescription drugs also known as gamma-hydroxybutyric acid, and benzodiazepines (such as flunitrazepam, also known as Rohypnol or "roofies");

an American 1997 study showed that alcohol still remains the drug most frequently implicated in substance-assisted sexual assa

Mickey Finn (or simply Mickey) is a slang term for a drink laced with a drug (especially chloral hydrate) given to someone withou knowledge in order to incapacitate them (see drink spiking). Serving someone a Mickey Finn is most commonly referred to as sli mickey, sometimes spelled "slipping a mickie".

answer insulin Q 335)Hutchinson-Gilford progeria syndrome (HGPS), false statement is? (1) Mutation in LMNA gene (2) Profound growth failure from 1'st yr of life (3) Death is mostly due to renal failure (4) Mutation is DOMINANT NEGATIVE MUTATION Q 336) Not a disease of affluent societies? (1) Obesity (2) Diverticular diseases (3) Dental caries (4) Diarrhea (5) Constipation

Posted: Thu Aug 26, 2010 1:30 pm

Post subject:

Q337. Which is most common lid ca? Q338. Which most common after chalazion?

A vaccine vial monitor, or VVM, is a circular indicator, printed directly on the vaccine vial label or affixed to the top of the vial or ampoule. The inner square of the VVM is made of heat-sensitive material that is initially light in colour and becomes darker when

exposed to heat over time. By comparing the colour of the square to the reference ring, health workers can determine the exten reference ring.

which the vaccine has been exposed to heat. The vaccine can be used as long as the colour of the inner square is lighter than th

Although developed as a heat-exposure indicator, VVM also contributes significantly to the reduction of vaccine freezing. VVM m and equipment that is a known source of freeze damage Vaccine vial monitors were introduced in 1996, and it has been 10 years of successful implementation. Today, all vaccines (with of exceptions) come with VVMs through UN procuremenmt agencies

possible to detect and avoid excessive heat exposure to vaccines when methods are employed to store and transport vaccines w

In order to mark the 10 years of successful implementation of VVMs and acknowledge the efforts put into this device by individu organizations, institutions and manufacturers, a celebration event took place in Geneva on 3 May 2007.

Q339. Pseudotumour like growth in hemophilic arthropathy is commonly seen in A) Hamstring muscle B) Gastrocnemius C) Iliopsoas D) Quadriceps femoris answer is choice c :: iliopsoas intramuscular Hemorrhage: If bleeding into deep muscle masses remains untreated, it may result in adjacent

joint contractures and pressure neuropathy, with the extent of damage depending upon the extent of bleeding and the containm however, similar hematomas in tight fascial compartments may cause significant ischemic myopathy and neuropathy . The iliopsoas is one large muscle in which extensive hemorrhage may occur but produce minimal symptoms.

provided by the various fascial compartments. Small hematomas in large muscles are generally reabsorbed without complication

Emile-Weil described it as the most common site of intramuscular hemorrhage in hemophilia .Significant clinical features include

severe pain in the groin accompanied by marked flexion deformity at the hip, followed by the formation of a mass in the iliac fos

Generally, there is no history of trauma.Bleeding may be severe, extending into the adjacent retroperitoneal structures; a 4-g dr

hemoglobin is not unusual. As pressure increases, thefemoral nerve may be compressed against the inguinal ligament, resulting neuropathy. Large hematomas with progressive enlargement can secondarily affect the bone by pressure necrosis. While we havenot seen a simultaneous bilateral iliopsoas hemorrhage, it is not unusual for each of the two sides to bleed during separate episodes.Any hemorrhage within the psoas or iliopsoas muscle is likely to cause asymmetric enlargement. Q340. Thurston- holland sign is seen in ; 1. oblique fracture of lower 1/3 of humerus. 2. reverse oblique fracture of intertrochanterisc femur. 3. salter-harris type2 fracture. a. coronal fracture of femoral condyles. Q341. Turn buckle splint is for ;; 1. brachual plexus palsy. 2. V.I.C (ans) 3. ulnar n palsy 4. radial n palsy Facts about Compartment Syndrome :- Most common fracture causing Compartment syndrome/VIC = Supracondylar # - Most common compartment = Deep flexor fore-arm - Most common muscle involved = FDP - Most common nerve involved = AIN - First symptom = pain out of proportion to injury - First sign = Pain on passive flexion Splint for VIC = Turn Buckle Splint Surgery = Maxpage Muscle slide Turn Buckle Cast = Scoliosis Q342. Site for first order bone grafting? a. pelvis b. tibial metaphysis c. medial malleolus d. femoral condyle e. greater trochanter Q343. PRAYERS SIGN IS OBSERVED IN PATIENTS OF :: 1. diabetes mellitus. 2. obstructive jaundice. 3. asthma. 4. mitral stenosis. Seen in Diabetic Cheiroathropathy. Prayer sign in a patient with long-standing type II diabetes mellitus. ...and neurogenic arthropathies, such as tabes dorsalis, syringomyelia Q344. Deformities due to malignancy classified by?

(1) Ollier (2) MESS (3) Massada (4) Tscherne's scherne Soft Tissue Classification Grade 0-Minimal soft tissue damage, indirect violence, simple # pattern Grade 1-Superficial abrasion of contusion caused by pressure from within, mild to mod. # pattern

Grade 2-Deep contaminated abrasions assoc. with localized skin or muscle contusion, impending compartment syndrome, severe injury, severe # Q345. Enchondroma's seen in? (1) MaCune Albright Syndrome (2) Maffucie's syndrome (3) Ollier's syndrome (4) Goltz syndrome (5) Terry's Syndrome Maffucci syndrome is a rare genetic disorder that affects both males and females. Maffucci syndrome is characterized by benign

Grade 3-Extensive skin contusion or crush, underlying severe muscle, decompensated compartment syndrome, assoc. major vas

enlargements of cartilage (enchondromas); bone deformities; and dark, irregularly shaped hemangiomas. No racial or sexual pre

is apparent in Maffucci syndrome. No familial pattern of inheritance has been shown, but Maffucci syndrome manifests early in li

usually around age 4-5 years, with 25% of cases being congenital. Maffucci syndrome appears to develop from mesodermal dys early in life. Patients apparently are of average intelligence, and no associated mental or psychiatric abnormalities seem to be pr Ollier disease

Enchondromas are common intraosseous, usually benign cartilaginous tumors, that develop in close proximity to growth plate ca

When multiple enchondromas are present, the condition is called enchondromatosis also known as Ollier disease (WHO terminolo

estimated prevalence of Ollier disease is 1/100,000. Clinical manifestations often appear in the first decade of life. Ollier disease

characterized by an asymmetric distribution of cartilage lesions and these can be extremely variable (in terms of size, number, l evolution of enchondromas, age of onset and of diagnosis, requirement for surgery). Clinical problems caused by enchondromas

skeletal deformities, limb-length discrepancy, and the potential risk for malignant change to chondrosarcoma. The condition in w

multiple enchondromatosis is associated with soft tissue hemangiomas is known as Maffucci syndrome. Until now both Ollier dise

Maffucci syndrome have only occurred in isolated patients and not familial. It remains uncertain whether the disorder is caused b single gene defect or by combinations of (germ-line and/or somatic) mutations. The diagnosis is based on clinical and convention radiological evaluations. Histological analysis has a limited role and is mainly used if malignancy is suspected. There is no medic treatment for enchondromatosis. Surgery is indicated in case of complications (pathological fractures, growth defect, malignant transformation). The prognosis for Ollier disease is difficult to assess. As is generally the case, forms with an early onset appear severe. Enchondromas in Ollier disease present a risk of malignant transformation of enchondromas into chondrosarcomas. Q346. Not a stage of Fracture healing? (1) Stage of Hematome (2) Stage of granulation (3) Stage of Remodelling (4) Stage of Modelling (5) Stage of Consolidation Q347. Non-union never seen in? (1) #clavicle (2) Colle's # (3) Inter-trochanteric # (4) Tibial # (5) SupraCondylar #

Q348. Most common muscle involved in Myositis ossfican's? (1) Anconeus (2) Brachialis (3) Brachioradialis (4) Biceps Brachii Q349. Most common joing replacement affected by Myositis Ossificans? (1) Shoulder (2) Elbow (3) Hip (4) Knee Q350. Most common cause of death after THR? (1) MI (2) SEPSIS (3) PULMONARY EMBOLISM (4) OTHERS Q351. Milkman's # seen in? (1) Pagets disease (2) Osteomalacia (3) Osteopetrosis (4) Osteosarcoma Q352. Most common joint in Charcots joint? (1) Subtalar (2) Mid-tarsal (3) Inter-tarsal (4) Phalngeal Q353. Drop attacks are seen in a/e a. Dermatomyositis b. Polymyositis c. ALS d. Inclusion BM e. Arnold chiari Q354. Most specific test for Dengue? Q355. Most specific test for Dengue? Q356. Bombay blood group? Q357. Cystic fibrosis show s fecal elastase-1 positivity. T/F Q358. Babes nodules are seen in Rabies. T/F Histopathologic evidence of rabies encephalomyelitis (inflammation) in brain tissue and meninges Mononuclear infiltration Perivascular cuffing of lymphocytes or polymorphonuclear cells or inflammation around a blood vessel CDC Lymphocytic foci Babes nodules consisting of glial cells Negri bodies

Q359. Metabolic Syndrome criteria involves raised LDL levels. T/F Q360. Neurexin mutation is seen in Autism. T/F

Posted: Fri Aug 27, 2010 1:24 pm

Post subject:

Q353. Drop attacks are seen in a/e a. Dermatomyositis b. Polymyositis c. ALS d. Inclusion BM e. Arnold chiari

Drop attacks are sudden spontaneous falls while standing or walking, with complete recovery in seconds or minutes. There is us recognized loss of consciousness and the event is remembered. It is a symptom, not a diagnosis, and it can have diverse causes high to us. Nevertheless, drop attacks are a very serious problem. DIAGNOSIS OF DROP ATTACKS 1.Drop attacks due to cardiac disorders are similar to brief fainting episodes. They are best diagnosed with "ambulatory event monitoring". 2.Drop attacks due to seizures and related problems are diagnosed with EEG test and its variants. a variety of other tests also

Sheldon (1960) reported that drop accounts accounted for about 1/4 of 500 consecutive falls in older patients. This estimate see

3.Drop attacks due to Meniere's disease, also called "Otolithic Crises of Tumarkin", are diagnosed with an audiogram and ENG. T

4.Drop attacks from. SCD is diagnosed with a VEMP and a CT scan of the temporal bone (if the VEMP is abnormal). 5.Drop attacks due to psychological problems are very difficult to confirm but sometimes this can be established via inpatient EE monitoring. 6.Cataplexy (a variant of narcolepsy) can present as drop attacks. Narcolepsy is diagnosed with EEG. CAUSES Proximal weakness of the legs

- some causes include muscular dystrophy, neurogenic atrophy, polymyositis, inclusion body myositis, myasthenia gravis and sp ischemic attacks

- Duchenne's muscular dystrophy is the most common muscle dystrophy to cause weakness of the hip and knee extensors, whic important for maintaining an erect posture

- rarely, middle-aged patients may develop drop attacks due to isolated quadriceps weakness, secondary to diseases like polymy inclusion body myositis and neurogenic atrophy, and the legs may suddenly give way without warning to cause falls

- intermittent spinal ischemia can produce sudden attacks of transient paraparesis or falls, and causes include syphilitic arteritis, often precipitated by exercise and relieved by rest Neurodegenerative diseases

arteriosclerosis of the distal aorta, adhesive arcachnoiditis, spinal compression due to spinal stenosis or disc herniation; the atta

- patients with Parkinson's disease often fall due to defective postural reflexes, sudden freezing, and uncoordinated turning; thei stooped flexed posture predisposes them to falling forward because their centre of gravity is displaced forward when standing or drug-induced dyskinesias and orthostatic hypotension can also predispose them to falls

- patients with Parkinon's plus syndromes (other akinetic-rigid disorders) are more prone to fall

- falling is the most frequent initial presenting symptom in progressive supranuclear palsy, and PSP patients tend to extend their and fall backwards; stiff legs + wide-based gait + absent postural reflexes + characteristic down-gaze palsy predisposes PSP pa frequent falls

- other neurodegenerative diseases associated with frequent falls include corticobasal degeneration, Huntingdon's chorea, multip system atrophy and Alzheimer's disease Transient ischemic attacks

- the patient with vertebro-basilar artery ischemia can present with a sudden loss of tone in their legs, causing their knees to bu the patient falls powerlessly without protective arm movements; there may be no precipitating head or neck movements or loss ischemia are present (vertigo, diplopia, bluured vision, sensory changes, motor weakness and variable LOC), which suggest the diagnosis Epilepsy

consciousness, and the patient can ususally get up and walk immediately or within one minute; usually other symptoms of brain

- drop attacks can be due to sudden muscle contractions associated with generalized convulsive epilepsy, or a sudden loss of po tone due to atonic epilepsy or epileptic negative myoclonus - the Lennox-Gastaut syndrome consists of a variable mixture of myoclonic, atonic, tonic and abscence seizures; these patients past history of brain damage and mental retardation, and seizures occur between 6 months and 7 years

- atonic epileptic attacks in children can occur when standing or sitting or lying, and there is no aura or postictal state of confusio

sudden drop attacks can cause the patient to collapse like an accordion with triple flexion of the legs and no involvement of the a the patient can usually get up immediately after an attack Various myoclonic syndromes

- mycoclonus is defined as a sudden brief, shock-like muscle movement caused by a brief muscle contraction (positive myoclonu inhibition of a muscle contraction (negative myoclonus) - they can mimic idiopathic drop attacks, and the patient can fall suddenly with buckling of the legs - many patients exhibit repetitive bouncing of the legs when they attempt to stand, due to repetitive negative myoclonus in the support muscles of the trunk and hips Startle reactions

- pathological startle reactions (hyperekplexia) are characterized by an exaggerated motor response to unexpected auditory stim less frequently to visual or somasthetic stimuli - pathologic startle reactions consist of both brief myoclonic jerks and longer tonic spasms

- hyperekplexia may have a variety of causes eg. idiopathic, hereditary, or secondary to acquired brainstem diseases - post-ano encephalopathy, viral or paraneoplastic brainstem encephalitis, multiple sclerosis, sarcoidosis or brainstem strokes - myoclonic jerks occur frequently during the day, but also at night when the patient is asleep, and mainly affect the legs

- sterotyped tonic spasms consist of facial grimacing, flexion of the neck and trunk, flexion or extension of the hips and knees, a backwards; protective reflexes are impossible and injuries common; consciousness in maintained during the fall and recovery is

characteristic abduction of the shoulders with flexion of the elbows => the patient falls forwards rigidly like a log, but sometimes

Cataplexy and paroxysmal kinesigenic choreoathetosis - cataplexy can cause sudden falls without LOC, but the patient cannot speak during the fall - characteristic precipitating factors include laughter or a sudden emotional stimulus

- minor symptoms of cataplexy may consist of a brief drop of the jaw and sagging of the head, but severe attacks can cause loss in all the antigravity muscles => the patient falls powerlessly and loosely and remains immobile for many seconds (longer than hyperekplexia)

- attacks of paroxysmal kinesigenic choreoathetosis are usually precipitated by sudden leg movements during standing or runnin social embarrassment => the patient may suddenly lose control due to tonic spasms or an involuntary choreoathetotic muscle m and fall like a log Focal structural lesions

- mass lesions around the foramen magnum can cause drop attacks (eg. odontoid process fractures, Arnold-Chiari type I malform presents with a stiff neck, spastic paraplegia. Lhermitte's sign and limb paresthesias

and drop attacks can be precipitated by neck movements, coughing or sneezing, or vertical head compression; the drop attack p

- other posterior fossa pathology can also cause drop attacks eg. posterior fossa arachnoid cysts, midline metastatic cerebellar t vermis hemangioblastomas, and obstructive hydrocephalus due to third ventricle cysts Meniere's drop attacks (Tumarkin's otolithic crisis)

- seen in the early or late stages of Meniere's disease when there is a sudden change in endolymphatic pressure which causes en stimulation with a reflex-like vestibulospinal loss of postural tone

- the patient suddenly loses control of balance and falls; the patient does not lose consciousness and he sometimes has the disti feeling of being pushed or thrown to the ground

(* a patient with Wallenberg's syndrome, due to a lateral medullary stroke, may have a similar transient sensation of being pulle pushed to the side, and the patient suddenly falls to the side without realizing that it is his active shift of the centre of gravity lateropulsion - that causes the momentary imbalance) Idiopathic drop attacks - proposed diagnostic criteria include:- "A fall without warning, not associated with loss of consciousness, not apparently due to

malfunction of the legs, not induced by changes of posture or movement of the head, and not accompanied by vertigo or other c sensation, and not associated with myoclonic jerks" - usually appear between the ages 40 - 60 years - patients only fall when standing or walking, and they usually buckle at the knees and fall forwards

- the attacks occur very suddenly and the patient cannot remember if he lost consciousness, or not; the patient can get up imme unless accidental injury occurs (minor upper limb fractures and/or dislocations are common in the elderly) - serial attacks are rare, and spontaneous remission is common (25 - 80%)

als often referred to as Lou Gehrig's disease, is a progressive neurodegenerative disease that attacks nerve cells in the brain and spinal cord.

Early symptoms vary with each individual but usually include tripping, dropping things, abnormal fatigue of the arms and/or legs speech, muscle cramps and twitches and/or uncontrollable periods of laughing or crying.

Q354. Most specific test for Dengue? answer pnrt

NS-1 antigen tests for the dengue antigen whereas the IgM capture Elisa which is the only government approved test for deng

tests for antibodies. ''NS-1 is the fastest and more accurate test for dengue virus. It takes three to four days for the antibodies t disease earlier CDC 2010 Dengue can be diagnosed by isolation of the virus, by serological tests, or by molecular methods. Diagnosis of acute (on-going) or recent dengue infection can be established by testing serum samples during the first 5 days of symptoms and/or early convalescent phase (more than 5 days of symptoms).

and by the time you test it, the disease has either subsided or become serious. But with NS-1 we can test the antigen and pick u

Acute infection with dengue virus is confirmed when the virus is isolated from serum or autopsy tissue specimens, or the specific

virus genome is identified by reverse transcription-polymerase chain reaction (RT-PCR) from serum or plasma, cerebrospinal flui autopsy tissue specimens during an acute febrile illness. Patients who have IgM antibodies to dengue detected in their serum specimen via an IgM antibody capture enzyme-linked immunosorbent assay (MAC-ELISA) and had either 1.) A negative RT-PCR result in the acute phase specimen or 2.) Did not submit an acute phase specimen, are classified as having a recent probable dengue infection. This is due to the fact that IgM antibodies for dengue may remain elevated for 2 to 3 months after the illness. The elevated IgM in a sample could be the result of an infection that occurred 2 to 3 months ago. In addition, there is cross reactivity with other flaviviruses including West Nile virus (WNV), St. Louis encephalitis virus (SLE), Japanese encephalitis virus (JEV) and yellow fever virus (YFV).

So if a patient with suspected dengue infection submits a late acute phase specimen that is negative (e.g., by RT-PCR and MACand they do not submit a convalescent specimen, they are classified as a laboratory-indeterminate case.

MAC-ELISA has become an important tool for routine dengue diagnosis, MAC-ELISA has a sensitivity and specificity of approxima 90% and [snip]%, respectively but only when used five or more days after onset of fever (i.e., in convalescent phase) . Different formats such as capture ELISA, capture ultramicroELISA, dot-ELISA, AuBioDOT IgM capture and dipsticks have been phase of illness algorithm::: a. PCR $ DENV can be detected in the blood (serum) from patients for approximately the first 5 days of symptoms.

developed. Serums, blood on filter paper, and saliva (but not urine) are useful for IgM detection if samples are taken in convales

$Real time RT-PCR assays have been developed and automated; but none of these tests are yet commercially available. Because

antibodies are detected later, RT-PCR has become a primary tool to detect virus early in the course of illness. Current tests are b 80-90% sensitive, and more that 95% specific.

$ A positive PCR result is a definite proof of current infection and it usually confirms the infecting serotype as well. However, a n result is interpreted as indeterminate. b.MAC ELISA

IgM antibody capture ELISA (MAC-ELISA) format is most commonly employed in diagnostic laboratories and commercial availabl the addition of dengue virus specific antigen (DENV1-4) antigens used for this assay are derived from the envelope protein of the virus limitation of this testing is the cross reactivity between other circulating flaviviruses c.IgG ELISA

diagnostic kits. The assay is based on capturing human IgM antibodies on a microtiter plate using anti-human-IgM antibody follo

The IgG ELISA used for the detection of a past dengue infection utilizes the same viral antigens as the MAC ELISA. This assay co with the hemagglutination assay (HI) previously used. IgG ELISA lacks specificity within the flavivirus serocomplex groups

differentiate primary n secondary Primary versus secondary dengue infection can be determined using a simple algorithm. Samples with a negative IgG in the acu and a positive IgG in the convalescent phase of the infection are primary dengue infections

Samples with a positive IgG in the acute phase and a 4 fold rise in IgG titer in the convalescent phase (with at least a 7 day inte between the two samples) is a secondary dengue infection. d. NS1 ELISA The non-structural protein 1 (NS1) of the dengue viral genome has been shown to be useful as a tool for the diagnosis of acute (DPO), and up to 18 DPO e. PRNT

infections. Dengue NS1 antigen has been detected in the serum of DENV infected patients as early as 1 day post onset of sympt

Plaque Reduction and Neutralization Test (PRNT) and the microneutralization PRNT can be used when a serological specific diagn required, as this assay is the most specific serological tool for the determination of dengue antibodies The PRNT test is used to determine the infecting serotype in convalescent sera microneutralization assay is based on the same principle however instead of counting the number of plaques per well the assay colorimetric measurement of the virus induced cell lysis to determine the end-point dilution. This assay was developed to utilize reagents and for high throughput purposes for larger number of samples for testing.

Q355. 355:: Stool Osmolal gap?

Stool Osmolal Gap = Stool Osm - (2 * (Na + K) ) The 290 is the value of the stool osmolality

Causes of osmotic diarrhea include:{ sog >100:: i e high} Bile salt deficiency Pancreatic insufficiency Celiac/Tropical Sprue Whipple's Dz Intestinal Lymphoma Medications Lactose Intolerance Laxative abuse (depending on the type of laxative) Causes of secretory diarrhea include:{sog <100::: i.e low} Laxative abuse (depending on the type of laxative) Hormonal, Endocrine Tumors

Q356. Bombay blood group?

Bombay blood group is a type of blood group which is present in people who are of O+ve blood group. O+ve and Bombay O+ve groups are different. Bombay O+ve people are very less in population. There is no A, B, H antigen in the blood of these people. doctors are saying. People, who are having Bombay O + ve blood group, can donate only to Bombay O+ve people and can only from Bombay O+ve blood group people

antigen present in O+ve group people is not there in the Bombay O+ve s. Instead of H antigen they have H antibody, this is wh

detection:::There is a serum grouping also called as reverse grouping (accurate test of a persons ABO group i.e. O group). If th

conducting then only we can detect the presence of H antibody, which indicate Bombay O+ve blood group. This test is conducted the help of a reagent called H Lectin. We can find these people mostly in Maharashtra and some places of Karnataka which is boarder of Maharashtra.

Dr. Bhende discovered Bombay O+ve blood group in 1952 at Mumbai. This is the reason why this blood group got the name Bom O+ve blood group

Posted: Fri Aug 27, 2010 1:29 pm

Post subject:

Q360. Neurexin mutation is seen in Autism. T/F it is seen in autism spectrum diseases-true Cell adhesion represents the most direct way of coordinating synaptic connectivity in the brain. Recent evidence highlights the importance of a trans-synaptic interaction between a. postsynaptic neuroligins and b. presynaptic neurexins.

These transmembrane molecules bind each other extracellularly to promote adhesion between dendrites and axons. This signals

recruitment of presynaptic and postsynaptic molecules to form a functional synapse. Remarkably, neuroligins alone can induce t formation of fully functional presynaptic terminals in contacting axons. Conversely, neurexins alone can induce postsynaptic differentiation and clustering of receptors in dendrites.

Therefore, the neuroliginneurexin interaction has the unique ability to act as a bi-directional trigger of synapse formation.

Many studies have supported a genetic etiology for autism. Mutations in two X-linked genes encoding neuroligins NLGN3 and NL siblings with autism-spectrum disorders.

These mutations affect cell-adhesion molecules localized at the synapse and suggest that a defect of synaptogenesis may predis autism. Q361. which of the following CANNOT cause pseudo tumor cerebri:: a. Iron def b. Polio c. GBS d. Chronic Otitis media e. Oudated Salicylic acid Q362. Blood and mucus in stool in a/e a. Yersinia b. Camphylobacter c. Vibrio cholera d. vibrio parahemolyticus e. EIEC answer vibrio cholera and v .parahemolyticus{c & d}

Vibrio parahaemolyticus: Although it is a common worldwide pathogen, in the United States, V. parahaemolyticus infection is res

geographically to the Atlantic and Gulf coasts. The diarrhea is profuse and watery, and blood is not commonly present in the sto others are example of enteroinvasive bacteria

Q363. NORMAL protein CSF levels at 4 weeks infant maybe as high as:: a. 200 mg% b. 150 mg% c. 100mg% d. 70mg% e. 25mg%

The cell and protein content of the cerebrospinal fluid (CSF) and the CSF/serum ratio of glucose, sodium, potassium, calcium, ph

and chloride were investigated in 71 children without cerebromeningeal illness and aged 0-4 weeks, 5-8 weeks, 3-6 months, 7-1

months, 1-6 years and 7-14 years. The protein content of the CSF was 730 +/- 146 (530-950) mg/l for the age group 0-4 week 530 +/- 221 (270-950) mg/l for the age group 5-8 weeks Q364. Agranulocytosis not seen in:: a. Captopril b. Phenylbutazone c. Gold salts d. Cyclosporine

Although all the drugs mentioned can cause AGRANULOCYTOSIS (YES EVEN CAPTOPRIL INDUCED AGRANULOCYTOSIS has been reported) but still it's very rare with captopril as compared with others... So Ans = Captopril Q365. True about CPEO a/e:: a. ragged red fibers

b. single MC sign of mitochon myopathy c. diplopia in all directions d. ptosis may be present

Q366. A 30 year old para 6 delivers vaginally following normal labour with spontaneous delivery of intact placenta.Excesive blee step in the management of thi spatient:a) Packing of the uterus b) Immediate hysterectomy c) Bilateral internal iliac ligation d) IM injn of PGF 2 alpha Q367. Platelet Serotonin is superior to 5HIAA levels in urine for Foregut carcinoid. T/F Q368. Vit B12 massive dose can overcome perniciopus anemia. T/F false

continues,despite manual exploration,bimanual massage,intravenous oxytocin and I.V methergin..which one of the following will

A 2003 study found that oral and sublingual B12 were absorbed equally well in a group of patients with very low B12. In this stu

of the subjects that agreed to undergo the test (5 of 23), had abnormal Schilling tests, but showed no difference in treatment le the other subjects. When oral tablets are used to treat PA, higher-than-normal doses may be needed.The efficacy of using high d

tablets to treat ordinary PA (i.e. anemia due to atrophic gastritis) is well established. Oral supplementation allows B12 to be abs potential to be as effective as injections. Q369. Bulimia Nervosa is associated with anti-social behaviour. T/F Q370. Haller cells can be confused for Agger nasi Cells T/F

places other than the terminal ileum (where B12 absorption usually takes place). A 2006 study found that oral B12 repletion has

Posted: Fri Aug 27, 2010 1:29 pm

Post subject:

Management after Delivery of Placenta Ref:-Williams Obstertrics 22nd Ed The fundus should always be palpated following placental delivery to make certain that the uterus is well contracted. If it is not firm, vigorous fundal massage is indicated. Most often, 20 U of oxytocin in 1000 mL of lactated Ringer or normal saline proves effective when administered intravenously at approximately 10 mL/min (200 mU of oxytocin per minute) simultaneously with effective uterine massage. Oxytocin should never be given as an undiluted bolus dose, because serious hypotension or cardiac arrhythmias may occur Uterine Atony Failure of the uterus to contract properly following delivery is a common cause of obstetrical hemorrhage. In many women, uterine atony can at least be suspected well in advance of delivery (see Table 355). For example, the overdistended uterus is prone to be hypotonic after delivery. Thus, the woman with a large fetus, multiple fetuses, or hydramnios is prone to hemorrhage from uterine atony. The woman whose labor is characterized by uterine activity that is either remarkably vigorous or barely effective is also likely to bleed excessively from uterine atony after delivery. Similarly, labor either initiated or augmented with oxytocin is more likely to be followed by postdelivery uterine atony and hemorrhage. The woman of high parity may be at increased risk for uterine atony. Fuchs and colleagues (1985) described the outcomes of nearly 5800 women para 7 or greater. They reported that the 2.7 percent incidence of postpartum hemorrhage in these women was increased fourfold compared with that of the general obstetrical population. Babinszki and colleagues (1999) reported the incidence of postpartum hemorrhage to be 0.3 percent in women of low parity, but it was 1.9 percent in those para 4 or greater. Another risk is if the woman has previously suffered postpartum hemorrhage. Finally, mismanagement of the third stage of labor involves an attempt to hasten delivery of

the placenta short of manual removal. Constant kneading and squeezing of the uterus that already is contracted likely impedes the physiological mechanism of placental detachment, causing incomplete placental separation and increased blood loss. Ergot Derivatives If oxytocin given by rapid infusion as described in Management after Delivery of Placenta does not prove effective, some clinicians administer intramuscular methylergonovine (0.2 mg). This may stimulate the uterus to contract sufficiently to control hemorrhage. Any superior therapeutic effects of ergot derivatives over oxytocin are speculative, and if these agents are intravenously administered, they may cause dangerous hypertension, especially in women with preeclampsia. Prostaglandins The 15-methyl derivative of prostaglandin F2 (carboprost tromethamine) was approved in the mid-1980s by the U.S. Food and Drug Administration (FDA) for treatment of uterine atony. The initial recommended dose is 250 g (0.25 mg) given intramuscularly, and this is repeated if necessary at 15- to 90-minute intervals up to a maximum of eight doses. Oleen and Mariano (1990) studied use of carboprost for postpartum hemorrhage at 12 cooperating obstetrical units. Arrest of bleeding was considered successful in 208 (88 percent) of 237 women treated. Another 17 women (7 percent) required other oxytocics for control of hemorrhage. The remaining 12 women (5 percent) required surgical intervention. Carboprost is associated with side effects in about 20 percent of women (Oleen and Mariano, 1990). In descending order of frequency, these include diarrhea, hypertension, vomiting, fever, flushing, and tachycardia. We have encountered serious hypertension in a few women so treated. In addition, Hankins and colleagues (1988) observed that intramuscular carboprost was followed by arterial oxygen desaturation that averaged 10 percent and developed within 15 minutes. They concluded that this was due to pulmonary airway and vascular constriction. Rectally administered prostaglandin E2 20-mg suppositories have been used for uterine atony but not studied in clinical trials. A few reports have suggested that misoprostol (Cytotec), a synthetic prostaglandin E1 analogue, may be effective for the treatment of uterine atony (Abdel-Aleem and associates, 2001; O'Brien and colleagues, 1998). In the larger of these uncontrolled studies, misoprostol (1000 g) given rectally was effective in 16 of 18 women unresponsive to usual oxytocics. The mean response time was 1.4 minutes. Misoprostol has also been studied as a potential prophylactic treatment for preventing postpartum hemorrhage. Based on their study of 325 women, Gerstenfeld and Wing (2001) concluded that rectal misoprostol (400 g) was no more effective than intravenous oxytocin in preventing postpartum hemorrhage. Moreover, Villar and co-workers (2002) found in their systematic review that oxytocin and ergot preparations administered during the third stage of labor were more effective than misoprostol for the prevention of postpartum hemorrhage. Bleeding Unresponsive to Oxytocics Continued bleeding after multiple administrations of oxytocics may be from unrecognized genital tract lacerations, including in some cases uterine rupture. Thus, if bleeding persists, no time should be lost in haphazard efforts to control hemorrhage, but the following management should be initiated immediately: 1. Use bimanual uterine compression (Fig. 3516). The technique consists simply of massage of the posterior aspect of the uterus with the abdominal hand and massage through the vagina of the anterior uterine aspect with the other fist. This procedure controls most hemorrhage. 2. Obtain help! 3. Add a second large-bore intravenous catheter so that crystalloid with oxytocin may be continued at the same time blood is given. 4. Begin blood transfusions. The blood group of every obstetrical patient should be known, if possible, before labor and an indirect Coombs test done to detect erythrocyte antibodies. If the latter is negative, then crossmatching of blood is not necessary (see Type and Screen Versus Crossmatch). In an extreme emergency, type O D-negative "universal donor" blood is given. 5. Explore the uterine cavity manually for retained placental fragments or lacerations. 6. Thoroughly inspect the cervix and vagina after adequate exposure. 7. Insert a Foley catheter to monitor urine output, which is a good measure of renal perfusion. Resuscitation is then carried out as described subsequently in Resuscitation and Acute Management. Blood transfusion should be considered in any woman with postpartum hemorrhage in whom abdominal uterine massage and oxytocic agents fail to control the bleeding. With transfusion and simultaneous manual uterine compression and intravenous oxytocin, additional measures are rarely required. Intractable atony may mandate hysterectomy as a lifesaving

measure (see Chap. 25, Indications). Alternatively, uterine artery ligation (Fig. 3517), internal iliac artery ligation (Figs. 3518 and 3519), uterine compression sutures (Fig. 3520), or angiographic embolization as described in Angiographic Embolization may prove successful. Internal Iliac Artery Ligation Ligation of the internal iliac arteries at times reduces the hemorrhage appreciably (Allahbadia, 1993; Clark and colleagues, 1985). However, the procedure may be technically difficult and is successful in less than half of the patients in whom it is attempted (American College of Obstetricians and Gynecologists, 1998). With adequate exposure, ligation is accomplished by opening the peritoneum over the common iliac artery and dissecting down to the bifurcation of the external and internal iliac arteries (see Figs. 3518 and 3519). The areolar sheath covering the internal iliac artery is incised longitudinally, and a right-angle clamp is carefully passed just beneath the artery. Care must be taken not to perforate contiguous large veins, especially the internal iliac vein. Suture, usually nonabsorbable, is then inserted into the open clamp, the jaws are locked, the suture is carried around the vessel, and the vessel is securely ligated. Pulsations in the external iliac artery, if present before tying the ligature, should be present afterward as well. If not, pulsations must be identified after arterial hypotension has been successfully treated to ensure that the blood flow through the external iliac vessel has not been compromised by the ligature. The most important mechanism of action with internal iliac artery ligation is an 85-percent reduction in pulse pressure in those arteries distal to the ligation (Burchell, 1968). This converts an arterial pressure system into one with pressures approaching those in the venous circulation and more amenable to hemostasis via simple clot formation. Bilateral ligation of these arteries does not appear to interfere with subsequent reproduction. Nizard and associates (2003) documented 21 pregnancies in 17 women after bilateral internal iliac artery ligation including three abortions, three miscarriages, two ectopic pregnancies, and 13 normal pregnancies. Uterine Compression Sutures In 1997, B-Lynch and colleagues described a surgical technique performed in five women with severe postpartum hemorrhage in which a pair of vertical brace #2 chromic sutures were secured around the uterus, giving the appearance of suspenders, to compress together the anterior and posterior walls (see Fig. 3520). A simpler modification of the technique has been described by Hayman and associates (2002). We have found this suture to be effective in some cases, however, published experience with these techniques remains limited. Uterine Packing This technique should be considered in women with refractory postpartum hemorrhage related to uterine atony who wish to preserve fertility. Popular during the first half of the 20th century, uterine packing subsequently fell out of favor because of concerns about concealed bleeding and infection (Hsu and co-workers, 2003). Newer techniques, however, have allayed some of these concerns (Roman and Rebarber, 2003). In one technique, the tip of a no. 24F Foley catheter with a 30-mL balloon is guided into the uterine cavity and filled with 60 to 80 mL of saline. The open tip permits continuous drainage from the uterus. If bleeding subsides, the catheter is typically removed after 12 to 24 hours (Roman and Rebarber, 2003). Alternatively, the uterus or pelvis may be packed directly with gauze (Gilstrap, 2002). After hysterectomy, another technique, the umbrella pack, can be constructed of a sterile x-ray cassette bag, filled with gauze rolls knotted together, providing enough volume to fill the pelvis (Howard and co-workers, 2002). The pack is introduced transabdominally with the stalk exiting the vagina. Mild traction is applied by tying the stalk to a one-liter intravenous fluid bag and hanging the bag over the foot of the bed. An indwelling urinary catheter is used to monitor urine output and to prevent urinary obstruction. Placement of percutaneous pelvic drains should be considered to monitor ongoing bleeding within the peritoneal cavity (Dildy, 2002). Broad-spectrum antimicrobials should be administered, and the umbrella pack is removed vaginally after 24 hours.

Posted: Sat Aug 28, 2010 12:29 pm

Post subject:

Q.372 A hemodynamically stable nullliparous patient with ectopic pregnancy has adnexal mass of 2.5 X 3 cms and beta hCG titre miu/ml. What modality treatment is suitable for her: a) Conservative management b) Medical management c) Laparoscopic surgery

d) Laparotomy Q. 373 What is the treatment of choice of unruptured tubal pregnancy with serum beta hcg 2000 iu/l a) Single Dose methotrexate b)Variable Dose c) Expectant Treatment d) Laparoscopic Salpingostomy Q374. UROTENSIN-2 associated with a/e:: a. Diabetes Mellitus b. Palosuran-antagonist c. Renal failure d. Conivaptan-commercial name e. Hypertension Q. 375. Human Colustrum is rich in Lactose T/F Q376. Phenylephrine has little effect on HR of Transplanted heart. T/F Q377. Sumatriptan suppresses both pain and vomiting in migraine like ergotamine. T/F

Posted: Mon Aug 30, 2010 11:24 am

Post subject:

Q. 375. Human Colustrum is rich in Lactose T/F TRUE Table 2510 Composition of Colostrum and Milk.* Units are weight per decilitre(per 100gm) 1 ounce = 28.3 gm Component Human Colostrum Human Milk Cows' Milk Water, g . . . 88 88 Lactose, g 5.3 6.8 5.0 Protein, g 2.7 1.2 3.3 Casein: lactalbumin ratio. . . 1:2 3:1 Fat, g 2.9 3.8 3.7 Linoleic acid . . . 8.3% of fat 1.6% of fat Sodium, mg 92 15 58 Potassium, mg 55 55 138 Chloride, mg 117 43 103 Calcium, mg 31 33 125 Magnesium, mg 4 4 12 Phosphorus, mg 14 15 100 Iron, mg 0.09a 0.15a 0.10a Vitamin A, g 89 53 34 Vitamin D, g . . . 0.03a 0.06a Thiamine, g 15 16 42 Riboflavin, g 30 43 157 Nicotinic acid, g 75 172 85

Ascorbic acid, mg 4.4a 4.3a 1.6a a=square Ref Ganong Q:- Lactose and energy minimal in cow milk acid and vit A Q:- Protein ,fat and calcium max in buffalo milk Q:- Lactose and water max in Human Milk Q:- Vit A max in Human Colostrum Q:- Casein Lactalbumin Ratio:- Imp 1:2 IN Human Milk Q:- Calcium: 1200 per litre or 120 per 100 gm in cow milk,210 IN BUFFALO Q:- Cow Milk:- Adv is more VIT K

Q:- ADV. of cow milk:- Less lactose,max proteins,good minerals,max calcium and phosphorus,max riboflavin and thiamine,less a

Q. 378 A 20 year old young female comes to a male gynaecologist for pelvic examination The consent for examination is : a) Implied consent b) Written. consent c) Verbal consent d) Informed consent Q. 379 Who can commute a death sentence-(JIPMER 80,99) a) High court b) Supreme court c) President d) Any of the above Q. 380. Dying declaration can be received by - (AI89) a) Medical officer b) Lawyer c) Police officer d) All Q381. If a patient survives after having given dying declaration then it stands as - (Jipmer 04) a) Corroborative evidence b) Non valued c) Valid for 48 hours d) None Q382. Law does not consider the following doctrine in a Charge of criminal negligence - (PGI 93, AP 91) a) Vicarious laibility b) Contributory negligence c) Resipsa loquitur d) Novus actus interveniens Q383. imp fatty acid prsnt in brst milk which is imp for growth?? a. dha b. palmitic acid c. linoleic acid d. linolenic acid Docosa Hexanoic Acid - The natural sources of DHA include: (1) Breast milk

(2) Fish oil Commercially it's manufacture is done from the microalgae - Crypthecodinium cohnii and another of the genus Schizochytrium. In pure vegetarians and in individuals not eating sea food, DHA in body is derived from alfa-linolenic acid. DHA is an imp. fatty acid in Sperm, Brain, Retina. Synthesis of DHA from EPA = Spercher's Shunt. DHA in breast milk = 0.07% to 1% with a mean of 0.34% and directly proportional to mother's diet in fish. RDA of DHA for Pregnant and Lactating Women = 300 mg/day Q384. in disc prolapse c5 -6 which of following nerve root pressed?? a. c5 b. c6 c. c7 d. c4

Posted: Tue Aug 31, 2010 12:28 pm

Post subject:

Q385. The age of a 15 year old female is best determined by the radiograph of - (AIIMS 86, PGI 8 7) a) Lower end of Radius and Ulna b) Upper end of humerus c) Upper end of Radius and Ulna d) Xiphisternum Ans is C acc to parikh in females 13-14 yrs -- Elbow 16-17 Yrs -- Wrist for 16 yrs female ofcourse ans is lower end of radius and ulna...Im not sure about 15 yrs...

Both ages have been asked and these ages are imp as before 15 yrs sex with wife even with consent is rape. And below 16 yrs s anyone even with consent is rape but not with wife.

Q 386. the operative procedure known as "microfracture" is done for ? A. delayes union of femur B. nonunion of tibia C. loose bodies of ankle joint D. osteochondral defect of femur Q :387::m/n of smith # is? A. ORIF B. POP in pronation C. CR with below elbow cast D. POP in supination

Q 388::in children, the displacement of fractured fragment in which plane is not remodeled? A. angualtion B. side to side displacement C. rotation D. shortening Q 389::TOC for displaced fracture of radius and ulna in an adult is? A. POP 8wks B. closed reduction and cast bracing C. ORIF--by plating D. Im nailing Q 390::pathological # seen in which stage of paget's? a. phase of repair b. vascular soft phase c. phase of remission d. phase of sarcomatous change

Typical radiologic appearance includes areas of focal bone resorption and formation and a disordered trabecular pattern . * Bone size is expanded and the cortices are thickened. Trabeculae are coarse and widely separated. Density is decreased in the stage and increased in the sclerotic stage. Disease involvement is usually seen at one end of the bone (generally proximal). * A flame-shaped osteolytic wedge may be seen(in vascular stage), as may bony deformities and frank pathologic fractures.

* Vertebrae may have a framed picture appearance. Acetabular involvement may have protrusion. * Pagetic bone may also have small transverse lucencies or pseudofractures, typically on the convex surfaces of the weight-bear bones. Bowing if the femur or tibia may also occur (as above) either with or without pseudofracture. * Only 65% of the lesions seen on bone scan will be seen on x-rays.

Posted: Wed Sep 01, 2010 12:50 pm

Post subject:

Q:: 391:: In NARP there is salt and pepper retina. T/F Q:: 392:: Cannabinidol is used in Pharmacological Preconditioning T/F Q:: 392:: Cannabinidol is used in Pharmacological Preconditioning T/F Q:: 394:: To treat Morphine poisoning acidify the urine. T/F Q:: 395:: H2O in Liquor amnii is recycled every 6 hrs T/F answer:: false water in amniotic fluid is completely changes and replaced by 3 hours.

as you quoted...decreasing amnionic fluid osmolality with advancing gestation.>>>and other point is 250mOsm/l indicates fetal maturity. 396 To detemine the age of a 16 year old girl which of the following is to be rediographed - (PGI88) a) Wrist b) Knee c) Ankle d) Elbow

397. Crural index is calcutated as-(PGI 80, AIIMS 82) a) Length of Radius b) Length of tibia c) Length of Humerus d) Length of ulna 398 .A girl of 10 years wilt have ---- permanent and --temporary teeth - (PGI 79, DNB 90) a) 8,12 b)8,16 c) 12,12 d) 16,8 399. The age of a 15 year old female is best determined by the radiograph of - (AIIMS 86, PGI 8 7) a) Lower end of Radius and Ulna --answer?? NOT SURE b) Upper end of humerus c) Upper end of Radius and Ulna d) Xiphisternum 400. In case of malpractice punishment is given by- (Calcutta 2K) a) State medical council b) MCI C) IMA d) High court 401. Which causes epiphyseal closure? LH FSH Estrogen Testosterone 402. Priapism occurs in - (AIIMS 06) a) Snakebite b) Rat poisoning . c) Cantharide poisoning d) Arsenic poisoning 403. Forceps are applied in all the following except :A. After coming head in breech presentation B. Face presentation C. Occipitoposterior D. Brow presentation 404. Which is false regarding nutritional surveillance a. It is a Diagnostic Method b. Precise weight reading required c. Done in all children less than 5 years d. Done by a trained worker 405. About Delphi method ????

a. Formation of a team to undertake and monitor a Delphi on a given subject. b. Selection of one or more panels to participate in the exercise. Customarily, the panelists are experts in the area to be investig c. Development of the first round Delphi questionnaire d. All are true 406. In full term neonates ,the ductus arteriosus closed by? Cardiac output changes Prostaglandins

high pressure of o2 Low pressure of o2

Posted: Thu Sep 02, 2010 1:05 pm

Post subject:

biosafety level 1 - exposure only to infectious agents that do not ordinarily cause human disease

biosafety level 2 - exposure to infectious agents that can cause disease in humans but whose potential for transmission is limited

biosafety level 3 - exposure to infectious agents that can be transmitted by the respiratory route and which can cause serious in

biosafety level 4 - exposure to exotic infectious agents that pose a high risk of life-threatening disease and can be transmitted a aerosol and for which there is no vaccine or therapy

Posted: Mon Sep 06, 2010 1:07 pm

Post subject:

The following are radiological features of sigmoid vovulus except: a-Inverted U shaped bowel loop b-liver overlap sign c-Bird of prey sign d-Cupola sign........................ans

The cupola sign (1) is seen at supine radiography as an arcuate lucency overlying the lower thoracic spine and projecting caudad

heart (Fig 1). The superior border is well defined and the inferior margin is poorly delineated. The term cupola is used to indicate inverted cupshaped configuration of the lucency

Cupola sign refers to peritoneal air that is superior to the left lobe of the liver, having the appearance of a long, flat hat, hence t "cupola" meaning hat

Posted: Mon Sep 06, 2010 1:21 pm

Post subject:

A 16-yrs-old girl with primary amenorrhea has poor secondary sex characteristics, short stature, normal appearing external gen and elevated serum gonadotropins. Which test would be most appropriate for confirming the diagnosis? A. GnRH stimulation B. Serum prolactin level C. Serum TSH D. Buccal smear blank option D:: Turner presenting with high or "castrate" gonadotrophin levels. this is most probably a case of turner's syndrome.a buccal smear shows absence of barr body.

A buccal smear is a test where cells are taken from the tongue. Cells are collected by scraping the tongue with a spatula. The ce mass seen in a normal female sex chromosome). The buccal smear test can confirm whether the patient is a male or female.

then placed on a slide and the sample is taken to the laboratory for evaluation. The cells are evaluated for the presence of Barr b

You might also like